kysymyksiä -tahdon vastauksia

Olen kysellyt tällä keskustelupalstalla jo muutamaan kertaan muutamia kysymyksiä, mutta en ole koskaan saanut niihin vastausta keneltäkään. Lisään kysymyksiä myöhemmin.

- Miksi ihmisellä on häntäluu? Eikö se ole todiste evoluutiosta?
http://fi.wikipedia.org/wiki/Häntäluu

- Jos Jumala on luonut maailmankaikkeuden tai ihmisen omaksi kuvakseen, miten on Jumala syntynyt? Tyhjiöstäkö se on tänne tullut?

Toteamukseni: Ihminen on luonnut Jumalan, eikä Jumala ole luonnut ihmistä

141

1279

    Vastaukset

    Anonyymi (Kirjaudu / Rekisteröidy)
    5000
    • lusikkasoppaan

      Häntäluu on jäänne, surkastunut elin. Voidaan olettaa ihmisten ja simpanssien esi-isien olleen hännällisiä, mutta hännän muuttuessa tarpeettomaksi se on evoluutiossa kätevästi surkastunut pois. Sen katsotaan olevan yksi todiste myös evoluutiosta siis.

      Jumala ei ole "syntynyt", Jumala ei elä ajassa vaan ikuisuudessa. Jumalalla ei ole alkua ja loppua.

      Nämä vastaukset tuskin sinua miellyttävät, toinen pohjautuu tieteeseen ja toinen uskoon. Itsehän vastasit oman uskosi mukaan tuohon toiseen kysymykseen ja varmasti tiedät myös monia muita vastausmahdollisuuksia.
      Tässä minun "uskiksen" vastaukset tai uskomukset... (se onko kysymyksessä vastaus vai uskomus riippuu tietenkin lukijasta)

      • "Jumala ei ole "syntynyt", Jumala ei elä ajassa vaan ikuisuudessa. Jumalalla ei ole alkua ja loppua. "

        ei voi olla sellaista, jolla ei ole alkua tai loppua eli sellainen on täysin mahdotonta. Onhan sitäkin väitetty että "Jumala olisi kuollut" eli jos luomiskertomus olisi totta, mutta kunnei se pidä paikkansa!


      • tajokinen
        ateisti1982 kirjoitti:

        "Jumala ei ole "syntynyt", Jumala ei elä ajassa vaan ikuisuudessa. Jumalalla ei ole alkua ja loppua. "

        ei voi olla sellaista, jolla ei ole alkua tai loppua eli sellainen on täysin mahdotonta. Onhan sitäkin väitetty että "Jumala olisi kuollut" eli jos luomiskertomus olisi totta, mutta kunnei se pidä paikkansa!

        JA SANA OLI JUMALA
        (rukous) Tule Jeesus ja ilmaise itsesi hänelle
        Sanan kuuleminen autaa ja antaa ymmrtmystä ja snanlukeminen .

        NETISTÄ vaikka Juha Norolampi saarnoja


      • tajokinen kirjoitti:

        JA SANA OLI JUMALA
        (rukous) Tule Jeesus ja ilmaise itsesi hänelle
        Sanan kuuleminen autaa ja antaa ymmrtmystä ja snanlukeminen .

        NETISTÄ vaikka Juha Norolampi saarnoja

        "NETISTÄ vaikka Juha Norolampi saarnoja "

        Koitan just uskovaisten saarnoja nimenomaa välttää.


      • tajokinen kirjoitti:

        JA SANA OLI JUMALA
        (rukous) Tule Jeesus ja ilmaise itsesi hänelle
        Sanan kuuleminen autaa ja antaa ymmrtmystä ja snanlukeminen .

        NETISTÄ vaikka Juha Norolampi saarnoja

        Jumala siis käytti sanaa Tylönä? Tylö on kiuasmerkki. Oliko siis nyt kirjoitusvirhe sanassa "sauna->sana"?

        Oliko tarkoitus kirjoittaa "Alussa oli sauna, ja sauna oli jumalauta Tylö!"
        http://www.tylo.se/
        Onko kaikki siis väärinkäsitystä ja elämä on syntynyt Tylösandilaisessa saunassa!


      • Veripurkki
        ateisti1982 kirjoitti:

        "Jumala ei ole "syntynyt", Jumala ei elä ajassa vaan ikuisuudessa. Jumalalla ei ole alkua ja loppua. "

        ei voi olla sellaista, jolla ei ole alkua tai loppua eli sellainen on täysin mahdotonta. Onhan sitäkin väitetty että "Jumala olisi kuollut" eli jos luomiskertomus olisi totta, mutta kunnei se pidä paikkansa!

        Universumi/multiversumi on saattanut aina olla olemassa.


      • liian yksinkertaista
        ateisti1982 kirjoitti:

        "Jumala ei ole "syntynyt", Jumala ei elä ajassa vaan ikuisuudessa. Jumalalla ei ole alkua ja loppua. "

        ei voi olla sellaista, jolla ei ole alkua tai loppua eli sellainen on täysin mahdotonta. Onhan sitäkin väitetty että "Jumala olisi kuollut" eli jos luomiskertomus olisi totta, mutta kunnei se pidä paikkansa!

        Toisia asioita on vaikea käsittää, varsinkin sellaisia joita emme voi nähdä.
        Saat tästä lisää mietittävää.

        Mitä ajattelet pakastimen toiminnasta, mistä sinne saadaan "kylmää"?
        Vai tarvitseeko "kylmää" tehdä? Olisiko kyse paremminkin lämmön poistamisesta?
        Mistä tuli lämpö? Loiko Jumala lämmönlähteet, jos loi, kylmän on täytynyt aina olla.

        Tämä esimerkki vaan siksi, että me emme välttämättä aina osaa ottaa kaikkia seikkoja huomioon.
        Tämä oli vaan tämmönen "heitto"


      • liian yksinkertaista kirjoitti:

        Toisia asioita on vaikea käsittää, varsinkin sellaisia joita emme voi nähdä.
        Saat tästä lisää mietittävää.

        Mitä ajattelet pakastimen toiminnasta, mistä sinne saadaan "kylmää"?
        Vai tarvitseeko "kylmää" tehdä? Olisiko kyse paremminkin lämmön poistamisesta?
        Mistä tuli lämpö? Loiko Jumala lämmönlähteet, jos loi, kylmän on täytynyt aina olla.

        Tämä esimerkki vaan siksi, että me emme välttämättä aina osaa ottaa kaikkia seikkoja huomioon.
        Tämä oli vaan tämmönen "heitto"

        "Mitä ajattelet pakastimen toiminnasta, mistä sinne saadaan "kylmää"?
        Vai tarvitseeko "kylmää" tehdä? Olisiko kyse paremminkin lämmön poistamisesta?
        Mistä tuli lämpö? Loiko Jumala lämmönlähteet, jos loi, kylmän on täytynyt aina olla. "

        Eiköhän pakastin ole ihan ihmisen keksintö? Pakastimen lämpöpumppu toimii sähköllä ja Jumalasi polkee polkupyörää jossa on generaattori ja samalla se pitää kynttilässä vissiin tulenkin, niinkö se meni?! ;)


      • toinen toope
        ateisti1982 kirjoitti:

        "Mitä ajattelet pakastimen toiminnasta, mistä sinne saadaan "kylmää"?
        Vai tarvitseeko "kylmää" tehdä? Olisiko kyse paremminkin lämmön poistamisesta?
        Mistä tuli lämpö? Loiko Jumala lämmönlähteet, jos loi, kylmän on täytynyt aina olla. "

        Eiköhän pakastin ole ihan ihmisen keksintö? Pakastimen lämpöpumppu toimii sähköllä ja Jumalasi polkee polkupyörää jossa on generaattori ja samalla se pitää kynttilässä vissiin tulenkin, niinkö se meni?! ;)

        Mistäs se kylmä tuli


      • toinen toope kirjoitti:

        Mistäs se kylmä tuli

        Kylmä on ihan defaulttina aina, jollei mikään lämmitä.


    • Onko niin että et ole saanut vastausta vai niin että et ole saanut haluamasi laista vastausta?

      Ensimmäinen kysymys liittyy varsinaisesti evoluutiotieteeseen ja sitä varmaan kannattaa kysyä ensisijaisesti sen tieteenalan tutkijoilta. Oman käsitykseni mukaan näin asiaa on tulkittu.

      Kun siirrytään aikaan ennen alkuräjähdystä (tai muunlaista alkua siinä tapauksessa että tämä teoria ei ole se lopullinen totuus) päädymme jokatapauksessa mainitsemaasi tyhjiöön. Kysymyksesi voi siis esittää muodossa... mistä maailmankaikkeus ja aine on syntynyt? Tyhjiöstäkö? Tai toinen tähän vertautuva asia on se miten joku joka on alkanut tietystä pisteestä räjähdyksessä voi olla ääretön niinkuin maailmankaikkeuden sanotaan olevan?

      Jukka Karlsson
      Kirkon nuorisotyönohjaaja

      • fysiikkaa

        universumin ominaisuus, ja jos alkuräjähdysteoria pitää paikkansa, puhuminen ajasta ennen sitä on yhtä mieletöntä kuin miettiä, mitä on pohjoisnavalta pohjoiseen (Hawking).

        Aine on kasautunutta energiaa ja muodostaa painovoiman kanssa nollasummapelin. Voi siis olla syntynyt aivan tyhjästä.


      • "Ensimmäinen kysymys liittyy varsinaisesti evoluutiotieteeseen ja sitä varmaan kannattaa kysyä ensisijaisesti sen tieteenalan tutkijoilta. Oman käsitykseni mukaan näin asiaa on tulkittu. "

        Pitääkö Jukka sun venkoilla, että muka tulkittu? Ennemminkin tutkittu kuin tulkittu. Missä kohtaan tiedettä on sinusta tulkittu ja luomiskertomusta tutkittu?

        Kysyn nyt vielä kerran: "Miksi ihmisellä on häntäluu? Eikö se ole todiste evoluutiosta?", kun kerran pystytte niin varmalla kädellä puhumaan luomiskertomuksen puolesta, jolla ei ole hitustakaan todistetta missään muuta kuin uskovaisten omassa päässään tai satukirjassa nimeltä Raamattu.

        Kirkolla ei ole minkäänlaista kantaa tiedettä kohtaan? Vain teologia on teistä yksinomaan tiede, joka minulle se ei tiedettä ole nähnytkään! Uskovaiset oikeastaan tahtovat herjata muuta tiedettä jonka, mutta annappa olla kun joku herjaa, pilkkaa tai kyseenalaistaa Raamattua/Jumalaa niin siitä nousee suunnaton poru ja kohu.


      • tutkailija_

        Joo tuo aika ennen alkuräjähdystä ei ole mielekäs, sillä alkuräjähdyksestä alkoi aika.


      • cei5

      • kasvattaja
        ateisti1982 kirjoitti:

        "Ensimmäinen kysymys liittyy varsinaisesti evoluutiotieteeseen ja sitä varmaan kannattaa kysyä ensisijaisesti sen tieteenalan tutkijoilta. Oman käsitykseni mukaan näin asiaa on tulkittu. "

        Pitääkö Jukka sun venkoilla, että muka tulkittu? Ennemminkin tutkittu kuin tulkittu. Missä kohtaan tiedettä on sinusta tulkittu ja luomiskertomusta tutkittu?

        Kysyn nyt vielä kerran: "Miksi ihmisellä on häntäluu? Eikö se ole todiste evoluutiosta?", kun kerran pystytte niin varmalla kädellä puhumaan luomiskertomuksen puolesta, jolla ei ole hitustakaan todistetta missään muuta kuin uskovaisten omassa päässään tai satukirjassa nimeltä Raamattu.

        Kirkolla ei ole minkäänlaista kantaa tiedettä kohtaan? Vain teologia on teistä yksinomaan tiede, joka minulle se ei tiedettä ole nähnytkään! Uskovaiset oikeastaan tahtovat herjata muuta tiedettä jonka, mutta annappa olla kun joku herjaa, pilkkaa tai kyseenalaistaa Raamattua/Jumalaa niin siitä nousee suunnaton poru ja kohu.

        Jos edes vähän viitsisi kysymyksensä taustaan tutustua, voi kysymykseensä saada edes perusvastauksen. Tässä tapauksessa kirkon suhteesta evoluutiotieteeseen.

        Kopioin sen kuitenkin vaivaasi säästääkseni tänne, myös linkki alhaalla.
        www.evl.fi

        "Evoluutio
        Aamenesta öylättiin - kirkon ja uskon sanasto
        Loikkaa: valikkoon, hakuun

        Evoluutio (lat. evolutio) merkitsee kehitystä. Englantilainen Charles Darwin (1809–1882) loi evoluutioteorian eli opin lajien kehityksestä. Teoksessaan Lajien synty (1859) Darwin kumosi empiirisen todistusmateriaalin avulla vanhan opin lajien muuttumattomuudesta.

        Darwinin kuollessa johtavat biologit olivat jo omaksuneet hänen käsityksensä, ja vaikka Darwinin teoriaa kehityksen tekijöistä on myöhemmin korjattu, hänen käsitystään lajien kehittymisestä ei ole laajassa mitassa tieteellisesti kyseenalaistettu.

        Aikanaan Darwinin evoluutioteoria herätti kiivaan keskustelun, johon toisella puolen osallistuivat radikaalit, vapaa-ajattelijat ja antiklerikaalit, toisella puolen traditionalistiset, biblisistiset ja fundamentalistiset anglikaanit, protestantit ja roomalaiskatoliset.

        Luterilainen kirkko ei katso evoluutioteorian luonnontieteellisenä selitysmallina olevan ristiriidassa luomisuskon kanssa. Luomisusko on kokonaisvaltainen uskonnollinen tulkinta elämän ihmeestä ja sen synnystä. Evoluutioteoria on totena pidetty luonnontieteellinen selitysmalli lajien kehittymisestä.
        Haettu osoitteesta http://www.evl2.fi/sanasto/index.php/Evoluutio"


        Kuten luit, kirkko pitää evoluutioteoriaa tieteellisenä selitysmallina lajien kehittymisestä ja taas kirkko tarkastelee olevaisuuden kehittymistä uskonnollisen selitysmallin kautta, jotka todellakaan eivät ole ristiriidassa keskenään. Mitä johtopäätöksiä ja tulkintoja näistä kukin tekee, riippuu ihmisen katsomuksesta. Toisinaan oma asenteellinen katsomus sumentaa näkökykyä niin, että ei näe muita selitysmalleja ja tämän pohjalta pyrkii tukemaan omaa katsomustaan.


      • tietävä ateisti
        kasvattaja kirjoitti:

        Jos edes vähän viitsisi kysymyksensä taustaan tutustua, voi kysymykseensä saada edes perusvastauksen. Tässä tapauksessa kirkon suhteesta evoluutiotieteeseen.

        Kopioin sen kuitenkin vaivaasi säästääkseni tänne, myös linkki alhaalla.
        www.evl.fi

        "Evoluutio
        Aamenesta öylättiin - kirkon ja uskon sanasto
        Loikkaa: valikkoon, hakuun

        Evoluutio (lat. evolutio) merkitsee kehitystä. Englantilainen Charles Darwin (1809–1882) loi evoluutioteorian eli opin lajien kehityksestä. Teoksessaan Lajien synty (1859) Darwin kumosi empiirisen todistusmateriaalin avulla vanhan opin lajien muuttumattomuudesta.

        Darwinin kuollessa johtavat biologit olivat jo omaksuneet hänen käsityksensä, ja vaikka Darwinin teoriaa kehityksen tekijöistä on myöhemmin korjattu, hänen käsitystään lajien kehittymisestä ei ole laajassa mitassa tieteellisesti kyseenalaistettu.

        Aikanaan Darwinin evoluutioteoria herätti kiivaan keskustelun, johon toisella puolen osallistuivat radikaalit, vapaa-ajattelijat ja antiklerikaalit, toisella puolen traditionalistiset, biblisistiset ja fundamentalistiset anglikaanit, protestantit ja roomalaiskatoliset.

        Luterilainen kirkko ei katso evoluutioteorian luonnontieteellisenä selitysmallina olevan ristiriidassa luomisuskon kanssa. Luomisusko on kokonaisvaltainen uskonnollinen tulkinta elämän ihmeestä ja sen synnystä. Evoluutioteoria on totena pidetty luonnontieteellinen selitysmalli lajien kehittymisestä.
        Haettu osoitteesta http://www.evl2.fi/sanasto/index.php/Evoluutio"


        Kuten luit, kirkko pitää evoluutioteoriaa tieteellisenä selitysmallina lajien kehittymisestä ja taas kirkko tarkastelee olevaisuuden kehittymistä uskonnollisen selitysmallin kautta, jotka todellakaan eivät ole ristiriidassa keskenään. Mitä johtopäätöksiä ja tulkintoja näistä kukin tekee, riippuu ihmisen katsomuksesta. Toisinaan oma asenteellinen katsomus sumentaa näkökykyä niin, että ei näe muita selitysmalleja ja tämän pohjalta pyrkii tukemaan omaa katsomustaan.

        mukaan ihminen oli luotu fyysisesti kuolemattomaksi ja vasta syntiinlankeemus toi hänelle fyysisen kuoleman. Sen kuoleman Jeesus voitti ylösnousemuksellaan.

        Tutustukaa niihin kirkkoisiin, joiden oppeja ei hylätty uskonpuhdistuksessa.


      • jason_dax

        "Kun siirrytään aikaan ennen alkuräjähdystä..."

        Emme siirry, koska mainitsemaasi tilaa ei ole. Aika ja avaruus ovat yhtä, ei ole yhtä ilma toista. Ei ole ajanhetkeä, jolloin maailmankaikkeutta ei olisi - maailmankaikkeus on aina ollut olemassa.


      • veripurkki
        kasvattaja kirjoitti:

        Jos edes vähän viitsisi kysymyksensä taustaan tutustua, voi kysymykseensä saada edes perusvastauksen. Tässä tapauksessa kirkon suhteesta evoluutiotieteeseen.

        Kopioin sen kuitenkin vaivaasi säästääkseni tänne, myös linkki alhaalla.
        www.evl.fi

        "Evoluutio
        Aamenesta öylättiin - kirkon ja uskon sanasto
        Loikkaa: valikkoon, hakuun

        Evoluutio (lat. evolutio) merkitsee kehitystä. Englantilainen Charles Darwin (1809–1882) loi evoluutioteorian eli opin lajien kehityksestä. Teoksessaan Lajien synty (1859) Darwin kumosi empiirisen todistusmateriaalin avulla vanhan opin lajien muuttumattomuudesta.

        Darwinin kuollessa johtavat biologit olivat jo omaksuneet hänen käsityksensä, ja vaikka Darwinin teoriaa kehityksen tekijöistä on myöhemmin korjattu, hänen käsitystään lajien kehittymisestä ei ole laajassa mitassa tieteellisesti kyseenalaistettu.

        Aikanaan Darwinin evoluutioteoria herätti kiivaan keskustelun, johon toisella puolen osallistuivat radikaalit, vapaa-ajattelijat ja antiklerikaalit, toisella puolen traditionalistiset, biblisistiset ja fundamentalistiset anglikaanit, protestantit ja roomalaiskatoliset.

        Luterilainen kirkko ei katso evoluutioteorian luonnontieteellisenä selitysmallina olevan ristiriidassa luomisuskon kanssa. Luomisusko on kokonaisvaltainen uskonnollinen tulkinta elämän ihmeestä ja sen synnystä. Evoluutioteoria on totena pidetty luonnontieteellinen selitysmalli lajien kehittymisestä.
        Haettu osoitteesta http://www.evl2.fi/sanasto/index.php/Evoluutio"


        Kuten luit, kirkko pitää evoluutioteoriaa tieteellisenä selitysmallina lajien kehittymisestä ja taas kirkko tarkastelee olevaisuuden kehittymistä uskonnollisen selitysmallin kautta, jotka todellakaan eivät ole ristiriidassa keskenään. Mitä johtopäätöksiä ja tulkintoja näistä kukin tekee, riippuu ihmisen katsomuksesta. Toisinaan oma asenteellinen katsomus sumentaa näkökykyä niin, että ei näe muita selitysmalleja ja tämän pohjalta pyrkii tukemaan omaa katsomustaan.

        En ymmärrä sitä, miten uskonnollinen ja tieteellinen selitysmalli voivat elää yhtäaikaa ollessaan ristiriidassa. Molemmat käsittelevät todellisuutta, joten vain yksi voi olla oikeassa. Varsinkin kun empiiriset todisteet tukevat toista. Evoluutio on fakta, evoluutioteoria selittää miten evoluutio toimii.

        Evoluutiossa ei ole kyse elämän synnystä vaan sen kehityksestä. Elämän alkuun uskonto voi aukkojen jumalansa kanssa vielä pesiytyä mutta ei elämän kehitykseen, siinä sille ei ole enää tilaa.


    • HEKO HEKO KÖH

      PUAAAHHHHHHHH ... HEH HEH HEH ... JUMALA ON SATUA JA HÖPÖTYSTÄ.

      • Harmittaako ?

        sen tiedät ?
        Yksi iso mies 5,5 milj Suomalaisesta.

        1 vastaan 5,5 milj.


      • Harmittaako ? kirjoitti:

        sen tiedät ?
        Yksi iso mies 5,5 milj Suomalaisesta.

        1 vastaan 5,5 milj.

        Aika harvassa ovat jumaliin uskovat Suomessa. Oikeasti.


      • Mistäs sinä tiedät?
        UskonnotonLähimmäinen kirjoitti:

        Aika harvassa ovat jumaliin uskovat Suomessa. Oikeasti.

        Eivät kaikki tunnusta uskoaan Jumalaan ja kuitenkin sisimmässään uskovat.


      • Mistäs sinä tiedät? kirjoitti:

        Eivät kaikki tunnusta uskoaan Jumalaan ja kuitenkin sisimmässään uskovat.

        Ja mistäs sinä muiden ajatukset tiedät?


    • Miten selitätte ihmisten viisaudenhampaat? Evoluutio?

      Miten selitätte sen että on olemassa monia kissaeläimiä, kuten esim. muinainen sapellihammastiikeri ja nykyinen tiikeri. Kaikki nykyiset kissaeläimet muistuttavat toisiaan, kuten ihminenkin apinoita.

      Miten voi olla mahdollista, että TUTKIMUSTEN mukaan simpanssilla ja ihmisellä on yli 95% samankaltainen geeniperimä, mikäli evoluutio ei olisi mahdollista. 4-5% todellakin on se erotus, johon liittyy muuttujia ihmisestä simpanssista.

      Oliko se joku nuorisotyöntekijä, joka tahtoi todistaa että mitä jos verrataan ihmistä ja kastematoa, niin kuinka paljon siinä on eroavaisuuksia. Ei voi olla totta. :) Miten kastemato liittyy apinan ja ihmisen yhteiseen kantaisään? Ollaanko me sukuja lieroille!? :D

      Miten selitätte dinosaurusten olemassa olon, joiden "jälkeläisinä" ollaan todettu linnut?
      http://fi.wikipedia.org/wiki/Dinosaurukset


      Teillä täytyy olla nuo asiat selvillä, jotta voisitte tuomita edes evoluution "heikoksi" ja luomiskertomuksen paikkansa pitävimmäksi!?

      HUOM! Vastaukset mielummin muualta kuin Raamatusta, kiitos!

      • vain_ihminen

        En edelleenkään ymmärrä, miksi ajattelet etteivät kirkon työntekijät hyväksy evoluutioteoriaa?
        Omassa tuttavapiirissäni, jossa sattuu olemaan laaja kirjo erilaisia kirkon työntekijöitä, on tästä asiasta keskusteltu ja siinä tullut selväksi että usko ei sulje pois tiedettä millään tavalla. Rippikouluissakin opetetaan että tiede kertoo sen miten asiat ovat täällä maapallolla tapahtuneet.

        Vai onko tämä koko hulabaloo vain seurausta siitä, että joku kirkon työntekijöistä on sanonut evoluutioteoriankin olevan lopulta vain teoria?


      • cei5
        vain_ihminen kirjoitti:

        En edelleenkään ymmärrä, miksi ajattelet etteivät kirkon työntekijät hyväksy evoluutioteoriaa?
        Omassa tuttavapiirissäni, jossa sattuu olemaan laaja kirjo erilaisia kirkon työntekijöitä, on tästä asiasta keskusteltu ja siinä tullut selväksi että usko ei sulje pois tiedettä millään tavalla. Rippikouluissakin opetetaan että tiede kertoo sen miten asiat ovat täällä maapallolla tapahtuneet.

        Vai onko tämä koko hulabaloo vain seurausta siitä, että joku kirkon työntekijöistä on sanonut evoluutioteoriankin olevan lopulta vain teoria?

        jos te uskotte evoluutioteoriaan, niin teidän luomiskertomukseltanne ja niin ollen myös syntiinlankeemuskertomukselta menee totaalisesti pohja.

        Uskotko sinä painovoimateoriaan? Sehän on "vain" teoria. Menepäs hyppimään kuudennesta kerroksesta ja katsotaan kuinka "vain" se teoria on.


      • vain_ihminen kirjoitti:

        En edelleenkään ymmärrä, miksi ajattelet etteivät kirkon työntekijät hyväksy evoluutioteoriaa?
        Omassa tuttavapiirissäni, jossa sattuu olemaan laaja kirjo erilaisia kirkon työntekijöitä, on tästä asiasta keskusteltu ja siinä tullut selväksi että usko ei sulje pois tiedettä millään tavalla. Rippikouluissakin opetetaan että tiede kertoo sen miten asiat ovat täällä maapallolla tapahtuneet.

        Vai onko tämä koko hulabaloo vain seurausta siitä, että joku kirkon työntekijöistä on sanonut evoluutioteoriankin olevan lopulta vain teoria?

        "En edelleenkään ymmärrä, miksi ajattelet etteivät kirkon työntekijät hyväksy evoluutioteoriaa? "

        Siksi koska on olemassa kirkko ja kirkko velvoittaa uskomaan satukirjaan nimeltä Raamattu, kuten myös sitoo kuulumaan kirkkoon.

        "Rippikouluissakin opetetaan että tiede kertoo sen miten asiat ovat täällä maapallolla tapahtuneet. "

        En ole koskaan kuullut, että rippikouluissa opetetaan tiedeaineitakin!? Vai onko teologia sinusta tiede, että asiat ovat tapahtuneet maapallolla niin kuten teologi väittää uskonnon avulla tapahtuneen. Ei muistini mukaan rippikoulussa puhuttu mistään alkuräjähdyksestä vaan luomisopista, joka ei ole ikävä kyllä tiede!

        "Vai onko tämä koko hulabaloo vain seurausta siitä, että joku kirkon työntekijöistä on sanonut evoluutioteoriankin olevan lopulta vain teoria? "

        Heidän pitää jo virkansa puolesta todeta, että evoluutio on vain teoria, joka ei lyö vertoja luomiskertomukselle. Niin ja se, joka luomiskertomuksen kyseenalaistaa, surmattakoon! :)


      • vain_ihminen
        cei5 kirjoitti:

        jos te uskotte evoluutioteoriaan, niin teidän luomiskertomukseltanne ja niin ollen myös syntiinlankeemuskertomukselta menee totaalisesti pohja.

        Uskotko sinä painovoimateoriaan? Sehän on "vain" teoria. Menepäs hyppimään kuudennesta kerroksesta ja katsotaan kuinka "vain" se teoria on.

        Monen mielestä se ei vie pohjaa pois noista kertomuksista, ei myöskään minun mielestäni. Kysehän on sen ajan ihmisten kokemuksesta ja tavasta selittää maailmaa omista lähtökohdistaan käsin (ja tuo kehityksen kulku kertomuksessakin on jossain määrin yhtenevä, ihminen on majaillut maapallolla vain vähän aikaa).

        Ja itse tosiaan ajattelen syntiinlankeemuskertomuksen olevan enemmänkin vertauskuvallinen, se on jotain mikä tapahtuu ihmisessä itsessään joka päivä... ei jotain mikä on tapahtunut vuosituhansia sitten. Joskus tuntuu, että nimenomaan ateistit haluavat tulkita Raamatun sanaa kirjaimellisesti sillä tahtoa syvällisempään tulkintaan ei ole.

        Minä luotan tieteen tutkimuksiin ja tuloksiin. Joten uskon evoluutioteoriaan kuin myös painovoimateoriaan.


      • vain_ihminen kirjoitti:

        Monen mielestä se ei vie pohjaa pois noista kertomuksista, ei myöskään minun mielestäni. Kysehän on sen ajan ihmisten kokemuksesta ja tavasta selittää maailmaa omista lähtökohdistaan käsin (ja tuo kehityksen kulku kertomuksessakin on jossain määrin yhtenevä, ihminen on majaillut maapallolla vain vähän aikaa).

        Ja itse tosiaan ajattelen syntiinlankeemuskertomuksen olevan enemmänkin vertauskuvallinen, se on jotain mikä tapahtuu ihmisessä itsessään joka päivä... ei jotain mikä on tapahtunut vuosituhansia sitten. Joskus tuntuu, että nimenomaan ateistit haluavat tulkita Raamatun sanaa kirjaimellisesti sillä tahtoa syvällisempään tulkintaan ei ole.

        Minä luotan tieteen tutkimuksiin ja tuloksiin. Joten uskon evoluutioteoriaan kuin myös painovoimateoriaan.

        "Monen mielestä se ei vie pohjaa pois noista kertomuksista, ei myöskään minun mielestäni. Kysehän on sen ajan ihmisten kokemuksesta ja tavasta selittää maailmaa omista lähtökohdistaan käsin (ja tuo kehityksen kulku kertomuksessakin on jossain määrin yhtenevä, ihminen on majaillut maapallolla vain vähän aikaa). "

        Joka selittää vielä monien nykyisten ihmisten kokemuksia ja tapoja, vaikka eivät olisi samoin koskaan mitään kokeneetkaan. Eikö sellaisia käsityksiä nimeenomaa tulisi mitenkään kyseenalaistaa, jota ei ole koskaan itse kokenut vaan niistä on joku kertonut.

        Raamatusta on ilmeisemmin lähtenyt terminäkin käytetty sana "huhu" eli uskotaan siihen, mitä muut ovat aikoinaan todenneet ja sen pitäisi riittää todisteeksi. Vähän sama ilmiö, mitä esimerkiksi naapurin akat höpöttävät keskenään jostakin toisesta naapurista, eikä lopulta yksikään asia pidä paikkaansa edes osittain. :)


      • vain_ihminen
        ateisti1982 kirjoitti:

        "Monen mielestä se ei vie pohjaa pois noista kertomuksista, ei myöskään minun mielestäni. Kysehän on sen ajan ihmisten kokemuksesta ja tavasta selittää maailmaa omista lähtökohdistaan käsin (ja tuo kehityksen kulku kertomuksessakin on jossain määrin yhtenevä, ihminen on majaillut maapallolla vain vähän aikaa). "

        Joka selittää vielä monien nykyisten ihmisten kokemuksia ja tapoja, vaikka eivät olisi samoin koskaan mitään kokeneetkaan. Eikö sellaisia käsityksiä nimeenomaa tulisi mitenkään kyseenalaistaa, jota ei ole koskaan itse kokenut vaan niistä on joku kertonut.

        Raamatusta on ilmeisemmin lähtenyt terminäkin käytetty sana "huhu" eli uskotaan siihen, mitä muut ovat aikoinaan todenneet ja sen pitäisi riittää todisteeksi. Vähän sama ilmiö, mitä esimerkiksi naapurin akat höpöttävät keskenään jostakin toisesta naapurista, eikä lopulta yksikään asia pidä paikkaansa edes osittain. :)

        Niin, siksi siis ainakin Suomessa iso osa uskovia kuitenkin luottaa tieteen kertomaan siinä, miten elämä on maapallolla syntynyt ja kehittynyt.

        Luomiskertomus taas kertoo jostain ihan muusta kuin tieteellisistä asioista, sen ydinidea (jos niin voi sanoa) on mielestäni vain siinä, että Jumalalla on ollut näppinsä pelissä jollain lailla ja että Jumala on luonut (nykytietämyksen mukaan evoluution kautta) ihmisen omaksi kuvakseen, joten kaikki ihmiset ovat tässä suhteessa yhtä arvokkaita ja tärkeitä.


      • cei5
        vain_ihminen kirjoitti:

        Monen mielestä se ei vie pohjaa pois noista kertomuksista, ei myöskään minun mielestäni. Kysehän on sen ajan ihmisten kokemuksesta ja tavasta selittää maailmaa omista lähtökohdistaan käsin (ja tuo kehityksen kulku kertomuksessakin on jossain määrin yhtenevä, ihminen on majaillut maapallolla vain vähän aikaa).

        Ja itse tosiaan ajattelen syntiinlankeemuskertomuksen olevan enemmänkin vertauskuvallinen, se on jotain mikä tapahtuu ihmisessä itsessään joka päivä... ei jotain mikä on tapahtunut vuosituhansia sitten. Joskus tuntuu, että nimenomaan ateistit haluavat tulkita Raamatun sanaa kirjaimellisesti sillä tahtoa syvällisempään tulkintaan ei ole.

        Minä luotan tieteen tutkimuksiin ja tuloksiin. Joten uskon evoluutioteoriaan kuin myös painovoimateoriaan.

        No siinä tapauksessa et siis voi uskoa jumalan luoneen taivasta ja maata, et pyhän hengen siittäneen Jeesusta, et hänen ylösnousemukseensa, et iankaikkiseen elämään, etkä oikeastaan mihinkään muuhunkaan, mitä raamatussa puhutaan.


      • vain_ihminen
        cei5 kirjoitti:

        No siinä tapauksessa et siis voi uskoa jumalan luoneen taivasta ja maata, et pyhän hengen siittäneen Jeesusta, et hänen ylösnousemukseensa, et iankaikkiseen elämään, etkä oikeastaan mihinkään muuhunkaan, mitä raamatussa puhutaan.

        Mistä niin päättelet?
        Jumala on mielestäni luonnonlakien yläpuolella/ulkopuolella. Kaikkivoipa ja kaikkivaltias. Kyllä minä uskon että minulla on sielu, joku olemus ja henki tämän ruumiin ulkopuolellakin. Sitä nyt ei voi todistaa tieteellisesti, niinkuin ei Jumalaakaan.

        Itse en voi ymmärtää sitä suppeata maailmankatsomusta, jossa kaikki mitä on on kirjaimellista, tieteellisesti todistettavaa tai konkreettisesti havaittavissa. Tai sanotaan mielummin niin, että voin toki ymmärtää, että jollekin muulle olemassaolo on vain matematiikkaa ja konkreettisia havaintoja, että ajatukset ovat hermoimpulsseja, hormonaalisia muutoksia, aivojen sähköistä toimintaa. Mutta en ymmärrä, miksi on vaikea tajuta sitä, että joku muu voi uskoa elämässä olevan jotain muutakin kuin se todellisuus jota voidaan tieteellisesti tutkia.

        Miksi minä en voisi/saisi uskoa sekä Jumalaan ja kristinuskon perussanomaan, sekä samalla myös luottaa tieteen tutkimukseen?


      • cei5
        vain_ihminen kirjoitti:

        Mistä niin päättelet?
        Jumala on mielestäni luonnonlakien yläpuolella/ulkopuolella. Kaikkivoipa ja kaikkivaltias. Kyllä minä uskon että minulla on sielu, joku olemus ja henki tämän ruumiin ulkopuolellakin. Sitä nyt ei voi todistaa tieteellisesti, niinkuin ei Jumalaakaan.

        Itse en voi ymmärtää sitä suppeata maailmankatsomusta, jossa kaikki mitä on on kirjaimellista, tieteellisesti todistettavaa tai konkreettisesti havaittavissa. Tai sanotaan mielummin niin, että voin toki ymmärtää, että jollekin muulle olemassaolo on vain matematiikkaa ja konkreettisia havaintoja, että ajatukset ovat hermoimpulsseja, hormonaalisia muutoksia, aivojen sähköistä toimintaa. Mutta en ymmärrä, miksi on vaikea tajuta sitä, että joku muu voi uskoa elämässä olevan jotain muutakin kuin se todellisuus jota voidaan tieteellisesti tutkia.

        Miksi minä en voisi/saisi uskoa sekä Jumalaan ja kristinuskon perussanomaan, sekä samalla myös luottaa tieteen tutkimukseen?

        Sinä valitset itsellesi mitä haluat. Jos uskot tieteeseen, niin etkö tajua, ettei tarina pyhästä hengestä Jeesuksen siittäjänä voi olla totta? Etkä voi uskoa samanaikaisesti "taivaalliseen ylösnousemukseen" ja biologian todisteisiin.

        Tai voit varmaan, siksihän sinä uskis oletkin. Kaikki käy, mikä itsestä tuntuu kivalta, vaikka ne olisivat kuinka ristiriidassa keskenään.


      • vain_ihminen
        cei5 kirjoitti:

        Sinä valitset itsellesi mitä haluat. Jos uskot tieteeseen, niin etkö tajua, ettei tarina pyhästä hengestä Jeesuksen siittäjänä voi olla totta? Etkä voi uskoa samanaikaisesti "taivaalliseen ylösnousemukseen" ja biologian todisteisiin.

        Tai voit varmaan, siksihän sinä uskis oletkin. Kaikki käy, mikä itsestä tuntuu kivalta, vaikka ne olisivat kuinka ristiriidassa keskenään.

        Biologiako todistaa ettei ihmisen sielu voi "vilahtaa" taivaaseen viimeisenä päivänä?

        Ja mitenkä tuota Pyhän Hengen siittämisominaisuutta on sitten tutkittu tieteellisesti?
        (Uutisissa oli taannoin hauska otsikko, Kuningasboa oli synnyttänyt neitseellisesti kaksikymmentä poikasta) :)


      • Uskonnollinen

        "Miten selitätte ihmisten viisaudenhampaat? Evoluutio?"

        En ole hammaslääkäri enkä tiedä viisaudenhampaista mitään muuta kuin että joillekin ne kasvavat vinoon ja pitää leikkauksella poistaa (kuten minä) ja joillakin ne mahtuvat kasvamaan ihan hyvin (kuten äitini) Ei tällä pitäisi olla evoluution kanssa mitään tekemistä...

        "Miten selitätte sen että on olemassa monia kissaeläimiä, kuten esim. muinainen sapellihammastiikeri ja nykyinen tiikeri. Kaikki nykyiset kissaeläimet muistuttavat toisiaan, kuten ihminenkin apinoita."

        Jumala loi kaikki eläimet, miten Hän sen teki, ei ole tiedossa. Itse ajattelisin että Hän on käyttänyt evoluutiota hyväkseen luomistyössään.

        "Miten voi olla mahdollista, että TUTKIMUSTEN mukaan simpanssilla ja ihmisellä on yli 95% samankaltainen geeniperimä, mikäli evoluutio ei olisi mahdollista. 4-5% todellakin on se erotus, johon liittyy muuttujia ihmisestä simpanssista."

        Uskon vahvasti että Jumala on luonut ihmisen, miten Hän sen teki, en tiedä. En sulje kokonaan pois mahdollisuutta että ihminen on kehitetty simpanssista ja kun kehitys oli valmis, oli ihminen luotu, Adam, joka oli ensimmäinen jota nimitettiin ihmiseksi ja jonka henki tuli taivaasta.

        "Oliko se joku nuorisotyöntekijä, joka tahtoi todistaa että mitä jos verrataan ihmistä ja kastematoa, niin "kuinka paljon siinä on eroavaisuuksia. Ei voi olla totta. :) Miten kastemato liittyy apinan ja ihmisen yhteiseen kantaisään? Ollaanko me sukuja lieroille!? :D""

        Mielestäni ei millään tavalla, tästä en tiedä mitään.

        "Miten selitätte dinosaurusten olemassa olon, joiden "jälkeläisinä" ollaan todettu linnut?
        http://fi.wikipedia.org/wiki/Dinosaurukset"

        Dinosaurukset ovat tietenkin olleet olemassa ja niistä on todennäköisesti kehittynyt muita eläimiä ennen kun ovat hävinneet kokonaan.


        "Teillä täytyy olla nuo asiat selvillä, jotta voisitte tuomita edes evoluution "heikoksi" ja luomiskertomuksen paikkansa pitävimmäksi!?"

        Jep, vaikka en kyllä tuomitse evoluutiota heikoksi, vaan Jumalan välineeksi luomistyössään.

        "HUOM! Vastaukset mielummin muualta kuin Raamatusta, kiitos! "

        Ei tainnut tulla yhtään lainausta Pyhästä Kirjasta :)


      • järkeä kehiin
        vain_ihminen kirjoitti:

        Biologiako todistaa ettei ihmisen sielu voi "vilahtaa" taivaaseen viimeisenä päivänä?

        Ja mitenkä tuota Pyhän Hengen siittämisominaisuutta on sitten tutkittu tieteellisesti?
        (Uutisissa oli taannoin hauska otsikko, Kuningasboa oli synnyttänyt neitseellisesti kaksikymmentä poikasta) :)

        neitseelliset jälkeläiset olisivat mitä ilmeisimmin tyttöjä.

        Antiikin aikana oli monia taruja neitseiden ja jumalien yhteisistä jälkeläisistä. Sikäli Jeesuksen syntytaru on ajalleen ominainen, samoin taivaaseennousu kuolleista heränneenä. Todistajalausuntoja on Rooman keisareistakin, jotka teki saman tempun. Sinne vaan taivaaseen jumalaksi jumalten asuinsijoille.


      • taru vastaan tiede
        vain_ihminen kirjoitti:

        Niin, siksi siis ainakin Suomessa iso osa uskovia kuitenkin luottaa tieteen kertomaan siinä, miten elämä on maapallolla syntynyt ja kehittynyt.

        Luomiskertomus taas kertoo jostain ihan muusta kuin tieteellisistä asioista, sen ydinidea (jos niin voi sanoa) on mielestäni vain siinä, että Jumalalla on ollut näppinsä pelissä jollain lailla ja että Jumala on luonut (nykytietämyksen mukaan evoluution kautta) ihmisen omaksi kuvakseen, joten kaikki ihmiset ovat tässä suhteessa yhtä arvokkaita ja tärkeitä.

        apinan ja ihmisen välimuodot, tai yhteiset kantavanhemmat, joita evoluution mukaan on ollut olemassa? Olivatko nekin jumalankuvia, ja vielä nykyistä tarkempia, koska ne eivät olleet langenneet syntiin?


      • fffvb
        vain_ihminen kirjoitti:

        Biologiako todistaa ettei ihmisen sielu voi "vilahtaa" taivaaseen viimeisenä päivänä?

        Ja mitenkä tuota Pyhän Hengen siittämisominaisuutta on sitten tutkittu tieteellisesti?
        (Uutisissa oli taannoin hauska otsikko, Kuningasboa oli synnyttänyt neitseellisesti kaksikymmentä poikasta) :)

        ilman "rakastelua" itsestään lisääntyviä eläimiä (ja kasveja) muitakin kuin yksi boa.


      • cei5
        vain_ihminen kirjoitti:

        Biologiako todistaa ettei ihmisen sielu voi "vilahtaa" taivaaseen viimeisenä päivänä?

        Ja mitenkä tuota Pyhän Hengen siittämisominaisuutta on sitten tutkittu tieteellisesti?
        (Uutisissa oli taannoin hauska otsikko, Kuningasboa oli synnyttänyt neitseellisesti kaksikymmentä poikasta) :)

        Kun ihminen tai mikä tahansa eliö kuolee, niin se lakkaa olemasta. Sielua ei ole sen enempää sinulla kuin nasu-possullakaan.

        Pyhän hengen siittämistarinaa ei ole tutkittu tieteellisesti, koska se on vain tarina, taru, uskomus.


      • vain_ihminen
        cei5 kirjoitti:

        Kun ihminen tai mikä tahansa eliö kuolee, niin se lakkaa olemasta. Sielua ei ole sen enempää sinulla kuin nasu-possullakaan.

        Pyhän hengen siittämistarinaa ei ole tutkittu tieteellisesti, koska se on vain tarina, taru, uskomus.

        Siksipä en koe uskoni olevan ristiriidassa tieteen kanssa, voin uskoa sieluun ja Pyhään Henkeen kuin myös tieteen saavutuksiin ja teorioihin. Ne eivät sulje pois toisiaan.


      • Uskonnollinen kirjoitti:

        "Miten selitätte ihmisten viisaudenhampaat? Evoluutio?"

        En ole hammaslääkäri enkä tiedä viisaudenhampaista mitään muuta kuin että joillekin ne kasvavat vinoon ja pitää leikkauksella poistaa (kuten minä) ja joillakin ne mahtuvat kasvamaan ihan hyvin (kuten äitini) Ei tällä pitäisi olla evoluution kanssa mitään tekemistä...

        "Miten selitätte sen että on olemassa monia kissaeläimiä, kuten esim. muinainen sapellihammastiikeri ja nykyinen tiikeri. Kaikki nykyiset kissaeläimet muistuttavat toisiaan, kuten ihminenkin apinoita."

        Jumala loi kaikki eläimet, miten Hän sen teki, ei ole tiedossa. Itse ajattelisin että Hän on käyttänyt evoluutiota hyväkseen luomistyössään.

        "Miten voi olla mahdollista, että TUTKIMUSTEN mukaan simpanssilla ja ihmisellä on yli 95% samankaltainen geeniperimä, mikäli evoluutio ei olisi mahdollista. 4-5% todellakin on se erotus, johon liittyy muuttujia ihmisestä simpanssista."

        Uskon vahvasti että Jumala on luonut ihmisen, miten Hän sen teki, en tiedä. En sulje kokonaan pois mahdollisuutta että ihminen on kehitetty simpanssista ja kun kehitys oli valmis, oli ihminen luotu, Adam, joka oli ensimmäinen jota nimitettiin ihmiseksi ja jonka henki tuli taivaasta.

        "Oliko se joku nuorisotyöntekijä, joka tahtoi todistaa että mitä jos verrataan ihmistä ja kastematoa, niin "kuinka paljon siinä on eroavaisuuksia. Ei voi olla totta. :) Miten kastemato liittyy apinan ja ihmisen yhteiseen kantaisään? Ollaanko me sukuja lieroille!? :D""

        Mielestäni ei millään tavalla, tästä en tiedä mitään.

        "Miten selitätte dinosaurusten olemassa olon, joiden "jälkeläisinä" ollaan todettu linnut?
        http://fi.wikipedia.org/wiki/Dinosaurukset"

        Dinosaurukset ovat tietenkin olleet olemassa ja niistä on todennäköisesti kehittynyt muita eläimiä ennen kun ovat hävinneet kokonaan.


        "Teillä täytyy olla nuo asiat selvillä, jotta voisitte tuomita edes evoluution "heikoksi" ja luomiskertomuksen paikkansa pitävimmäksi!?"

        Jep, vaikka en kyllä tuomitse evoluutiota heikoksi, vaan Jumalan välineeksi luomistyössään.

        "HUOM! Vastaukset mielummin muualta kuin Raamatusta, kiitos! "

        Ei tainnut tulla yhtään lainausta Pyhästä Kirjasta :)

        "En ole hammaslääkäri enkä tiedä viisaudenhampaista mitään muuta kuin että joillekin ne kasvavat vinoon ja pitää leikkauksella poistaa (kuten minä) ja joillakin ne mahtuvat kasvamaan ihan hyvin (kuten äitini) Ei tällä pitäisi olla evoluution kanssa mitään tekemistä... "

        Et vastannut kysymykseen, minkä takia sellaiset tulee ihmisellä suuhun. Senkö takia että kun ne kasvaa ja alkaa särkemään niin ne voidaan repiä pois, eikä niillä tulisi olla mitään muuta merkitystä. Miksi Jumala olisi sellaiset ihmisten suuhun luonut, kun ne ovat turhaan ihmisten suussa, eikä niillä ole mielestäni mitään käyttöä!? Ilman "viisauden hampaitakin" ihminen pärjää.

        "Jumala loi kaikki eläimet, miten Hän sen teki, ei ole tiedossa. Itse ajattelisin että Hän on käyttänyt evoluutiota hyväkseen luomistyössään. "

        Jumala ei ole luonut mitään ja se miten Hän sen teki, tulisi olla tiedossanne ennen kuin evoluution torppaatte. Eikö järkevämpää olisi uskoa itse evoluutioon kuin uskotella että Jumala on käyttänyt luomistyössään evoluutiota. En ymmärrä koko sun kappalettasi!

        "Uskon vahvasti että Jumala on luonut ihmisen, miten Hän sen teki, en tiedä."

        Ongelma onkin siinä, että sinä uskot etkä liioin tiedä! Uskolla ei ole todistettu yhtään mitään!

        "Dinosaurukset ovat tietenkin olleet olemassa ja niistä on todennäköisesti kehittynyt muita eläimiä ennen kun ovat hävinneet kokonaan. "

        Mikseivät monet uskovaiset tällöin totea, että Dinosaurukset ovat olleet olemassa, kun ajattelevat että se on ns. "ateistien teoria". Kuvatkin on kuulemma väärennettyjä tms. Kreationistien puheet ja todistamiset ns. "nuoresta maailmasta" tulisi jo lopulta laittaa jo romukoppaan!

        "Jep, vaikka en kyllä tuomitse evoluutiota heikoksi, vaan Jumalan välineeksi luomistyössään. "

        Jumalan luomistyötä ei ole ollut! Millä sä sen todistat!?

        "Ei tainnut tulla yhtään lainausta Pyhästä Kirjasta :) "

        Otit edelleenkin esille Jumalan luomiskertomuksen, joka on peräisin Raamatusta!!


      • vain_ihminen
        ateisti1982 kirjoitti:

        "En ole hammaslääkäri enkä tiedä viisaudenhampaista mitään muuta kuin että joillekin ne kasvavat vinoon ja pitää leikkauksella poistaa (kuten minä) ja joillakin ne mahtuvat kasvamaan ihan hyvin (kuten äitini) Ei tällä pitäisi olla evoluution kanssa mitään tekemistä... "

        Et vastannut kysymykseen, minkä takia sellaiset tulee ihmisellä suuhun. Senkö takia että kun ne kasvaa ja alkaa särkemään niin ne voidaan repiä pois, eikä niillä tulisi olla mitään muuta merkitystä. Miksi Jumala olisi sellaiset ihmisten suuhun luonut, kun ne ovat turhaan ihmisten suussa, eikä niillä ole mielestäni mitään käyttöä!? Ilman "viisauden hampaitakin" ihminen pärjää.

        "Jumala loi kaikki eläimet, miten Hän sen teki, ei ole tiedossa. Itse ajattelisin että Hän on käyttänyt evoluutiota hyväkseen luomistyössään. "

        Jumala ei ole luonut mitään ja se miten Hän sen teki, tulisi olla tiedossanne ennen kuin evoluution torppaatte. Eikö järkevämpää olisi uskoa itse evoluutioon kuin uskotella että Jumala on käyttänyt luomistyössään evoluutiota. En ymmärrä koko sun kappalettasi!

        "Uskon vahvasti että Jumala on luonut ihmisen, miten Hän sen teki, en tiedä."

        Ongelma onkin siinä, että sinä uskot etkä liioin tiedä! Uskolla ei ole todistettu yhtään mitään!

        "Dinosaurukset ovat tietenkin olleet olemassa ja niistä on todennäköisesti kehittynyt muita eläimiä ennen kun ovat hävinneet kokonaan. "

        Mikseivät monet uskovaiset tällöin totea, että Dinosaurukset ovat olleet olemassa, kun ajattelevat että se on ns. "ateistien teoria". Kuvatkin on kuulemma väärennettyjä tms. Kreationistien puheet ja todistamiset ns. "nuoresta maailmasta" tulisi jo lopulta laittaa jo romukoppaan!

        "Jep, vaikka en kyllä tuomitse evoluutiota heikoksi, vaan Jumalan välineeksi luomistyössään. "

        Jumalan luomistyötä ei ole ollut! Millä sä sen todistat!?

        "Ei tainnut tulla yhtään lainausta Pyhästä Kirjasta :) "

        Otit edelleenkin esille Jumalan luomiskertomuksen, joka on peräisin Raamatusta!!

        "Jumala loi kaikki eläimet, miten Hän sen teki, ei ole tiedossa. Itse ajattelisin että Hän on käyttänyt evoluutiota hyväkseen luomistyössään. "

        Jumala ei ole luonut mitään ja se miten Hän sen teki, tulisi olla tiedossanne ennen kuin evoluution torppaatte. Eikö järkevämpää olisi uskoa itse evoluutioon kuin uskotella että Jumala on käyttänyt luomistyössään evoluutiota. En ymmärrä koko sun kappalettasi!

        .......

        Pisti oikein naurattamaan, kiitos tästä! :D
        Toinen sanoo että uskoo evoluutioon ja toinen sitten haukkuu siitä että tämä evoluutioon luottava torppaa evoluution.

        Tässä hyvä esimerkki siitä, miten ollaan niin varmoja toisen mielipiteistä ettei edes vaivauduta lukemaan niitä.


      • cei5
        vain_ihminen kirjoitti:

        Siksipä en koe uskoni olevan ristiriidassa tieteen kanssa, voin uskoa sieluun ja Pyhään Henkeen kuin myös tieteen saavutuksiin ja teorioihin. Ne eivät sulje pois toisiaan.

        "Sipille
        kuunteleva_kirkko Käyttäjän viestit 8.11.2010 13:54 Vastauksia: 32 Lukukertoja: 366 TEKSTIKOKO: Viestien esitystapa: Hei Sipi!

        Pyhä henki on nimensä mukainen, joten sitä ei tarvitse pelätä. Ei ole mahdollista tulla raskaaksi Pyhästä hengestä. Se ei voi siittää eikä muutenkaan toimia fyysisesti. Koska lapsi ei voi saada alkuaan Pyhästä hengestä, en usko, että muut sellaista myöskään uskoisivat. Haluatko kertoa lisää, miksi mietit tätä?

        Paula Ruotsalainen, perheneuvoja"

        http://keskustelu.suomi24.fi/node/9449214/thread

        Eikö teillä ole mitään yhtenevää kantaa mihinkään asiaan? Siis kantojahan ne ovat, koska uskonnot eivät perustu muuhun kuin vanhoihin taruihin. Tuntuu kuumalliselta, että Kirkko kuulolla palstalla muka annetaan vastauksia ja silti te löpisette niitä näitä.


      • vain_ihminen kirjoitti:

        "Jumala loi kaikki eläimet, miten Hän sen teki, ei ole tiedossa. Itse ajattelisin että Hän on käyttänyt evoluutiota hyväkseen luomistyössään. "

        Jumala ei ole luonut mitään ja se miten Hän sen teki, tulisi olla tiedossanne ennen kuin evoluution torppaatte. Eikö järkevämpää olisi uskoa itse evoluutioon kuin uskotella että Jumala on käyttänyt luomistyössään evoluutiota. En ymmärrä koko sun kappalettasi!

        .......

        Pisti oikein naurattamaan, kiitos tästä! :D
        Toinen sanoo että uskoo evoluutioon ja toinen sitten haukkuu siitä että tämä evoluutioon luottava torppaa evoluution.

        Tässä hyvä esimerkki siitä, miten ollaan niin varmoja toisen mielipiteistä ettei edes vaivauduta lukemaan niitä.

        Täytyy olla pikkasen samaa mieltä. En oikein käsitä miksi evoluutioon pitäisi kenenkään uskoa. Sehän on myöskin vain termi. Se tarkoittaa kehittymistä.


      • vain_ihminen kirjoitti:

        "Jumala loi kaikki eläimet, miten Hän sen teki, ei ole tiedossa. Itse ajattelisin että Hän on käyttänyt evoluutiota hyväkseen luomistyössään. "

        Jumala ei ole luonut mitään ja se miten Hän sen teki, tulisi olla tiedossanne ennen kuin evoluution torppaatte. Eikö järkevämpää olisi uskoa itse evoluutioon kuin uskotella että Jumala on käyttänyt luomistyössään evoluutiota. En ymmärrä koko sun kappalettasi!

        .......

        Pisti oikein naurattamaan, kiitos tästä! :D
        Toinen sanoo että uskoo evoluutioon ja toinen sitten haukkuu siitä että tämä evoluutioon luottava torppaa evoluution.

        Tässä hyvä esimerkki siitä, miten ollaan niin varmoja toisen mielipiteistä ettei edes vaivauduta lukemaan niitä.

        Itse nauroin taas sille ajatukselle että Jumala olisi käyttänyt evoluutiota luomistyössään. Miten helvetissä sen todistatte porukalle!? :D Joten kiitos siitä!!!


      • uskonnollinen
        ateisti1982 kirjoitti:

        Itse nauroin taas sille ajatukselle että Jumala olisi käyttänyt evoluutiota luomistyössään. Miten helvetissä sen todistatte porukalle!? :D Joten kiitos siitä!!!

        Miksikä ihmeessä pitää kaikki todistaa tieteen avulla? Jos minä uskon että huomenna sataa, niin se on uskomus, ei faktaa. Vaikka millä yrittäisit todistella tieteen avulla että huomenna ei varmasti sada, saan minä silti uskoa että huomenna sataa. Ja sataako huomenna, se nähdään varmuudella vasta huomenna! Ja mihin uskoni huomisesta säästä perustuu, säätiedotuksiin ja muutaman edeltäneen päivän säähän.

        Ei voi mitenkään varmuudella todistaa huomistä säätä, se ei ole mahdollista koska siitä ei ole vielä tietoa. Täsmälleen sama asia on uskonnon kanssa. Johan nimikin sanoo että se on uskontoa, ei tiedontoa! Et voi millään todistaa ettei Jumalaa olisi olemassa enkä ymmärrä miksi sinulla on siihen edes tarvetta?? Meille ei ole annettu tietoa kaikista luomistyön tapahtumista ja miten kaikki on tehty ja kuinka monen miljoonan vuoden aikana. Pientä osviittaa saamme argeologisista tutkimuksista, mutta nekin ovat pitkälti arvailuja, niin monta muuttujaa on pelissä.

        Tietenkään ei voi myöskään todistaa tieteenavulla että Jumala olisi olemassa. Siihenkään ei ole tarvetta. Katsoppa ympärillesi, kaikki siellä todistavat Jumalan olemassaolosta. Kaikki on järjestetty täydellisesti. Ainoa heikko lenkki on ihmisen tekemät valinnat!


      • outoa täydellisyyttä
        uskonnollinen kirjoitti:

        Miksikä ihmeessä pitää kaikki todistaa tieteen avulla? Jos minä uskon että huomenna sataa, niin se on uskomus, ei faktaa. Vaikka millä yrittäisit todistella tieteen avulla että huomenna ei varmasti sada, saan minä silti uskoa että huomenna sataa. Ja sataako huomenna, se nähdään varmuudella vasta huomenna! Ja mihin uskoni huomisesta säästä perustuu, säätiedotuksiin ja muutaman edeltäneen päivän säähän.

        Ei voi mitenkään varmuudella todistaa huomistä säätä, se ei ole mahdollista koska siitä ei ole vielä tietoa. Täsmälleen sama asia on uskonnon kanssa. Johan nimikin sanoo että se on uskontoa, ei tiedontoa! Et voi millään todistaa ettei Jumalaa olisi olemassa enkä ymmärrä miksi sinulla on siihen edes tarvetta?? Meille ei ole annettu tietoa kaikista luomistyön tapahtumista ja miten kaikki on tehty ja kuinka monen miljoonan vuoden aikana. Pientä osviittaa saamme argeologisista tutkimuksista, mutta nekin ovat pitkälti arvailuja, niin monta muuttujaa on pelissä.

        Tietenkään ei voi myöskään todistaa tieteenavulla että Jumala olisi olemassa. Siihenkään ei ole tarvetta. Katsoppa ympärillesi, kaikki siellä todistavat Jumalan olemassaolosta. Kaikki on järjestetty täydellisesti. Ainoa heikko lenkki on ihmisen tekemät valinnat!

        järjestetty täydellisesti, miksi suuri osa eläinlajeista on kuollut sukupuuttoon jo ennen ihmisen ilmaantumista?


      • mummomuori*
        ateisti1982 kirjoitti:

        Itse nauroin taas sille ajatukselle että Jumala olisi käyttänyt evoluutiota luomistyössään. Miten helvetissä sen todistatte porukalle!? :D Joten kiitos siitä!!!

        Näin entisenä ateistina osaan katsoa molemmat puolet. Kun termit ymmärretään eri leireissä vähän toisin, jää jäljelle tyhjän kinaus.

        Jos on tietoa jostakin, ei tarvita uskoa. Me siis joudumme uskomaan, koska tietoa ei vielä ole. Se on jokaisen vapaa valinta, haluaako uskoa ja mihin haluaa uskoa.

        Se mihin kukin uskoo, on synnyttänyt eri uskontoja. Tähän siis luetaan myös eri ”pakanauskonnot”. Uskonnot ovat sitten tulkinneet mm. Raamatussa olevia kirjoituksia kukin tavallaan. Ateistin tulisi olla siis tietoinen minkä uskonnon edustajan kanssa keskustelee. Kuten tämä kreationismi joka on Amerikan tuliainen ja liittyy sikäläiseen ”lahkolaisuuteen”.

        Tiede ei ole erehtymätön. Evoluutio on teoria, eli yhdessä sovittu käsitys siitä, kuinka elämä on kehittynyt. Siihenkin liittyy yhä paljon oletuksia – siis uskotaan asioiden olevan tietyllä tavalla, sekä tulkintoja. Tiede kehittyy koko ajan. Tieteen yksi perusongelma on kuinka ilmiöitä mitataan. Mikä mittari on luotettava ja se voidaanko ylipäätään tätä ilmiötä mitata millään mittarilla. Ehkä ei ole vielä keksitty sellaisia mittareita, joilla voitaisiin mitata ”ei aineellista”?

        Eri tieteiden kesken on paljon ristivetoa siitä, mikä on se ”totuus” ja mikä ei, samoin kuin uskontojen. Samoin joskus uskonnon ja tieteen ja on häilyvä. Joiden teoreetikkojen teoksiin uskotaan yhtä hartaasti kuin raamattuun.


      • uskonnollinen
        outoa täydellisyyttä kirjoitti:

        järjestetty täydellisesti, miksi suuri osa eläinlajeista on kuollut sukupuuttoon jo ennen ihmisen ilmaantumista?

        Katsos kun maailma rakennettiin ihmistä varten. Se mitä täällä oli ennen Adamia oli rakennustyömaata.

        Kaikkeen on hyvin järkevä selitys...


      • uskonnollinen kirjoitti:

        Miksikä ihmeessä pitää kaikki todistaa tieteen avulla? Jos minä uskon että huomenna sataa, niin se on uskomus, ei faktaa. Vaikka millä yrittäisit todistella tieteen avulla että huomenna ei varmasti sada, saan minä silti uskoa että huomenna sataa. Ja sataako huomenna, se nähdään varmuudella vasta huomenna! Ja mihin uskoni huomisesta säästä perustuu, säätiedotuksiin ja muutaman edeltäneen päivän säähän.

        Ei voi mitenkään varmuudella todistaa huomistä säätä, se ei ole mahdollista koska siitä ei ole vielä tietoa. Täsmälleen sama asia on uskonnon kanssa. Johan nimikin sanoo että se on uskontoa, ei tiedontoa! Et voi millään todistaa ettei Jumalaa olisi olemassa enkä ymmärrä miksi sinulla on siihen edes tarvetta?? Meille ei ole annettu tietoa kaikista luomistyön tapahtumista ja miten kaikki on tehty ja kuinka monen miljoonan vuoden aikana. Pientä osviittaa saamme argeologisista tutkimuksista, mutta nekin ovat pitkälti arvailuja, niin monta muuttujaa on pelissä.

        Tietenkään ei voi myöskään todistaa tieteenavulla että Jumala olisi olemassa. Siihenkään ei ole tarvetta. Katsoppa ympärillesi, kaikki siellä todistavat Jumalan olemassaolosta. Kaikki on järjestetty täydellisesti. Ainoa heikko lenkki on ihmisen tekemät valinnat!

        "Miksikä ihmeessä pitää kaikki todistaa tieteen avulla?"

        Uskominen voi olla harhakuvitteellinen ja virheellinen. Jos sä rakennat kaiken tietosi uskolle niin tällöinhän voit uskotella/arvata tuloksen esim. 1 1=3, mutta jos se perustuu todistettuun väitteeseen että 1 1=2, niin eikö sen avulla saa oikean tuloksen, kun muulla tavalla ei voi asiaa tulkita?!

        "Jos minä uskon että huomenna sataa, niin se on uskomus, ei faktaa. Vaikka millä yrittäisit todistella tieteen avulla että huomenna ei varmasti sada, saan minä silti uskoa että huomenna sataa. "

        Sääennustuksella ei ole mitään tekemistä uskon kanssa.
        http://ilmatieteenlaitos.fi

        "Et voi millään todistaa ettei Jumalaa olisi olemassa enkä ymmärrä miksi sinulla on siihen edes tarvetta??"

        Suurella todennäköisyydellä Jumalaa ei ole olemassa, koska ei ole ennenkään ollut. Miksi sinulla on tarvetta taas todistaa "Jumalan olemassa oloa"?

        Lisäksi luomisjorinanne ei yhdelläkään uskovaisella ole ollut samanlainen vaan siihen riittää kaikenlaisia tulkintoja.


      • uskonnollinen kirjoitti:

        Katsos kun maailma rakennettiin ihmistä varten. Se mitä täällä oli ennen Adamia oli rakennustyömaata.

        Kaikkeen on hyvin järkevä selitys...

        Dinosaurukset kuolivat asteroidin törmäyksestä!


      • mummomuori*
        ateisti1982 kirjoitti:

        "Miksikä ihmeessä pitää kaikki todistaa tieteen avulla?"

        Uskominen voi olla harhakuvitteellinen ja virheellinen. Jos sä rakennat kaiken tietosi uskolle niin tällöinhän voit uskotella/arvata tuloksen esim. 1 1=3, mutta jos se perustuu todistettuun väitteeseen että 1 1=2, niin eikö sen avulla saa oikean tuloksen, kun muulla tavalla ei voi asiaa tulkita?!

        "Jos minä uskon että huomenna sataa, niin se on uskomus, ei faktaa. Vaikka millä yrittäisit todistella tieteen avulla että huomenna ei varmasti sada, saan minä silti uskoa että huomenna sataa. "

        Sääennustuksella ei ole mitään tekemistä uskon kanssa.
        http://ilmatieteenlaitos.fi

        "Et voi millään todistaa ettei Jumalaa olisi olemassa enkä ymmärrä miksi sinulla on siihen edes tarvetta??"

        Suurella todennäköisyydellä Jumalaa ei ole olemassa, koska ei ole ennenkään ollut. Miksi sinulla on tarvetta taas todistaa "Jumalan olemassa oloa"?

        Lisäksi luomisjorinanne ei yhdelläkään uskovaisella ole ollut samanlainen vaan siihen riittää kaikenlaisia tulkintoja.

        lukenut tuota alla olevaa?


      • mummomuori* kirjoitti:

        Näin entisenä ateistina osaan katsoa molemmat puolet. Kun termit ymmärretään eri leireissä vähän toisin, jää jäljelle tyhjän kinaus.

        Jos on tietoa jostakin, ei tarvita uskoa. Me siis joudumme uskomaan, koska tietoa ei vielä ole. Se on jokaisen vapaa valinta, haluaako uskoa ja mihin haluaa uskoa.

        Se mihin kukin uskoo, on synnyttänyt eri uskontoja. Tähän siis luetaan myös eri ”pakanauskonnot”. Uskonnot ovat sitten tulkinneet mm. Raamatussa olevia kirjoituksia kukin tavallaan. Ateistin tulisi olla siis tietoinen minkä uskonnon edustajan kanssa keskustelee. Kuten tämä kreationismi joka on Amerikan tuliainen ja liittyy sikäläiseen ”lahkolaisuuteen”.

        Tiede ei ole erehtymätön. Evoluutio on teoria, eli yhdessä sovittu käsitys siitä, kuinka elämä on kehittynyt. Siihenkin liittyy yhä paljon oletuksia – siis uskotaan asioiden olevan tietyllä tavalla, sekä tulkintoja. Tiede kehittyy koko ajan. Tieteen yksi perusongelma on kuinka ilmiöitä mitataan. Mikä mittari on luotettava ja se voidaanko ylipäätään tätä ilmiötä mitata millään mittarilla. Ehkä ei ole vielä keksitty sellaisia mittareita, joilla voitaisiin mitata ”ei aineellista”?

        Eri tieteiden kesken on paljon ristivetoa siitä, mikä on se ”totuus” ja mikä ei, samoin kuin uskontojen. Samoin joskus uskonnon ja tieteen ja on häilyvä. Joiden teoreetikkojen teoksiin uskotaan yhtä hartaasti kuin raamattuun.

        "Näin entisenä ateistina osaan katsoa molemmat puolet."

        Juu, kaikkihan me ateisteina synnymme. :)

        "Jos on tietoa jostakin, ei tarvita uskoa. Me siis joudumme uskomaan, koska tietoa ei vielä ole."

        On kuitenkin perustavaa laatua oleva ero siinä, uskooko niihin vaihtoehtoihin jotka ovat todennäköisempiä, vai valitseeko uskonsa vastoin todennäköisyyksiä, jonkun muun kriteerin (esim. "mikä tuntuu itsestä mukavimmalta") perusteella.

        Harva asia on sellainen, josta meillä ei olisi yhtään mitään tietoa. Vähäisenkin tiedon perusteella voidaan jo päätellä jotain. Ja jos esimerkiksi jokin hypoteesi on ristiriidassa aiemmin selville saadun, hyvin luotettavaksi osoittautuneen tiedon kanssa, vähentää se kyseisen hypoteesin todennäköisyyttä. Näillä yksinkertaisilla arkijärjen menetelmillä voimme jo päätyä todennäköisesti oikeampiin oletuksiin, varsinkin jos niiden tueksi harjoitetaan systemaattista tietojen hankintaa.

        Ja jos taas on asioita joista ei kerta kaikkiaan ole mahdollista saada tietoa, jollainen vaikkapa alkuräjähdystä edeltänyt aika tällä hetkellä näyttäisi olevan, ei mikään pakota meitä uskomaan yhtään mitään. Voimme ihan suoraan tunnustaa itsellemme, ettei siitä tiedetä ainakaan vielä mitään. Se ei estä meitä spekuloimasta, keskustelemasta ja kehittelemästä hypoteeseja - mutta ei meidän myöskään pidä mennä selittämään omia hypoteesejamme todeksi, ja esim. tekemään yhteiskunnallisia päätöksiä niiden perusteella…

        "Tiede ei ole erehtymätön."

        Ja tämän tunnustaminen on tieteen yksi peruskriteereistä. Sen tietävät kaikki millään tavalla tieteeseen perehtyneet.

        "Evoluutio on teoria, eli yhdessä sovittu käsitys siitä, kuinka elämä on kehittynyt."

        Ei. Evoluutio on luonnossa havaittu fakta. Evoluutioteoria taas on teoria - mutta se on tieteellinen teoria, joka on taas täysin eri asia kuin puhekielen teoria. Evoluutioteoria selittää ja ennustaa evoluutiota erittäin tarkasti. Sen tukena on tuhansia tutkimuksia. Se siis ei ole mikään yhdessä sovittu käsitys. Tämä on merkittävä ero tieteen ja uskontojen välillä.

        "Ehkä ei ole vielä keksitty sellaisia mittareita, joilla voitaisiin mitata ”ei aineellista”?"

        Jos tämä "ei aineellinen" millään tavalla onnistuu vaikuttamaan fyysiseen maailmaamme, niin mikä voisi estää havaitsemasta sitä? Jos taas sillä ei ole mitään vaikutusta, niin silloin sillä ei myöskään ole mitään merkitystä, sen enempää kuin mitään mahdollisuutta sen havaitsemiseen.

        "Eri tieteiden kesken on paljon ristivetoa siitä, mikä on se ”totuus” ja mikä ei"

        Niinkuin jo totesin, "totuutta" ei kukaan väitäkään tieteessä tietävänsä. On paljon asioita joista meillä ei ole vielä tarpeeksi tietoa, ja niissä on tietenkin kilpailevia hypoteeseja ja teorioita. Miten muuten voisikaan olla? Tiede on kuitenkin toistaiseksi paras menetelmä luotettavan tiedon hankkimiseen. Jos tiedät paremman, kerro muillekin. Jos tiede ei ole jotakin saanut selville, ei sitä tiedä varmsti kukaan muukaan.

        "Joiden teoreetikkojen teoksiin uskotaan yhtä hartaasti kuin raamattuun."

        Tieteessä usko ei valitettavasti riitä. Vai onko sinulla tästä jotain esimerkkejä?


      • kansalainen..
        uskonnollinen kirjoitti:

        Katsos kun maailma rakennettiin ihmistä varten. Se mitä täällä oli ennen Adamia oli rakennustyömaata.

        Kaikkeen on hyvin järkevä selitys...

        neandertalinihmisetkin ennen Adamia? Kuuluivatko hekin rakennustyömaavaiheseen?
        Jos eivät kuuluneet, niin miksi heitä ei enää ole?


      • mummomuori* kirjoitti:

        Näin entisenä ateistina osaan katsoa molemmat puolet. Kun termit ymmärretään eri leireissä vähän toisin, jää jäljelle tyhjän kinaus.

        Jos on tietoa jostakin, ei tarvita uskoa. Me siis joudumme uskomaan, koska tietoa ei vielä ole. Se on jokaisen vapaa valinta, haluaako uskoa ja mihin haluaa uskoa.

        Se mihin kukin uskoo, on synnyttänyt eri uskontoja. Tähän siis luetaan myös eri ”pakanauskonnot”. Uskonnot ovat sitten tulkinneet mm. Raamatussa olevia kirjoituksia kukin tavallaan. Ateistin tulisi olla siis tietoinen minkä uskonnon edustajan kanssa keskustelee. Kuten tämä kreationismi joka on Amerikan tuliainen ja liittyy sikäläiseen ”lahkolaisuuteen”.

        Tiede ei ole erehtymätön. Evoluutio on teoria, eli yhdessä sovittu käsitys siitä, kuinka elämä on kehittynyt. Siihenkin liittyy yhä paljon oletuksia – siis uskotaan asioiden olevan tietyllä tavalla, sekä tulkintoja. Tiede kehittyy koko ajan. Tieteen yksi perusongelma on kuinka ilmiöitä mitataan. Mikä mittari on luotettava ja se voidaanko ylipäätään tätä ilmiötä mitata millään mittarilla. Ehkä ei ole vielä keksitty sellaisia mittareita, joilla voitaisiin mitata ”ei aineellista”?

        Eri tieteiden kesken on paljon ristivetoa siitä, mikä on se ”totuus” ja mikä ei, samoin kuin uskontojen. Samoin joskus uskonnon ja tieteen ja on häilyvä. Joiden teoreetikkojen teoksiin uskotaan yhtä hartaasti kuin raamattuun.

        "Evoluutio on teoria, eli yhdessä sovittu käsitys siitä, kuinka elämä on kehittynyt."

        Jos evoluutio olisi yhdessä sovittu käsitys, niin täällä ei olisi yhtäkään uskovaista tai kreationistia paukuttamassa omia tulkintojaan luomiskertomuksesta!


      • mummomuori*
        Schlechterwisser kirjoitti:

        "Näin entisenä ateistina osaan katsoa molemmat puolet."

        Juu, kaikkihan me ateisteina synnymme. :)

        "Jos on tietoa jostakin, ei tarvita uskoa. Me siis joudumme uskomaan, koska tietoa ei vielä ole."

        On kuitenkin perustavaa laatua oleva ero siinä, uskooko niihin vaihtoehtoihin jotka ovat todennäköisempiä, vai valitseeko uskonsa vastoin todennäköisyyksiä, jonkun muun kriteerin (esim. "mikä tuntuu itsestä mukavimmalta") perusteella.

        Harva asia on sellainen, josta meillä ei olisi yhtään mitään tietoa. Vähäisenkin tiedon perusteella voidaan jo päätellä jotain. Ja jos esimerkiksi jokin hypoteesi on ristiriidassa aiemmin selville saadun, hyvin luotettavaksi osoittautuneen tiedon kanssa, vähentää se kyseisen hypoteesin todennäköisyyttä. Näillä yksinkertaisilla arkijärjen menetelmillä voimme jo päätyä todennäköisesti oikeampiin oletuksiin, varsinkin jos niiden tueksi harjoitetaan systemaattista tietojen hankintaa.

        Ja jos taas on asioita joista ei kerta kaikkiaan ole mahdollista saada tietoa, jollainen vaikkapa alkuräjähdystä edeltänyt aika tällä hetkellä näyttäisi olevan, ei mikään pakota meitä uskomaan yhtään mitään. Voimme ihan suoraan tunnustaa itsellemme, ettei siitä tiedetä ainakaan vielä mitään. Se ei estä meitä spekuloimasta, keskustelemasta ja kehittelemästä hypoteeseja - mutta ei meidän myöskään pidä mennä selittämään omia hypoteesejamme todeksi, ja esim. tekemään yhteiskunnallisia päätöksiä niiden perusteella…

        "Tiede ei ole erehtymätön."

        Ja tämän tunnustaminen on tieteen yksi peruskriteereistä. Sen tietävät kaikki millään tavalla tieteeseen perehtyneet.

        "Evoluutio on teoria, eli yhdessä sovittu käsitys siitä, kuinka elämä on kehittynyt."

        Ei. Evoluutio on luonnossa havaittu fakta. Evoluutioteoria taas on teoria - mutta se on tieteellinen teoria, joka on taas täysin eri asia kuin puhekielen teoria. Evoluutioteoria selittää ja ennustaa evoluutiota erittäin tarkasti. Sen tukena on tuhansia tutkimuksia. Se siis ei ole mikään yhdessä sovittu käsitys. Tämä on merkittävä ero tieteen ja uskontojen välillä.

        "Ehkä ei ole vielä keksitty sellaisia mittareita, joilla voitaisiin mitata ”ei aineellista”?"

        Jos tämä "ei aineellinen" millään tavalla onnistuu vaikuttamaan fyysiseen maailmaamme, niin mikä voisi estää havaitsemasta sitä? Jos taas sillä ei ole mitään vaikutusta, niin silloin sillä ei myöskään ole mitään merkitystä, sen enempää kuin mitään mahdollisuutta sen havaitsemiseen.

        "Eri tieteiden kesken on paljon ristivetoa siitä, mikä on se ”totuus” ja mikä ei"

        Niinkuin jo totesin, "totuutta" ei kukaan väitäkään tieteessä tietävänsä. On paljon asioita joista meillä ei ole vielä tarpeeksi tietoa, ja niissä on tietenkin kilpailevia hypoteeseja ja teorioita. Miten muuten voisikaan olla? Tiede on kuitenkin toistaiseksi paras menetelmä luotettavan tiedon hankkimiseen. Jos tiedät paremman, kerro muillekin. Jos tiede ei ole jotakin saanut selville, ei sitä tiedä varmsti kukaan muukaan.

        "Joiden teoreetikkojen teoksiin uskotaan yhtä hartaasti kuin raamattuun."

        Tieteessä usko ei valitettavasti riitä. Vai onko sinulla tästä jotain esimerkkejä?

        voitaisiin suomentaan yleisnäkemys, yleiskäsitys.

        ”Vähäisenkin tiedon perusteella voidaan jo päätellä jotain…”

        Jos lähdetään tästä, niin juuri siksi olen vuosien saatossa päätynyt uskomaan Jumalaan. Mutta se taas ei tarkoita että allekirjoittaisin yhdenkään kirkon teesit. Ehkä tiede auttaa tässä ja lopulta saamme molemmat vastauksen, en koe niitä mitenkään niin ristiriitaisina kuin sinä. Toki sinä voit uskoa ”eläinten vallankumoukseen”, mutta vielä se kaukana. Minulle riittää se vähäinen tieto sekä tietoisuus, loppuihin on vain uskottava.

        ”…yksinkertaisilla arkijärjen menetelmillä voimme jo päätyä todennäköisesti oikeampiin oletuksiin, varsinkin jos niiden tueksi harjoitetaan systemaattista tietojen hankintaa…” ja ”Voimme ihan suoraan tunnustaa itsellemme, ettei siitä tiedetä ainakaan vielä mitään.”

        Juuri näin. Toisaalta koen että ihmisen tieto on kuitenkin aika vähäistä vielä. Meidän on vain uskottava noihin todennäköisimpiin oletuksiin sen aikaa, jotta voimme olla varmoja tiedoistamme ja tietämyksestämme.

        ” Se ei estä meitä spekuloimasta, keskustelemasta ja kehittelemästä hypoteeseja - mutta ei meidän myöskään pidä mennä selittämään omia hypoteesejamme todeksi, ja esim. tekemään yhteiskunnallisia päätöksiä niiden perusteella… ”

        No hupsista, ettäkö päätöksiä ei tehdä. Mutta juuri noinhan tehdään. Mietipä nyt vaikka eri talousteorioita, vaikkapa tämä jatkuvan kasvun ideologia? Tai usko siihen että biologia määrää meitä enemmän kuin tietoinen ajattelu, johon usein viitataan seksikeskusteluissa tai henkinen pahoinvointi joka on medikalisoitu vaikka tutkimustulokset ovat erittäin epävarmoja sekä sairauden syistä että lääkkeen vaikutuksista.

        ”Evoluutioteoria selittää ja ennustaa evoluutiota erittäin tarkasti.”
        Tämä oli uutta, sillä evoluution ennustaminen on jo henkimaailman hommaa. Kuinka voit ennustaa vaikka tulevat luonnonmullistukset tai maailmantalouden murskaantumisen ja yhteiskuntarakeneiden mullistukset, tai uusien tautimuotojen kehittymisen ja sen kuinka ne vaikuttavat etenkin ihmisen evoluution suuntaan? Evoluutio tiedekin on jakautunut useaan alalajiin, kiistellään kovasti siitä mikä vaikuttaa ja mihin.

        Tieteessä usko ei valitettavasti riitä. Vai onko sinulla tästä jotain esimerkkejä?

        No, perinteinen esimerkki on ihmisiin liittyvät tieteet, psykologia, psykiatria, sosiologia, filosofia, historia, taloustieteet jne. Juu, tiedän että sitten kiistellään siitä, ettei ne tiedettä ole nekään. Asioiden uskotaan olevan jotenkin, ja kun tietoa tulee, muutetaan käsityksiä ja luodaan uusia uskomuksia.


      • mummomuori* kirjoitti:

        voitaisiin suomentaan yleisnäkemys, yleiskäsitys.

        ”Vähäisenkin tiedon perusteella voidaan jo päätellä jotain…”

        Jos lähdetään tästä, niin juuri siksi olen vuosien saatossa päätynyt uskomaan Jumalaan. Mutta se taas ei tarkoita että allekirjoittaisin yhdenkään kirkon teesit. Ehkä tiede auttaa tässä ja lopulta saamme molemmat vastauksen, en koe niitä mitenkään niin ristiriitaisina kuin sinä. Toki sinä voit uskoa ”eläinten vallankumoukseen”, mutta vielä se kaukana. Minulle riittää se vähäinen tieto sekä tietoisuus, loppuihin on vain uskottava.

        ”…yksinkertaisilla arkijärjen menetelmillä voimme jo päätyä todennäköisesti oikeampiin oletuksiin, varsinkin jos niiden tueksi harjoitetaan systemaattista tietojen hankintaa…” ja ”Voimme ihan suoraan tunnustaa itsellemme, ettei siitä tiedetä ainakaan vielä mitään.”

        Juuri näin. Toisaalta koen että ihmisen tieto on kuitenkin aika vähäistä vielä. Meidän on vain uskottava noihin todennäköisimpiin oletuksiin sen aikaa, jotta voimme olla varmoja tiedoistamme ja tietämyksestämme.

        ” Se ei estä meitä spekuloimasta, keskustelemasta ja kehittelemästä hypoteeseja - mutta ei meidän myöskään pidä mennä selittämään omia hypoteesejamme todeksi, ja esim. tekemään yhteiskunnallisia päätöksiä niiden perusteella… ”

        No hupsista, ettäkö päätöksiä ei tehdä. Mutta juuri noinhan tehdään. Mietipä nyt vaikka eri talousteorioita, vaikkapa tämä jatkuvan kasvun ideologia? Tai usko siihen että biologia määrää meitä enemmän kuin tietoinen ajattelu, johon usein viitataan seksikeskusteluissa tai henkinen pahoinvointi joka on medikalisoitu vaikka tutkimustulokset ovat erittäin epävarmoja sekä sairauden syistä että lääkkeen vaikutuksista.

        ”Evoluutioteoria selittää ja ennustaa evoluutiota erittäin tarkasti.”
        Tämä oli uutta, sillä evoluution ennustaminen on jo henkimaailman hommaa. Kuinka voit ennustaa vaikka tulevat luonnonmullistukset tai maailmantalouden murskaantumisen ja yhteiskuntarakeneiden mullistukset, tai uusien tautimuotojen kehittymisen ja sen kuinka ne vaikuttavat etenkin ihmisen evoluution suuntaan? Evoluutio tiedekin on jakautunut useaan alalajiin, kiistellään kovasti siitä mikä vaikuttaa ja mihin.

        Tieteessä usko ei valitettavasti riitä. Vai onko sinulla tästä jotain esimerkkejä?

        No, perinteinen esimerkki on ihmisiin liittyvät tieteet, psykologia, psykiatria, sosiologia, filosofia, historia, taloustieteet jne. Juu, tiedän että sitten kiistellään siitä, ettei ne tiedettä ole nekään. Asioiden uskotaan olevan jotenkin, ja kun tietoa tulee, muutetaan käsityksiä ja luodaan uusia uskomuksia.

        "teoria voitaisiin suomentaan yleisnäkemys, yleiskäsitys."

        Tieteellinen teoria on ihan oma käsitteensä, jolla on oma määritelmänsä. Tieteellisen teorian rakennetaan havintojen pohjalta, ja sen avulla pitää pystyä selittämään ja ennustamaan ilmiötä.

        "Toki sinä voit uskoa ”eläinten vallankumoukseen”, mutta vielä se kaukana."

        Hmm, mitähän tarkoittanet tällä eläinten vallankumouksella johon "uskon"? :)

        "Minulle riittää se vähäinen tieto sekä tietoisuus, loppuihin on vain uskottava."

        Kyse onkin mielestäni siitä, että *millä perusteella* asioihin uskotaan.

        ”Toisaalta koen että ihmisen tieto on kuitenkin aika vähäistä vielä."

        Suhteessa kyllä, ihan totta. Ja mitä enemmän opimme, sitä enemmän löydämme taas uutta tuntematonta ja selitettävää. Mutta toisaalta täytyy tunnustaa, että tiedämme kuitenkin aika hämmästyttävän paljon asioista, kun ottaa huomioon kuinka kauan tiedettä on vasta tehty. Ja etenemisvauhti on joillain alueilla aika huima. Mielestäni ei ole syytä myöskään vähätellä sitä työtä, jonka ihmiskunta on onnistunut tiedonhankinnassa tekemään.

        "Meidän on vain uskottava noihin todennäköisimpiin oletuksiin sen aikaa, jotta voimme olla varmoja tiedoistamme ja tietämyksestämme."

        Nimenomaan todennäköisimpiin. Ja muistaa, että ne ovat vain todennäköisimpiä, ei totuuksia. Eli mielestäni "uskoa" on vähän huono sana. Se kuulostaa jotenkin ehdottomalta suhtautumiselta. Mieluummin käyttäisin ilmaisua "pitää todennäköisenä" tms.

        "No hupsista, ettäkö päätöksiä ei tehdä..."

        Sanoin siinä, että päätöksiä ei pidä tehdä pelkkien todistamattomien hypoteesien pohjalta, ilman että meillä on osoittaa niistä jotain todisteita tai tutkimustuloksia. Emme voi kieltää homojen adoptio-oikeutta esimerkiksi sillä perusteella, että homojen lapsista tulee epätasapainoisia jos heiltä puuttuu äidin tai isän malli tms. mikäli asiasta ei ole tutkimustuloksia. Asiat pitäisi päättää parhaan olemassaolevan tiedon perusteella, ja jos tietoa ei ole, sitä pitää hankkia.

        "Tämä oli uutta, sillä evoluution ennustaminen on jo henkimaailman hommaa. Kuinka voit ennustaa vaikka tulevat luonnonmullistukset tai maailmantalouden murskaantumisen…"

        Teorian ennustavuudella tarkoitetaan sitä, miten hyvin teoria testattaessa toimii. Eli yksinkertaistettuna: kokeillaan ilmeneekö ilmiö tietyissä olosuhteissa sen mukaan, miten kehitetty teoria ennustaa sen tekevän. Jos ei, teoria ei vielä ole valmis, ja sitä parannellaan kunnes voidaan varmuudella todeta että tässä tilanteessa tietty teorian osa pitää paikkansa tietyllä varmuudella. Eli se ei tarkoita sinällään tulevaisuuden ennustamista - muuten kuin pienen tarkasti rajatun ilmiön osalta.

        "Evoluutio tiedekin on jakautunut useaan alalajiin, kiistellään kovasti siitä mikä vaikuttaa ja mihin."

        En nyt tiedä mitä alalajeja tässä tarkoitat. Voi olla että puhun eri asiasta, mutta viittaan edelliseen viestiini: niin kauan kuin joitakin teorian osa-alueita ei ole riittävällä varmuudella verifioitu, on selvä että asioista on erilaisia hypoteeseja. Miten muuten voisi ollakaan?

        "No, perinteinen esimerkki on ihmisiin liittyvät tieteet, psykologia, psykiatria, sosiologia, filosofia, historia, taloustieteet jne. ... Asioiden uskotaan olevan jotenkin, ja kun tietoa tulee, muutetaan käsityksiä ja luodaan uusia uskomuksia."

        Edelleenkään en puhuisi uskosta ja uskomuksista. Ne ovat kuitenkin määritelmällisesti vähän eri asia - vaikka mieli tekisi itsekin niitä joskus käyttää joidenkin alojen yhteydessä…. Humanistisissa tieteissä on aloja, joissa on mahdotonta saada eksakteja tuloksia. Esimerkiksi monilla filosofian osa-alueilla siihen tuskin edes pyritään… Ja silti se voi olla merkityksellistä. Toisaalta vaikkapa sosiologinen tutkimus tuo erittäinkin relevanttia tietoa yhteiskunnasta, ja nimenomaan sellaista, jolla on välitöntä käytännön merkitystä.

        Se, että väärä tieto kumoutuu tai vanha täydentyy, on nimenomaan yksi tieteellisen metodin ansioista. Niinhän pitää tapahtuakin. Käytännössä yleisesti hyväksytty tieteellinen tieto menee hyvin harvoin uusiksi, vaikka sitä varsinkin uskovaispiireissä jaksetaan kovasti mainostaa. Kyse on tiedon lisääntymisestä ja tarkentumisesta, tai sitten hypoteesien tai vakiintumattomien teorioiden karsiutumisesta. Tieteentekijät nimittäin osaavat kyllä aika hyvin arvioida miten varmoja jotkin tiedot tai teoriat ovat, vaikka ne mediassa tai tavallisten ihmisten mielikuvissa kulkisivat jotenkin "varmoina".

        Ja tähänkin liittyen kysyisin taas: mikä on vaihtoehto? Jos olet keksinyt paremman keinon hankkia mahdollisimman luotettavaa tietoa kuin tieteellinen metodi, kerro se muillekin.


      • mummomuori* kirjoitti:

        Näin entisenä ateistina osaan katsoa molemmat puolet. Kun termit ymmärretään eri leireissä vähän toisin, jää jäljelle tyhjän kinaus.

        Jos on tietoa jostakin, ei tarvita uskoa. Me siis joudumme uskomaan, koska tietoa ei vielä ole. Se on jokaisen vapaa valinta, haluaako uskoa ja mihin haluaa uskoa.

        Se mihin kukin uskoo, on synnyttänyt eri uskontoja. Tähän siis luetaan myös eri ”pakanauskonnot”. Uskonnot ovat sitten tulkinneet mm. Raamatussa olevia kirjoituksia kukin tavallaan. Ateistin tulisi olla siis tietoinen minkä uskonnon edustajan kanssa keskustelee. Kuten tämä kreationismi joka on Amerikan tuliainen ja liittyy sikäläiseen ”lahkolaisuuteen”.

        Tiede ei ole erehtymätön. Evoluutio on teoria, eli yhdessä sovittu käsitys siitä, kuinka elämä on kehittynyt. Siihenkin liittyy yhä paljon oletuksia – siis uskotaan asioiden olevan tietyllä tavalla, sekä tulkintoja. Tiede kehittyy koko ajan. Tieteen yksi perusongelma on kuinka ilmiöitä mitataan. Mikä mittari on luotettava ja se voidaanko ylipäätään tätä ilmiötä mitata millään mittarilla. Ehkä ei ole vielä keksitty sellaisia mittareita, joilla voitaisiin mitata ”ei aineellista”?

        Eri tieteiden kesken on paljon ristivetoa siitä, mikä on se ”totuus” ja mikä ei, samoin kuin uskontojen. Samoin joskus uskonnon ja tieteen ja on häilyvä. Joiden teoreetikkojen teoksiin uskotaan yhtä hartaasti kuin raamattuun.

        vaikka sinäkin yrität sitä muuksi naamioida.

        On ilmeisesti totta että täysipäinen ei hurahda kamelikuskiuskontoihin.


      • lusikkasoppaan
        ateisti1982 kirjoitti:

        "Evoluutio on teoria, eli yhdessä sovittu käsitys siitä, kuinka elämä on kehittynyt."

        Jos evoluutio olisi yhdessä sovittu käsitys, niin täällä ei olisi yhtäkään uskovaista tai kreationistia paukuttamassa omia tulkintojaan luomiskertomuksesta!

        No eihän täällä olekaan, sinä vain tulkitset jokaisen kommentin, jossa kirjoittaja kertoo olevansa uskova, evoluution kieltäväksi.


      • mummomuori*
        Schlechterwisser kirjoitti:

        "teoria voitaisiin suomentaan yleisnäkemys, yleiskäsitys."

        Tieteellinen teoria on ihan oma käsitteensä, jolla on oma määritelmänsä. Tieteellisen teorian rakennetaan havintojen pohjalta, ja sen avulla pitää pystyä selittämään ja ennustamaan ilmiötä.

        "Toki sinä voit uskoa ”eläinten vallankumoukseen”, mutta vielä se kaukana."

        Hmm, mitähän tarkoittanet tällä eläinten vallankumouksella johon "uskon"? :)

        "Minulle riittää se vähäinen tieto sekä tietoisuus, loppuihin on vain uskottava."

        Kyse onkin mielestäni siitä, että *millä perusteella* asioihin uskotaan.

        ”Toisaalta koen että ihmisen tieto on kuitenkin aika vähäistä vielä."

        Suhteessa kyllä, ihan totta. Ja mitä enemmän opimme, sitä enemmän löydämme taas uutta tuntematonta ja selitettävää. Mutta toisaalta täytyy tunnustaa, että tiedämme kuitenkin aika hämmästyttävän paljon asioista, kun ottaa huomioon kuinka kauan tiedettä on vasta tehty. Ja etenemisvauhti on joillain alueilla aika huima. Mielestäni ei ole syytä myöskään vähätellä sitä työtä, jonka ihmiskunta on onnistunut tiedonhankinnassa tekemään.

        "Meidän on vain uskottava noihin todennäköisimpiin oletuksiin sen aikaa, jotta voimme olla varmoja tiedoistamme ja tietämyksestämme."

        Nimenomaan todennäköisimpiin. Ja muistaa, että ne ovat vain todennäköisimpiä, ei totuuksia. Eli mielestäni "uskoa" on vähän huono sana. Se kuulostaa jotenkin ehdottomalta suhtautumiselta. Mieluummin käyttäisin ilmaisua "pitää todennäköisenä" tms.

        "No hupsista, ettäkö päätöksiä ei tehdä..."

        Sanoin siinä, että päätöksiä ei pidä tehdä pelkkien todistamattomien hypoteesien pohjalta, ilman että meillä on osoittaa niistä jotain todisteita tai tutkimustuloksia. Emme voi kieltää homojen adoptio-oikeutta esimerkiksi sillä perusteella, että homojen lapsista tulee epätasapainoisia jos heiltä puuttuu äidin tai isän malli tms. mikäli asiasta ei ole tutkimustuloksia. Asiat pitäisi päättää parhaan olemassaolevan tiedon perusteella, ja jos tietoa ei ole, sitä pitää hankkia.

        "Tämä oli uutta, sillä evoluution ennustaminen on jo henkimaailman hommaa. Kuinka voit ennustaa vaikka tulevat luonnonmullistukset tai maailmantalouden murskaantumisen…"

        Teorian ennustavuudella tarkoitetaan sitä, miten hyvin teoria testattaessa toimii. Eli yksinkertaistettuna: kokeillaan ilmeneekö ilmiö tietyissä olosuhteissa sen mukaan, miten kehitetty teoria ennustaa sen tekevän. Jos ei, teoria ei vielä ole valmis, ja sitä parannellaan kunnes voidaan varmuudella todeta että tässä tilanteessa tietty teorian osa pitää paikkansa tietyllä varmuudella. Eli se ei tarkoita sinällään tulevaisuuden ennustamista - muuten kuin pienen tarkasti rajatun ilmiön osalta.

        "Evoluutio tiedekin on jakautunut useaan alalajiin, kiistellään kovasti siitä mikä vaikuttaa ja mihin."

        En nyt tiedä mitä alalajeja tässä tarkoitat. Voi olla että puhun eri asiasta, mutta viittaan edelliseen viestiini: niin kauan kuin joitakin teorian osa-alueita ei ole riittävällä varmuudella verifioitu, on selvä että asioista on erilaisia hypoteeseja. Miten muuten voisi ollakaan?

        "No, perinteinen esimerkki on ihmisiin liittyvät tieteet, psykologia, psykiatria, sosiologia, filosofia, historia, taloustieteet jne. ... Asioiden uskotaan olevan jotenkin, ja kun tietoa tulee, muutetaan käsityksiä ja luodaan uusia uskomuksia."

        Edelleenkään en puhuisi uskosta ja uskomuksista. Ne ovat kuitenkin määritelmällisesti vähän eri asia - vaikka mieli tekisi itsekin niitä joskus käyttää joidenkin alojen yhteydessä…. Humanistisissa tieteissä on aloja, joissa on mahdotonta saada eksakteja tuloksia. Esimerkiksi monilla filosofian osa-alueilla siihen tuskin edes pyritään… Ja silti se voi olla merkityksellistä. Toisaalta vaikkapa sosiologinen tutkimus tuo erittäinkin relevanttia tietoa yhteiskunnasta, ja nimenomaan sellaista, jolla on välitöntä käytännön merkitystä.

        Se, että väärä tieto kumoutuu tai vanha täydentyy, on nimenomaan yksi tieteellisen metodin ansioista. Niinhän pitää tapahtuakin. Käytännössä yleisesti hyväksytty tieteellinen tieto menee hyvin harvoin uusiksi, vaikka sitä varsinkin uskovaispiireissä jaksetaan kovasti mainostaa. Kyse on tiedon lisääntymisestä ja tarkentumisesta, tai sitten hypoteesien tai vakiintumattomien teorioiden karsiutumisesta. Tieteentekijät nimittäin osaavat kyllä aika hyvin arvioida miten varmoja jotkin tiedot tai teoriat ovat, vaikka ne mediassa tai tavallisten ihmisten mielikuvissa kulkisivat jotenkin "varmoina".

        Ja tähänkin liittyen kysyisin taas: mikä on vaihtoehto? Jos olet keksinyt paremman keinon hankkia mahdollisimman luotettavaa tietoa kuin tieteellinen metodi, kerro se muillekin.

        mielestäni siitä, että *millä perusteella* asioihin uskotaan."

        Aivan. Me ihmiset haemme vastauksia kysymykseen miksi, syitä ja seurauksia tai ennustettavuutta. Me teemme sen aina sen tiedon perusteella, mitä olemme havainneet (arkitieto) tai oppineet (tieteet, uskonnot jne.). Tieteet ovat aina vähän laahanneet tuon arkikokemuksen perässä. Me emme ole kyenneet selittämään jotain arkitiedon kautta niin olemme sitten joutuneet kehittämään erilaisia meidän mielestämme relevantteja mittareita (miten esim. tunteita mitataan eläimillä?). Niitä kehitetään yhä, jotta voitaisiin mm. ymmärtää ihmisen tai eläimen ajattelua, tunteita jne. Siinä olemme alkutaipaleella.

        Mikä saa ihmisen tekemään hyvää, rakastamaan tai elämään luonnon kanssa sovussa rinnakkain? Kuinka se onnistuu tai onnistui luonnonkansoilta, joilla kaikilla oli kova usko siihen, että jotain ihmistä suurempaa oli olemassa vaikka sitä ei ymmärretty? Mitä muuttui kun tiede otti uskonnon aseman?

        En voi puhua kaikkien uskovien suulla, sillä olen hylännyt sen uskon, mitä lapsena opetettiin. Kun asioita selitettiin konkretioina, joiden uskottiin auttavan ymmärtämään kristinuskoa. Materialistinen maailmankatsomus on siis tuttu, tarkoitan tällä Marxin näkemyksiä. Kun sitten opiskelin Raamattua ja Raamatun historiaa havahduin siihen että sosialismi ei ole mahdollista ilman Jeesuksen oppeja. Minusta tuli näin etsijä, ei kirjanoppinut.

        Huomasin että kyseenalaistin niitä uskovia, joita lapsena tai nuorena tunsin, enkä tuntenut oikeastaan yhtään uskontoa kunnolla. Nämä uskonjutut ovat niin vaikeita ymmärtää rajoittuneella järjellämme, että useimmat eivät kykene siihen – johtuen jo erilaisista taidoistamme ja kyvyistämme. On helpompaa nielaista auktoriteettien oppi, kuin purra se ja muodostaa oma käsitys. Tämä koskee myös ateisteja. Joku on jossain ollut älykkäämpi kuin minä, ja siksi uskon sen mitä hän kertoo – oli se sitten uskonto tai tiede.
        Sinusta on siis mukavampaa uskoa tieteisiin, koska se on helppoa perustella jonkun auktoriteetin suulla. Minusta on taas mukavampaa uskoa siihen, minkä olen oman käsitykseni mukaan hahmottanut.

        ”Eli mielestäni "uskoa" on vähän huono sana. Se kuulostaa jotenkin ehdottomalta suhtautumiselta. Mieluummin käyttäisin ilmaisua "pitää todennäköisenä" tms.”
        ”sosiologinen tutkimus tuo erittäinkin relevanttia tietoa yhteiskunnasta, ja nimenomaan sellaista, jolla on välitöntä käytännön merkitystä.”
        ”…teoria ei vielä ole valmis, ja sitä parannellaan kunnes voidaan varmuudella todeta että tässä tilanteessa tietty teorian osa pitää paikkansa tietyllä varmuudella. Eli se ei tarkoita sinällään tulevaisuuden ennustamista - muuten kuin pienen tarkasti rajatun ilmiön osalta.”
        ”mahdotonta saada eksakteja tuloksia.”

        Jos arkitiedon kautta olen huomannut että usko antaa paremman elämänlaadun, niin miksi en siis uskoisi? Sekö estää, etten tiedä varmasti tai se että joudun ehkä alati perustelemaan sitä?

        Aloitin siis siitä että jossain oli joku, joka oli tehnyt valtavan vaikutuksen ihmisiin. Niitä oli kirjattu ylös, vajavaisesti koska kyse oli ihmisistä. Nuo opit olivat vallankumouksellisia, suorastaan anarkistisia ja ovat sitä edelleen. Noissa opeissa on juuri sitä mitä me haemme, onnellista tasapanoista ja levollista elämää. Kuitenkin me kapinoimme niitä vastaan. Aivan kuten alkoholisti tai tupakoitsija kapinoi lopettamista vastaan. Tämänkään ”teoria” ei siis ole valmis vaan kehittyy edelleen.

        Pidän siis todennäköisempänä että noita oppeja pitää edes yrittää noudattaa, jotta minulla ja läheiselläni olisi hiukan parempi ja sisäisesti rauhallisempi elämä. Kokemukseni kautta näin on käynytkin. Tästä taas on näyttöä vaikkapa psykologisissa ja sosiologisissa tutkimuksissa.

        ”Jos olet keksinyt paremman keinon hankkia mahdollisimman luotettavaa tietoa kuin tieteellinen metodi, kerro se muillekin.”

        Tämä meni kohdaltani harakoille. Jos haluat väitellä kreationistien kanssa, niin esitä tuo väite heille. Muista että teistejä on monenlaisia, alkaen alkuperäiskansoista, buddhisteista aina näihin ufo uskovaisiin.


      • mummomuori*
        Nas.se kirjoitti:

        vaikka sinäkin yrität sitä muuksi naamioida.

        On ilmeisesti totta että täysipäinen ei hurahda kamelikuskiuskontoihin.

        Tottahan se on että elämä kehittyy edelleen, miksi se ei olisi? Kyse oli evoluutiosta tieteenalana, joka sekin kehittyy… - ei sekään ole valmis tai että jokaiseen teoreemaan tulisi uskoa kirjaimellisesti.


      • mummomuori* kirjoitti:

        Tottahan se on että elämä kehittyy edelleen, miksi se ei olisi? Kyse oli evoluutiosta tieteenalana, joka sekin kehittyy… - ei sekään ole valmis tai että jokaiseen teoreemaan tulisi uskoa kirjaimellisesti.

        että todellisuuden ja sadun raja on hämärä ja sitä pyritään kaikin keinon hämärtämään, jotta rationaalinen ajattelu - sikäli kuin uskovainen oman taikauskonsa kohdalta siihen kykenee - ei vahingossa tappaisi uskovaisen Jumalaa.

        Tuota pyrkimystäsi sekoittaa faktat ja epävarmat asiat tai näkemysten kehittäminen, kritisoin.


      • Kössönöm
        mummomuori* kirjoitti:

        Tottahan se on että elämä kehittyy edelleen, miksi se ei olisi? Kyse oli evoluutiosta tieteenalana, joka sekin kehittyy… - ei sekään ole valmis tai että jokaiseen teoreemaan tulisi uskoa kirjaimellisesti.

        "Tottahan se on että elämä kehittyy edelleen, miksi se ei olisi?"

        - Sotii kristinuskoa vastaan.

        "Kyse oli evoluutiosta tieteenalana, joka sekin kehittyy… - ei sekään ole valmis tai että jokaiseen teoreemaan tulisi uskoa kirjaimellisesti."

        - Evoluutio EI OLE tieteenala, vaan koko ajan todellisuudessa tapahtuvaa.


      • mummomuori*
        mummomuori* kirjoitti:

        Tottahan se on että elämä kehittyy edelleen, miksi se ei olisi? Kyse oli evoluutiosta tieteenalana, joka sekin kehittyy… - ei sekään ole valmis tai että jokaiseen teoreemaan tulisi uskoa kirjaimellisesti.

        Ellet kykenen tieteelliseen keskusteluun, en voi mitään. Jos suhtaudut kiihkomielisesti tieteeseen, ilman että et kykene kritisoimaan sitä, vastaat aika lailla fundamentalistia tai ortodoksisuutta. Tiede perustuu filosofian eri muotoihin, kuinka totuus on perusteltu. Samoin perustuu usko ja uskonnot.

        Sinä puhut todesta ja sadusta, en minä. Ellet kykene hyväksymän sitä että tieteeseenkin liittyy paljon epävarmuutta ja olettamuksia, et ole kyllin perehtynyt tieteen perusongelmiin.
        Minun tehtäväni ei ole soittaa, että usko Jumalaan on väärin, se on sinun ongelmasi! Et vain ole perehtynyt käsiteltävään aiheeseen kyllin hyvin.


      • mummomuori*
        Kössönöm kirjoitti:

        "Tottahan se on että elämä kehittyy edelleen, miksi se ei olisi?"

        - Sotii kristinuskoa vastaan.

        "Kyse oli evoluutiosta tieteenalana, joka sekin kehittyy… - ei sekään ole valmis tai että jokaiseen teoreemaan tulisi uskoa kirjaimellisesti."

        - Evoluutio EI OLE tieteenala, vaan koko ajan todellisuudessa tapahtuvaa.

        se sotii kristinuskoa vastaan? Ja etenkin, mitä uskontoa vastaan? Eivätkö esim. muhamettilaiset usko samoin?

        Tuota, jos evoluutio ei ole tieteen ala, se ei siis ole tieteellisesti todistettavissa?


      • mummomuori* kirjoitti:

        mielestäni siitä, että *millä perusteella* asioihin uskotaan."

        Aivan. Me ihmiset haemme vastauksia kysymykseen miksi, syitä ja seurauksia tai ennustettavuutta. Me teemme sen aina sen tiedon perusteella, mitä olemme havainneet (arkitieto) tai oppineet (tieteet, uskonnot jne.). Tieteet ovat aina vähän laahanneet tuon arkikokemuksen perässä. Me emme ole kyenneet selittämään jotain arkitiedon kautta niin olemme sitten joutuneet kehittämään erilaisia meidän mielestämme relevantteja mittareita (miten esim. tunteita mitataan eläimillä?). Niitä kehitetään yhä, jotta voitaisiin mm. ymmärtää ihmisen tai eläimen ajattelua, tunteita jne. Siinä olemme alkutaipaleella.

        Mikä saa ihmisen tekemään hyvää, rakastamaan tai elämään luonnon kanssa sovussa rinnakkain? Kuinka se onnistuu tai onnistui luonnonkansoilta, joilla kaikilla oli kova usko siihen, että jotain ihmistä suurempaa oli olemassa vaikka sitä ei ymmärretty? Mitä muuttui kun tiede otti uskonnon aseman?

        En voi puhua kaikkien uskovien suulla, sillä olen hylännyt sen uskon, mitä lapsena opetettiin. Kun asioita selitettiin konkretioina, joiden uskottiin auttavan ymmärtämään kristinuskoa. Materialistinen maailmankatsomus on siis tuttu, tarkoitan tällä Marxin näkemyksiä. Kun sitten opiskelin Raamattua ja Raamatun historiaa havahduin siihen että sosialismi ei ole mahdollista ilman Jeesuksen oppeja. Minusta tuli näin etsijä, ei kirjanoppinut.

        Huomasin että kyseenalaistin niitä uskovia, joita lapsena tai nuorena tunsin, enkä tuntenut oikeastaan yhtään uskontoa kunnolla. Nämä uskonjutut ovat niin vaikeita ymmärtää rajoittuneella järjellämme, että useimmat eivät kykene siihen – johtuen jo erilaisista taidoistamme ja kyvyistämme. On helpompaa nielaista auktoriteettien oppi, kuin purra se ja muodostaa oma käsitys. Tämä koskee myös ateisteja. Joku on jossain ollut älykkäämpi kuin minä, ja siksi uskon sen mitä hän kertoo – oli se sitten uskonto tai tiede.
        Sinusta on siis mukavampaa uskoa tieteisiin, koska se on helppoa perustella jonkun auktoriteetin suulla. Minusta on taas mukavampaa uskoa siihen, minkä olen oman käsitykseni mukaan hahmottanut.

        ”Eli mielestäni "uskoa" on vähän huono sana. Se kuulostaa jotenkin ehdottomalta suhtautumiselta. Mieluummin käyttäisin ilmaisua "pitää todennäköisenä" tms.”
        ”sosiologinen tutkimus tuo erittäinkin relevanttia tietoa yhteiskunnasta, ja nimenomaan sellaista, jolla on välitöntä käytännön merkitystä.”
        ”…teoria ei vielä ole valmis, ja sitä parannellaan kunnes voidaan varmuudella todeta että tässä tilanteessa tietty teorian osa pitää paikkansa tietyllä varmuudella. Eli se ei tarkoita sinällään tulevaisuuden ennustamista - muuten kuin pienen tarkasti rajatun ilmiön osalta.”
        ”mahdotonta saada eksakteja tuloksia.”

        Jos arkitiedon kautta olen huomannut että usko antaa paremman elämänlaadun, niin miksi en siis uskoisi? Sekö estää, etten tiedä varmasti tai se että joudun ehkä alati perustelemaan sitä?

        Aloitin siis siitä että jossain oli joku, joka oli tehnyt valtavan vaikutuksen ihmisiin. Niitä oli kirjattu ylös, vajavaisesti koska kyse oli ihmisistä. Nuo opit olivat vallankumouksellisia, suorastaan anarkistisia ja ovat sitä edelleen. Noissa opeissa on juuri sitä mitä me haemme, onnellista tasapanoista ja levollista elämää. Kuitenkin me kapinoimme niitä vastaan. Aivan kuten alkoholisti tai tupakoitsija kapinoi lopettamista vastaan. Tämänkään ”teoria” ei siis ole valmis vaan kehittyy edelleen.

        Pidän siis todennäköisempänä että noita oppeja pitää edes yrittää noudattaa, jotta minulla ja läheiselläni olisi hiukan parempi ja sisäisesti rauhallisempi elämä. Kokemukseni kautta näin on käynytkin. Tästä taas on näyttöä vaikkapa psykologisissa ja sosiologisissa tutkimuksissa.

        ”Jos olet keksinyt paremman keinon hankkia mahdollisimman luotettavaa tietoa kuin tieteellinen metodi, kerro se muillekin.”

        Tämä meni kohdaltani harakoille. Jos haluat väitellä kreationistien kanssa, niin esitä tuo väite heille. Muista että teistejä on monenlaisia, alkaen alkuperäiskansoista, buddhisteista aina näihin ufo uskovaisiin.

        Sinulla on hyvää pohdintaa näissä viesteissäsi! Kiitos kun jaksat vastailla.

        "Tieteet ovat aina vähän laahanneet tuon arkikokemuksen perässä. Me emme ole kyenneet selittämään jotain arkitiedon kautta niin olemme sitten joutuneet kehittämään erilaisia meidän mielestämme relevantteja mittareita…"

        Niin, eli kun tietoa ei saada arkijärjellä, niin otetaan tiede avuksi. Onko se "perässä laahaamista"? Niinkuin joku on joskus kuvannut, tiede on vain systemaattisesti harjoitettua arkijärkeä. Yksi ihminen pystyy ja ehtii arkijärjellään tekemään vain tietyn määrän luotettavia johtopäätöksiä lähiympäristöstään. Mutta kun hän ottaa avuksi aiemman systemaattisesti kerätyn ja testatun tiedon, ja tutkii samat asiat systemaattisesti ja kattavasti, on tulos paljon luotettavampi, ja tiedon määrä lisääntyy nopeammin. Ja tällä tavalla saatu uusi tieto saatetaan taas kaikkien muiden tietoon uuden tutkimuksen pohjaksi, mikäli se todetaan paikkansa pitäväksi. Eli paitsi systemaattista, tiede on yös yhteistä arkijärkeä.

        "Mikä saa ihmisen tekemään hyvää, rakastamaan tai elämään luonnon kanssa sovussa rinnakkain?… Mitä muuttui kun tiede otti uskonnon aseman?"

        Tässä sinulla näyttäisi olevan hyvin olennainen väärinymmärrys. Tiede ei ole uskonnon asemassa. Uskontoon liittyy paljon erilaisia dogmeja ja ulottuvuuksia, tiede ainoastaan kerää tietoa - jota uskonnot taas eivät tee. Eli molemmilla on eri tehtävät, eikä tiede siis voi edes teoriassa "ottaa uskonnon paikkaa". En ole kuullut kenenkään (muun kuin uskovaisen) koskaan edes ajattelevan mitään tuollaista. Tiede on vain tiedettä, työkalu siinä missä vasarakin.

        Se, että tieteen avulla on kerätty tietoa joka on ristiriidassa Raamatun kanssa (mikäli sitä tulkitaan kirjaimellisesti) on ainoa ulottuvuus jossa nämä kaksi kohtaavat. Ja se ristiriita on ihan todellinen. Mutta siitä on puhuttu muissa ketjuissa, joten ei ehkä tässä mennä sen pidemmälle siihen.

        "Sinusta on siis mukavampaa uskoa tieteisiin, koska se on helppoa perustella jonkun auktoriteetin suulla."

        Tässä on toinen virheellinen olettama. Tieteessä on käytännössä lähes mahdotonta yrittää käyttää auktoriteettia apuna varsinaisessa tieteellisessä työssä. Koska ainoastaan todisteet merkitsevät. Itse asiassa päinvastoin: kun tunnettu ja tunnustettu tiedemies saa vaikuttavia tuloksia, hyökkää lauma nuoria kunnianhimoisia kollegoja välittömästi tutkimuksen kimppuun. Sillä mikäpä olisi sen hienompaa oman uran kannalta, kuin löytää virhe ja pystyä kumoamaan tällaisen tutkijan työ!

        Otetaan esimerkiksi Darwin, jota jotkut uskovaiset näyttävät jostain syystä pitävän jonain "ateistien jumalana". Darwinin nimi on jäänyt historiaan ainoastaan sen takia, että hänen teoriansa on toistuvasti, 150 vuoden ajan ja tuhansissa tutkimuksissa osoittautunut paikkansapitäväksi. Yhtä Darwinia kohden on tuhansia tiedemiehiä joiden teoriat eivät ole tarkempaa tutkimusta kestäneet, ja ne on korvattu oikeammilla teorioilla. Darwin on maineensa ansainnut. Mutta ei siksi, että hänen teoriansa "sopii niin hyvin ateistien maailmankuvaan" tms., vaan yksinkertaisesti siksi että hän oivalsi jonkin asian oikein vähän ennen muita. Teoriaa kehittäessään Darwin ei vielä edes ollut auktoriteetti, vasta myöhempi tutkimus on tehnyt hänestä tunnetun.

        Ja puhun nimenomaan tieteellisestä työstä. Tunnetut tiedemiehet saavat tietysti mediahuomiota, ja pääsevät kertomaan omia mielipiteitään, kirjoittamaan kansantajuisia kirjoja ym. Siinä ei kuitenkaan ole kyse tieteestä. Kenenkään mielipidettä ei kirjata tiedoksi, oli sanoja sitten voittanut kuinka monta nobelia tahansa. Jos Hawking sanoo kirjassa, että Jumalaa ei tarvittu maailman luomisessa, niin se ei välttämättä ole todistettua tieteellistä tietoa, vaan ainoastaan Hawkingin oma mielipide. Näitä kahta asiaa ei maallikko aina pysty erottamaan.

        Ja edelleenkään, en näe millä perusteella väität tiedettä uskon asiaksi. Se ei ole sitä yhtään enempää kuin arkijärjen havainnot - itse asiassa vielä paljon vähemmän. Arkijärjellä voisimme helposti virheellisesti uskoa vaikkapa lapsia tarkkailtuamme että he tulevat herkkujen syömisestä sokerihumalaan ja ovat siksi vilkkaita. Tieteessä tämä hypoteesi pitää testata, ja kas, kaksoissokkokokeissa on huomattu että se ei pidä paikkaansa. Tai voimme lähes ilman epäilystä uskoa että vesi pyörii viemäriin valuessaan eri suuntaan pohjoisella ja eteläisellä pallonpuoliskolla - joku jopa osaa perustella että se johtuu coriolis-voimasta, ja se kuulostaa arkijärjellä uskottavalta. Tiede ei usko, se kokeilee. Ja toteaa että näin ei tapahdu, ja että coriolis-voimat vaikuttavat ainoastaan huomattavasti laajempiskaalaisiin ilmiöihin.

        ...jatkuu...


      • Schlechterwisser kirjoitti:

        Sinulla on hyvää pohdintaa näissä viesteissäsi! Kiitos kun jaksat vastailla.

        "Tieteet ovat aina vähän laahanneet tuon arkikokemuksen perässä. Me emme ole kyenneet selittämään jotain arkitiedon kautta niin olemme sitten joutuneet kehittämään erilaisia meidän mielestämme relevantteja mittareita…"

        Niin, eli kun tietoa ei saada arkijärjellä, niin otetaan tiede avuksi. Onko se "perässä laahaamista"? Niinkuin joku on joskus kuvannut, tiede on vain systemaattisesti harjoitettua arkijärkeä. Yksi ihminen pystyy ja ehtii arkijärjellään tekemään vain tietyn määrän luotettavia johtopäätöksiä lähiympäristöstään. Mutta kun hän ottaa avuksi aiemman systemaattisesti kerätyn ja testatun tiedon, ja tutkii samat asiat systemaattisesti ja kattavasti, on tulos paljon luotettavampi, ja tiedon määrä lisääntyy nopeammin. Ja tällä tavalla saatu uusi tieto saatetaan taas kaikkien muiden tietoon uuden tutkimuksen pohjaksi, mikäli se todetaan paikkansa pitäväksi. Eli paitsi systemaattista, tiede on yös yhteistä arkijärkeä.

        "Mikä saa ihmisen tekemään hyvää, rakastamaan tai elämään luonnon kanssa sovussa rinnakkain?… Mitä muuttui kun tiede otti uskonnon aseman?"

        Tässä sinulla näyttäisi olevan hyvin olennainen väärinymmärrys. Tiede ei ole uskonnon asemassa. Uskontoon liittyy paljon erilaisia dogmeja ja ulottuvuuksia, tiede ainoastaan kerää tietoa - jota uskonnot taas eivät tee. Eli molemmilla on eri tehtävät, eikä tiede siis voi edes teoriassa "ottaa uskonnon paikkaa". En ole kuullut kenenkään (muun kuin uskovaisen) koskaan edes ajattelevan mitään tuollaista. Tiede on vain tiedettä, työkalu siinä missä vasarakin.

        Se, että tieteen avulla on kerätty tietoa joka on ristiriidassa Raamatun kanssa (mikäli sitä tulkitaan kirjaimellisesti) on ainoa ulottuvuus jossa nämä kaksi kohtaavat. Ja se ristiriita on ihan todellinen. Mutta siitä on puhuttu muissa ketjuissa, joten ei ehkä tässä mennä sen pidemmälle siihen.

        "Sinusta on siis mukavampaa uskoa tieteisiin, koska se on helppoa perustella jonkun auktoriteetin suulla."

        Tässä on toinen virheellinen olettama. Tieteessä on käytännössä lähes mahdotonta yrittää käyttää auktoriteettia apuna varsinaisessa tieteellisessä työssä. Koska ainoastaan todisteet merkitsevät. Itse asiassa päinvastoin: kun tunnettu ja tunnustettu tiedemies saa vaikuttavia tuloksia, hyökkää lauma nuoria kunnianhimoisia kollegoja välittömästi tutkimuksen kimppuun. Sillä mikäpä olisi sen hienompaa oman uran kannalta, kuin löytää virhe ja pystyä kumoamaan tällaisen tutkijan työ!

        Otetaan esimerkiksi Darwin, jota jotkut uskovaiset näyttävät jostain syystä pitävän jonain "ateistien jumalana". Darwinin nimi on jäänyt historiaan ainoastaan sen takia, että hänen teoriansa on toistuvasti, 150 vuoden ajan ja tuhansissa tutkimuksissa osoittautunut paikkansapitäväksi. Yhtä Darwinia kohden on tuhansia tiedemiehiä joiden teoriat eivät ole tarkempaa tutkimusta kestäneet, ja ne on korvattu oikeammilla teorioilla. Darwin on maineensa ansainnut. Mutta ei siksi, että hänen teoriansa "sopii niin hyvin ateistien maailmankuvaan" tms., vaan yksinkertaisesti siksi että hän oivalsi jonkin asian oikein vähän ennen muita. Teoriaa kehittäessään Darwin ei vielä edes ollut auktoriteetti, vasta myöhempi tutkimus on tehnyt hänestä tunnetun.

        Ja puhun nimenomaan tieteellisestä työstä. Tunnetut tiedemiehet saavat tietysti mediahuomiota, ja pääsevät kertomaan omia mielipiteitään, kirjoittamaan kansantajuisia kirjoja ym. Siinä ei kuitenkaan ole kyse tieteestä. Kenenkään mielipidettä ei kirjata tiedoksi, oli sanoja sitten voittanut kuinka monta nobelia tahansa. Jos Hawking sanoo kirjassa, että Jumalaa ei tarvittu maailman luomisessa, niin se ei välttämättä ole todistettua tieteellistä tietoa, vaan ainoastaan Hawkingin oma mielipide. Näitä kahta asiaa ei maallikko aina pysty erottamaan.

        Ja edelleenkään, en näe millä perusteella väität tiedettä uskon asiaksi. Se ei ole sitä yhtään enempää kuin arkijärjen havainnot - itse asiassa vielä paljon vähemmän. Arkijärjellä voisimme helposti virheellisesti uskoa vaikkapa lapsia tarkkailtuamme että he tulevat herkkujen syömisestä sokerihumalaan ja ovat siksi vilkkaita. Tieteessä tämä hypoteesi pitää testata, ja kas, kaksoissokkokokeissa on huomattu että se ei pidä paikkaansa. Tai voimme lähes ilman epäilystä uskoa että vesi pyörii viemäriin valuessaan eri suuntaan pohjoisella ja eteläisellä pallonpuoliskolla - joku jopa osaa perustella että se johtuu coriolis-voimasta, ja se kuulostaa arkijärjellä uskottavalta. Tiede ei usko, se kokeilee. Ja toteaa että näin ei tapahdu, ja että coriolis-voimat vaikuttavat ainoastaan huomattavasti laajempiskaalaisiin ilmiöihin.

        ...jatkuu...

        "Minusta on taas mukavampaa uskoa siihen, minkä olen oman käsitykseni mukaan hahmottanut."

        Niin. Minäkin olen oman käsitykseni mukaan hahmottanut, että jos haluamme tietää miten jokin asia oikeasti on, se on syytä tutkia, eikä olettaa tai uskoa. Olen ihan itse lukenut tieteenfilosofiaa ja tieteen pelisäännöt, ja ihan itse hahmottanut, että en minä ainakaan keksi miten tiedonhankinnasta saisi luotettavampaa. Siksi uskon siihen, että se on tällä hetkellä paras keino saada tietää miten asiat todennäköisimmin ovat.

        "Jos arkitiedon kautta olen huomannut että usko antaa paremman elämänlaadun, niin miksi en siis uskoisi?"

        Tuo voi aivan hyvin olla totta, eikä kukaan järkevä ihminen haluakaan estää sinua uskomasta. (Voi tietysti olla, että elämänlaatusi olisi sittenkin parempi esimerkiksi jossain muussa uskonnossa tai uskonnottomana, mutta sitä on tietenkin vähän vaikea kovin kattavasti kokeilla ja testailla…) :D

        "Noissa opeissa on juuri sitä mitä me haemme, onnellista tasapanoista ja levollista elämää. Kuitenkin me kapinoimme niitä vastaan."

        Luulen (siis uskon, en tiedä), että tasapainoinen ja levollinen elämä syntyy eri ihmisille vähän eri tavoin. Minä olen esimerkiksi päässyt sitä lähemmäs ystävien, perheen, mukavien harrastusten, musiikin, kulttuurin ja mielenkiintoisen työn avulla. Uskontojen harrastamiseen tai jumaolentojen palvontaan en ole koskaan tuntenut minkäänlaista tarvetta, enkä ole kokenut että se voisi asiaa edistää, päinvastoin.

        "Muista että teistejä on monenlaisia, alkaen alkuperäiskansoista, buddhisteista aina näihin ufo uskovaisiin."

        Tiedän kyllä hyvin miten moniin eri jumaliin ihmiset uskovat, ja ovat historian kuluessa uskoneet. Aina yhtä suurella uskolla, aina kokien yhtä suurta yhteyttä, aina saaden omilta jumaliltaan täysin todellisiksi koettuja vastauksia, aina pitäen omaa uskoaan oikeana ja totena. :)

        Ja muista sinä myös, että ateisteja on monenlaisia. :)


      • mummomuori* kirjoitti:

        se sotii kristinuskoa vastaan? Ja etenkin, mitä uskontoa vastaan? Eivätkö esim. muhamettilaiset usko samoin?

        Tuota, jos evoluutio ei ole tieteen ala, se ei siis ole tieteellisesti todistettavissa?

        "Tuota, jos evoluutio ei ole tieteen ala, se ei siis ole tieteellisesti todistettavissa?"

        Ei, kun Kössönöm taitaa tuossa viitata käsitteiden määrittelyyn, eli että evoluutio ei ole tieteenala, vaan luonnossa havaittu ilmiö. Evoluutioteoria taas on tieteellinen teoria joka pyrkii selittämään evoluutiota, ja se puolestaan kuuluu biologian tieteenalaan.


      • Kössönöm
        mummomuori* kirjoitti:

        se sotii kristinuskoa vastaan? Ja etenkin, mitä uskontoa vastaan? Eivätkö esim. muhamettilaiset usko samoin?

        Tuota, jos evoluutio ei ole tieteen ala, se ei siis ole tieteellisesti todistettavissa?

        Aloitetaas nyt vaikka siitä, että tarinan mukaan jumala loi taivaan ja maan ja ihmisen (miehen) tomusta ja naisen hänen kylkiluustaan ja kaiken mitä ikinä nyt sitten raamatun ensilehdiltä voi lukea ja katso, näin on sanoi herra herra. No, jos tuo ei sodi evoluutiota vatsaan niin mikä sitten?

        Evoluutio on todistettavissa koska sitä todistettavasti koko ajan tapahtuu, mutta eihän tapahtuma ole tieteenala! Aaaaaaaaaaaaargh!


      • Kössönöm kirjoitti:

        Aloitetaas nyt vaikka siitä, että tarinan mukaan jumala loi taivaan ja maan ja ihmisen (miehen) tomusta ja naisen hänen kylkiluustaan ja kaiken mitä ikinä nyt sitten raamatun ensilehdiltä voi lukea ja katso, näin on sanoi herra herra. No, jos tuo ei sodi evoluutiota vatsaan niin mikä sitten?

        Evoluutio on todistettavissa koska sitä todistettavasti koko ajan tapahtuu, mutta eihän tapahtuma ole tieteenala! Aaaaaaaaaaaaargh!

        kyllä sen ymmärsin. Evoluutio ei kuitenkaan kerro, mistä tai miksi elämä on syntynyt. Eikö vain?

        Sinä siis haluat että Raamattua tulee tulkita kirjaimellisesti? Se on yksi filosofinen tapa, joka jostain kumman syystä on päässyt valloilleen. En ymmärrä miksi, mutta eri äärisuunnat tuntuvat viehättävän nykyajan ihmisiä. Näkemyksiä on kuitenkin monia ja ainakaan minulle ei koulussa opetettu mitään luomisoppia. Olen ymmärtänyt ettei luterilainen kirkkokaan nojaa siihen. http://fi.wikipedia.org/wiki/Kreationismi

        Kun asiaa lähestyy yleisen historian ja Raamatunhistorian kautta, asian voi tulkita toisin. Ja koska olen kristitty, niin jätän yleensä tuon Vanhan Testamentin tulkinnat kirjanoppineille ja keskityn Uuden Testamentin sanoman sisältöön.


      • Kössönöm
        mummomuori kirjoitti:

        kyllä sen ymmärsin. Evoluutio ei kuitenkaan kerro, mistä tai miksi elämä on syntynyt. Eikö vain?

        Sinä siis haluat että Raamattua tulee tulkita kirjaimellisesti? Se on yksi filosofinen tapa, joka jostain kumman syystä on päässyt valloilleen. En ymmärrä miksi, mutta eri äärisuunnat tuntuvat viehättävän nykyajan ihmisiä. Näkemyksiä on kuitenkin monia ja ainakaan minulle ei koulussa opetettu mitään luomisoppia. Olen ymmärtänyt ettei luterilainen kirkkokaan nojaa siihen. http://fi.wikipedia.org/wiki/Kreationismi

        Kun asiaa lähestyy yleisen historian ja Raamatunhistorian kautta, asian voi tulkita toisin. Ja koska olen kristitty, niin jätän yleensä tuon Vanhan Testamentin tulkinnat kirjanoppineille ja keskityn Uuden Testamentin sanoman sisältöön.

        mutta siitähän tässä ei ole kysymys. Evoluutio on tapahtumaketju, se ei "kerro" mitään, sillä ei ole päämäärää eikä tarkoitusta. Sitä vain tapahtuu koko ajan.

        Yhtä hyvin voisit kysyä miksi talvi ei kerro miksi se on.


      • Kössönöm kirjoitti:

        mutta siitähän tässä ei ole kysymys. Evoluutio on tapahtumaketju, se ei "kerro" mitään, sillä ei ole päämäärää eikä tarkoitusta. Sitä vain tapahtuu koko ajan.

        Yhtä hyvin voisit kysyä miksi talvi ei kerro miksi se on.

        sitten samaa mieltä.


      • lusikkasoppaan kirjoitti:

        No eihän täällä olekaan, sinä vain tulkitset jokaisen kommentin, jossa kirjoittaja kertoo olevansa uskova, evoluution kieltäväksi.

        Mielestäni kirkon tulisi perustaa kantansa jompaan kumpaan (luomiskertomukseen tai evoluutioon), koska täällä kirkon työntekijöiltä saa jokaiseen erilaisen vastauksen eli se tässä suuresti riitelee monien käsitysten kanssa.

        Esimerkiksi kirkolla ei ole yhtämielteistä kantaa edes homokysymykseen, johon kaikki samaistuisi! Yks pappi haluaa vihkiä homot, toinen pappi haluaa siunata homot, kolmas pappi tahtoo rukoilla homojen kanssa, neljäs pappi potkii naispapit kirkosta pois ja tahtoo kivittää homot.


      • Schlechterwisser kirjoitti:

        "Minusta on taas mukavampaa uskoa siihen, minkä olen oman käsitykseni mukaan hahmottanut."

        Niin. Minäkin olen oman käsitykseni mukaan hahmottanut, että jos haluamme tietää miten jokin asia oikeasti on, se on syytä tutkia, eikä olettaa tai uskoa. Olen ihan itse lukenut tieteenfilosofiaa ja tieteen pelisäännöt, ja ihan itse hahmottanut, että en minä ainakaan keksi miten tiedonhankinnasta saisi luotettavampaa. Siksi uskon siihen, että se on tällä hetkellä paras keino saada tietää miten asiat todennäköisimmin ovat.

        "Jos arkitiedon kautta olen huomannut että usko antaa paremman elämänlaadun, niin miksi en siis uskoisi?"

        Tuo voi aivan hyvin olla totta, eikä kukaan järkevä ihminen haluakaan estää sinua uskomasta. (Voi tietysti olla, että elämänlaatusi olisi sittenkin parempi esimerkiksi jossain muussa uskonnossa tai uskonnottomana, mutta sitä on tietenkin vähän vaikea kovin kattavasti kokeilla ja testailla…) :D

        "Noissa opeissa on juuri sitä mitä me haemme, onnellista tasapanoista ja levollista elämää. Kuitenkin me kapinoimme niitä vastaan."

        Luulen (siis uskon, en tiedä), että tasapainoinen ja levollinen elämä syntyy eri ihmisille vähän eri tavoin. Minä olen esimerkiksi päässyt sitä lähemmäs ystävien, perheen, mukavien harrastusten, musiikin, kulttuurin ja mielenkiintoisen työn avulla. Uskontojen harrastamiseen tai jumaolentojen palvontaan en ole koskaan tuntenut minkäänlaista tarvetta, enkä ole kokenut että se voisi asiaa edistää, päinvastoin.

        "Muista että teistejä on monenlaisia, alkaen alkuperäiskansoista, buddhisteista aina näihin ufo uskovaisiin."

        Tiedän kyllä hyvin miten moniin eri jumaliin ihmiset uskovat, ja ovat historian kuluessa uskoneet. Aina yhtä suurella uskolla, aina kokien yhtä suurta yhteyttä, aina saaden omilta jumaliltaan täysin todellisiksi koettuja vastauksia, aina pitäen omaa uskoaan oikeana ja totena. :)

        Ja muista sinä myös, että ateisteja on monenlaisia. :)

        Siitä olen kanssasi samaa mieltä että tiede todentaa ne tällä hetkellä havaittavat asiat. Kun mennään vaikka historiaan tai vieläkin kauemmas esihistoriaan jne. joudumme löydöstemme pohjalta olettamaan. Samoin jos kehitämme vaikka uuden lääkkeen, joudumme olettamaan mitkä ovatkaan vaikutukset vaikka 30 vuoden kuluttua. Syntyy teoria, jota sitten kokeillaan ja pyritään mittamaan sitä mukaa kun ensin saadaan luotettavat mittarit niille. Siinä auttaa tuo tieteellinen dialogi.

        ”tiede siis voi edes teoriassa "ottaa uskonnon paikkaa". ”… lähes mahdotonta yrittää käyttää auktoriteettia apuna...”

        Auktoriteetilla tarkoitin juuri tavalliselle ihmiselle merkityksellistä henkilöä. Ihmiset suorastaan janoavat näitä henkilöitä. Syystä tai toisesta joukot tarvitsevat näitä, joiden mukaan myös pyrkivät toimimaan. Huipulla voi olla yksinäistä, mutta vähän alemmalla tasolla eri tutkimukset ja niistä johdetetut päätelmät ovat kyllä suoraan lainattuja joltain mikä sillä hetkellä on auktoriteetti. Esimerkkinä vaikka psykologian ja psykiatrian tieteiden kehitys. Freud teorioineen on laulettu useaan kertaan suohon, mutta yhä on näitä jotka vannovat hänen nimeensä. Eikä Marx ole heistä se vähäisin.

        Aikoinaan ennustettiin että uskonnot tulevat häviämään, koska niitä ei tarvita. Tiede syrjäyttää tarpeen niihin, usko korvataan tiedolla. Näin ei vain olekaan käynyt. Itse asiassa tarve uskoa ihan vaikka mihin on lisääntynyt. Hämmästyttävintä minusta on että nyt kaivataan niitä vanhoja ahdasmielisiä uskontoja. Miksi se tieto ei riitä?

        ”…elämänlaatusi olisi sittenkin parempi esimerkiksi jossain muussa uskonnossa tai uskonnottomana, mutta sitä on tietenkin vähän vaikea kovin kattavasti kokeilla ja testailla…”

        Miksi tiede ei voi sitä testata? Omalta kohdalta voin tehdä vertailuja vain kokemukseni kautta. Jos olen ollut ateisti noin 20 vuotta ja sitten hiljalleen alkanutkin miettiä asioista toisin, niin voin kertoa että henkisesti laatu parani. Toki ja totta kai voi omaa asennettaan muuttaa muutoinkin, en vain keksinyt siihen toimivaa keinoa. Eikä kukaan osannut edes neuvoa muuta. Tai anteeksi, kyllä, minulle tarjottiin varsin kyynistä asennetta joka olisi vaatinut oman itseni turruttamista.

        Olen vuosien saatossa tutustunut moniin uskontoihin sekä tapoihin uskoa. Eniten teki vaikutuksen se henkinen voima joka useimmissa uskovissa on.
        Ihmisille on suuria asioita sellaiset kuin rakkaus, luottamus, yhteenkuuluvuus jne. Samoin elämää ylläpitäviä asioita on toivo ja usko siihen että jotain hyvää on olemassa. Kyllä näitäkin yritetään tutkitaan tieteellisesti. Tiede ei vain voi poistaa noita ilmiöitä ja kokemuksena ovat hyvin syvällisiä, johtuivat ne sitten mistä aivokemiasta tahansa. Ne myös määräävät sen, kuinka me elämämme koemme ja kuinka toimimme.

        Kun aikoinaan keskustelin ateistien kanssa mm. moraalista ja rakastamisesta, en oikein saanut kunnollisia vastauksia. Miksi meidän pitää yrittää tehdä hyvää, ellei meillä ole tiettyjä sääntöjä sekä uskoa siihen että noiden sääntöjen avulla voimme luoda parempi yhteiskunta. Meillä tulee olla myös uskoa siihen että muutkin ihmiset pyrkivät noudattamaan samaa moraalia. No, he tarjosivat tähän kyllin raskaita sanktioita jotta edes pelon avulla homma sujuu. Yllättävää kyllä anarkisteilla oli hyvin mielenkiintoinen selitys. He uskovat että kun kaikki saa tehdä mitä lystää, ja kun huomataan mitä siitä seuraa, löytyy automaattisesti nuo säännöt.

        Lopuksi, kuinka rahamarkkina toimivat. Kukaan ei osaa eikä pysty ennustamaan, mihin suuntaan ja milloin kurssit laskevat ja nousevat. Pienikin huhu vaikka Amerikasta siihen suuntaan, että nyt menee huonosti ja aalto alkaa. Se onko se todenperäinen vai ei, sillä ei ole merkitystä. ihmiset käyttäytyvät siis kaikesta huolimatta yhtä epärationaalisesti kuin ennen tiedettäkin.


      • ateisti1982 kirjoitti:

        Mielestäni kirkon tulisi perustaa kantansa jompaan kumpaan (luomiskertomukseen tai evoluutioon), koska täällä kirkon työntekijöiltä saa jokaiseen erilaisen vastauksen eli se tässä suuresti riitelee monien käsitysten kanssa.

        Esimerkiksi kirkolla ei ole yhtämielteistä kantaa edes homokysymykseen, johon kaikki samaistuisi! Yks pappi haluaa vihkiä homot, toinen pappi haluaa siunata homot, kolmas pappi tahtoo rukoilla homojen kanssa, neljäs pappi potkii naispapit kirkosta pois ja tahtoo kivittää homot.

        pitäisi niin tehdä? Johan mm. katollinen ja ortodoksinen kirkko toimii kuvaamallasi tavalla. Itse olen liittynyt kirkkoon juuri tuon moninaisuuden takia ja että kirkossa on tilaa erilaisuudelle. Onhan kirkon piirissä myös lestadiolaisia, körttejä jne. eli onko mielestäsi heidän vallattava koko kirkko?

        Sittenpä voi olla että moni lähtee etsimään itselleen sopivampaa paikkaa - ainakin minä teen niin.


      • mummomuori kirjoitti:

        kyllä sen ymmärsin. Evoluutio ei kuitenkaan kerro, mistä tai miksi elämä on syntynyt. Eikö vain?

        Sinä siis haluat että Raamattua tulee tulkita kirjaimellisesti? Se on yksi filosofinen tapa, joka jostain kumman syystä on päässyt valloilleen. En ymmärrä miksi, mutta eri äärisuunnat tuntuvat viehättävän nykyajan ihmisiä. Näkemyksiä on kuitenkin monia ja ainakaan minulle ei koulussa opetettu mitään luomisoppia. Olen ymmärtänyt ettei luterilainen kirkkokaan nojaa siihen. http://fi.wikipedia.org/wiki/Kreationismi

        Kun asiaa lähestyy yleisen historian ja Raamatunhistorian kautta, asian voi tulkita toisin. Ja koska olen kristitty, niin jätän yleensä tuon Vanhan Testamentin tulkinnat kirjanoppineille ja keskityn Uuden Testamentin sanoman sisältöön.

        rehellinen että ymmärätisi satukirjassaan olevan 4 kpl luomiskertomuksia. Kaikki erilaisia ja keskenään ristiriitaisia.

        Miten hihhuli reagoi? Luo ihan oman luomiskertomuksensa ja jauhaa mitä jauhaa, palstalla.


      • mummomuori* kirjoitti:

        Ellet kykenen tieteelliseen keskusteluun, en voi mitään. Jos suhtaudut kiihkomielisesti tieteeseen, ilman että et kykene kritisoimaan sitä, vastaat aika lailla fundamentalistia tai ortodoksisuutta. Tiede perustuu filosofian eri muotoihin, kuinka totuus on perusteltu. Samoin perustuu usko ja uskonnot.

        Sinä puhut todesta ja sadusta, en minä. Ellet kykene hyväksymän sitä että tieteeseenkin liittyy paljon epävarmuutta ja olettamuksia, et ole kyllin perehtynyt tieteen perusongelmiin.
        Minun tehtäväni ei ole soittaa, että usko Jumalaan on väärin, se on sinun ongelmasi! Et vain ole perehtynyt käsiteltävään aiheeseen kyllin hyvin.

        "Sinä puhut todesta ja sadusta, en minä. Ellet kykene hyväksymän sitä että tieteeseenkin liittyy paljon epävarmuutta ja olettamuksia, et ole kyllin perehtynyt tieteen perusongelmiin.
        Minun tehtäväni ei ole soittaa, että usko Jumalaan on väärin, se on sinun ongelmasi! Et vain ole perehtynyt käsiteltävään aiheeseen kyllin hyvin. "

        Eihän satuolentojen pitäminen totena ole mitenkään väärin tai ongelmallista.


      • zlkdbc
        Nas.se kirjoitti:

        "Sinä puhut todesta ja sadusta, en minä. Ellet kykene hyväksymän sitä että tieteeseenkin liittyy paljon epävarmuutta ja olettamuksia, et ole kyllin perehtynyt tieteen perusongelmiin.
        Minun tehtäväni ei ole soittaa, että usko Jumalaan on väärin, se on sinun ongelmasi! Et vain ole perehtynyt käsiteltävään aiheeseen kyllin hyvin. "

        Eihän satuolentojen pitäminen totena ole mitenkään väärin tai ongelmallista.

        Uusateistimaista tyypillistä jankkausta satuolennosta, joka keskustelu ei johda mihinkään. Puhut satuolennosta, kun et tiedä, että Jumalasta voi saada konkreettisen kokemuksen.


      • Joo, jos...
        zlkdbc kirjoitti:

        Uusateistimaista tyypillistä jankkausta satuolennosta, joka keskustelu ei johda mihinkään. Puhut satuolennosta, kun et tiedä, että Jumalasta voi saada konkreettisen kokemuksen.

        ...sattuu olemaan tyypillinen vanhoilliskristillinen ev.lutka!

        "...että Jumalasta voi saada konkreettisen kokemuksen."

        Voit myös saada ufoista todellisen tuntuisen kokemuksen, jos tiettyyn paikkaan aivoihisi johdetaan sähköä.


      • Nasse.
        zlkdbc kirjoitti:

        Uusateistimaista tyypillistä jankkausta satuolennosta, joka keskustelu ei johda mihinkään. Puhut satuolennosta, kun et tiedä, että Jumalasta voi saada konkreettisen kokemuksen.

        ja kuten tiedämme, sen jumalat, taikatemput ja loitsut ovat plagiointia aiemmista mytologoista.

        Jumalaharhasi ovat harhoja, aivojen virhetoimintoja. Sinun konkreettinen jumalasi kuolee mukanasi.

        Toisekseen, jos isnä olet sitä mieltä että kokemukset Jumalsta todistavat sen olevan olemassa, joudut myöntämään että kilpailevien väkivaltauskontojen ja muiden uskontojen Jumalat ovat olemassa.

        Mikäli väität että oma mielihyväjumalasi on ainoa todellinen, syylliistyt (siinäkin) älylliseen epärehellisyyteen.


      • zlkdbc kirjoitti:

        Uusateistimaista tyypillistä jankkausta satuolennosta, joka keskustelu ei johda mihinkään. Puhut satuolennosta, kun et tiedä, että Jumalasta voi saada konkreettisen kokemuksen.

        sanaa - uus voitaisiin käyttää monissa yhteyksissäilman että se tuo mitään lisäarvoa entiselle sanalle - ateismi. Uusoikeisto, uusherännäisyys, uuslastaadiolaisuus ym. ym.
        Se on vähän samaa kun jokin esim. poliittinen ryhämä nostaa päätään niin oikopäätä nostetaan kissa pöydälle ja huudetaan "se on sitä populismia"

        Märittele tarkemmin miten ns. ateismi ja uusateismi eroavat toisistaan.


      • mummomuori kirjoitti:

        Siitä olen kanssasi samaa mieltä että tiede todentaa ne tällä hetkellä havaittavat asiat. Kun mennään vaikka historiaan tai vieläkin kauemmas esihistoriaan jne. joudumme löydöstemme pohjalta olettamaan. Samoin jos kehitämme vaikka uuden lääkkeen, joudumme olettamaan mitkä ovatkaan vaikutukset vaikka 30 vuoden kuluttua. Syntyy teoria, jota sitten kokeillaan ja pyritään mittamaan sitä mukaa kun ensin saadaan luotettavat mittarit niille. Siinä auttaa tuo tieteellinen dialogi.

        ”tiede siis voi edes teoriassa "ottaa uskonnon paikkaa". ”… lähes mahdotonta yrittää käyttää auktoriteettia apuna...”

        Auktoriteetilla tarkoitin juuri tavalliselle ihmiselle merkityksellistä henkilöä. Ihmiset suorastaan janoavat näitä henkilöitä. Syystä tai toisesta joukot tarvitsevat näitä, joiden mukaan myös pyrkivät toimimaan. Huipulla voi olla yksinäistä, mutta vähän alemmalla tasolla eri tutkimukset ja niistä johdetetut päätelmät ovat kyllä suoraan lainattuja joltain mikä sillä hetkellä on auktoriteetti. Esimerkkinä vaikka psykologian ja psykiatrian tieteiden kehitys. Freud teorioineen on laulettu useaan kertaan suohon, mutta yhä on näitä jotka vannovat hänen nimeensä. Eikä Marx ole heistä se vähäisin.

        Aikoinaan ennustettiin että uskonnot tulevat häviämään, koska niitä ei tarvita. Tiede syrjäyttää tarpeen niihin, usko korvataan tiedolla. Näin ei vain olekaan käynyt. Itse asiassa tarve uskoa ihan vaikka mihin on lisääntynyt. Hämmästyttävintä minusta on että nyt kaivataan niitä vanhoja ahdasmielisiä uskontoja. Miksi se tieto ei riitä?

        ”…elämänlaatusi olisi sittenkin parempi esimerkiksi jossain muussa uskonnossa tai uskonnottomana, mutta sitä on tietenkin vähän vaikea kovin kattavasti kokeilla ja testailla…”

        Miksi tiede ei voi sitä testata? Omalta kohdalta voin tehdä vertailuja vain kokemukseni kautta. Jos olen ollut ateisti noin 20 vuotta ja sitten hiljalleen alkanutkin miettiä asioista toisin, niin voin kertoa että henkisesti laatu parani. Toki ja totta kai voi omaa asennettaan muuttaa muutoinkin, en vain keksinyt siihen toimivaa keinoa. Eikä kukaan osannut edes neuvoa muuta. Tai anteeksi, kyllä, minulle tarjottiin varsin kyynistä asennetta joka olisi vaatinut oman itseni turruttamista.

        Olen vuosien saatossa tutustunut moniin uskontoihin sekä tapoihin uskoa. Eniten teki vaikutuksen se henkinen voima joka useimmissa uskovissa on.
        Ihmisille on suuria asioita sellaiset kuin rakkaus, luottamus, yhteenkuuluvuus jne. Samoin elämää ylläpitäviä asioita on toivo ja usko siihen että jotain hyvää on olemassa. Kyllä näitäkin yritetään tutkitaan tieteellisesti. Tiede ei vain voi poistaa noita ilmiöitä ja kokemuksena ovat hyvin syvällisiä, johtuivat ne sitten mistä aivokemiasta tahansa. Ne myös määräävät sen, kuinka me elämämme koemme ja kuinka toimimme.

        Kun aikoinaan keskustelin ateistien kanssa mm. moraalista ja rakastamisesta, en oikein saanut kunnollisia vastauksia. Miksi meidän pitää yrittää tehdä hyvää, ellei meillä ole tiettyjä sääntöjä sekä uskoa siihen että noiden sääntöjen avulla voimme luoda parempi yhteiskunta. Meillä tulee olla myös uskoa siihen että muutkin ihmiset pyrkivät noudattamaan samaa moraalia. No, he tarjosivat tähän kyllin raskaita sanktioita jotta edes pelon avulla homma sujuu. Yllättävää kyllä anarkisteilla oli hyvin mielenkiintoinen selitys. He uskovat että kun kaikki saa tehdä mitä lystää, ja kun huomataan mitä siitä seuraa, löytyy automaattisesti nuo säännöt.

        Lopuksi, kuinka rahamarkkina toimivat. Kukaan ei osaa eikä pysty ennustamaan, mihin suuntaan ja milloin kurssit laskevat ja nousevat. Pienikin huhu vaikka Amerikasta siihen suuntaan, että nyt menee huonosti ja aalto alkaa. Se onko se todenperäinen vai ei, sillä ei ole merkitystä. ihmiset käyttäytyvät siis kaikesta huolimatta yhtä epärationaalisesti kuin ennen tiedettäkin.

        Anteeksi, vastaus vähän viipyi kun en ole ehtinyt kunnolla koneen ääreen. Joudun nytkin vastaamaan vähän pikaisesti.

        "Kun mennään vaikka historiaan tai vieläkin kauemmas esihistoriaan jne. joudumme löydöstemme pohjalta olettamaan. Samoin jos kehitämme vaikka uuden lääkkeen, joudumme olettamaan mitkä ovatkaan vaikutukset vaikka 30 vuoden kuluttua."

        Näinhän se on, mitä kauemmas mennään, sitä pienemmällä todennäköisyydellä osataan sanoa mitä todella on tapahtunut. Mutta jotain tietoa kuitenkin saadaan, ja sekin on parempi kuin nolla. Kun vielä yhdistetään erilaiset tutkimukset ja verrataan niiden tuloksia, niin pystytään jo päättelemään paljon, ja sulkemaan pois kaikenlaista.

        "…Esimerkkinä vaikka psykologian ja psykiatrian tieteiden kehitys. Freud teorioineen on laulettu useaan kertaan suohon, mutta yhä on näitä jotka vannovat hänen nimeensä. Eikä Marx ole heistä se vähäisin."

        Mutta tuossahan ei ole kyse tieteestä, vaan sokeasta auktoriteettiuskosta. Ei tuollaisesta ilmiöstä pidä kritisoida tiedettä, vaan ennemminkin tieteen liian vähäistä arvostusta, jos kritiikki ei ole mennyt perille.

        En kyllä tiedä mikä on Freudin arvostus nykyään. Marxista en ainakaan ole aikoihin kuullut kenenkään tosissaan meuhkaavan (ja oikeastihan Marx on sitäpaitsi hyvin väärinymmärretty, koska luullaan että jokin marxismi-leninismi olisi Marxin teorioiden mukaista).

        "Aikoinaan ennustettiin että uskonnot tulevat häviämään, koska niitä ei tarvita. Tiede syrjäyttää tarpeen niihin, usko korvataan tiedolla. Näin ei vain olekaan käynyt."

        Eikö? Uskonnottomien määrä on noussut tasaisesti nimenomaan niissä maissa, joissa koulutus on parempitasoista, ja tietoa enemmän saatavilla. En tiedä tulevatko uskonnot koskaan ihan kokonaan häviämään, mutta kyllä trendi on ihan selvä.

        "Miksi tiede ei voi sitä testata? Omalta kohdalta voin tehdä vertailuja vain kokemukseni kautta."

        Puhuin tosiaan juuri sinun elämäsi laadusta. Se selviäisi vain sillä, että itse kokeilisit eri uskontoja, eikä taida olla oikein mahdollista ruveta tuosta vaan uskomaan kokeeksi johonkin toiseen jumalaan, tai olemaan uskomatta mihinkään.. :) Ainakaan minä en osaisi alkaa edes kokeeksi tuosta vaan uskomaan...

        "Samoin elämää ylläpitäviä asioita on toivo ja usko siihen että jotain hyvää on olemassa. Kyllä näitäkin yritetään tutkitaan tieteellisesti. Tiede ei vain voi poistaa noita ilmiöitä ja kokemuksena ovat hyvin syvällisiä, johtuivat ne sitten mistä aivokemiasta tahansa."

        Miksi tiede niitä haluaisikaan poistaa? Tiede vain tutkii. Ihmismielen voima on suuri, ja sen toiminta on usein lähes käsittämätöntä. Tunteet kuuluvat tietoisuuden toimintaan.


      • Schlechterwisser kirjoitti:

        Anteeksi, vastaus vähän viipyi kun en ole ehtinyt kunnolla koneen ääreen. Joudun nytkin vastaamaan vähän pikaisesti.

        "Kun mennään vaikka historiaan tai vieläkin kauemmas esihistoriaan jne. joudumme löydöstemme pohjalta olettamaan. Samoin jos kehitämme vaikka uuden lääkkeen, joudumme olettamaan mitkä ovatkaan vaikutukset vaikka 30 vuoden kuluttua."

        Näinhän se on, mitä kauemmas mennään, sitä pienemmällä todennäköisyydellä osataan sanoa mitä todella on tapahtunut. Mutta jotain tietoa kuitenkin saadaan, ja sekin on parempi kuin nolla. Kun vielä yhdistetään erilaiset tutkimukset ja verrataan niiden tuloksia, niin pystytään jo päättelemään paljon, ja sulkemaan pois kaikenlaista.

        "…Esimerkkinä vaikka psykologian ja psykiatrian tieteiden kehitys. Freud teorioineen on laulettu useaan kertaan suohon, mutta yhä on näitä jotka vannovat hänen nimeensä. Eikä Marx ole heistä se vähäisin."

        Mutta tuossahan ei ole kyse tieteestä, vaan sokeasta auktoriteettiuskosta. Ei tuollaisesta ilmiöstä pidä kritisoida tiedettä, vaan ennemminkin tieteen liian vähäistä arvostusta, jos kritiikki ei ole mennyt perille.

        En kyllä tiedä mikä on Freudin arvostus nykyään. Marxista en ainakaan ole aikoihin kuullut kenenkään tosissaan meuhkaavan (ja oikeastihan Marx on sitäpaitsi hyvin väärinymmärretty, koska luullaan että jokin marxismi-leninismi olisi Marxin teorioiden mukaista).

        "Aikoinaan ennustettiin että uskonnot tulevat häviämään, koska niitä ei tarvita. Tiede syrjäyttää tarpeen niihin, usko korvataan tiedolla. Näin ei vain olekaan käynyt."

        Eikö? Uskonnottomien määrä on noussut tasaisesti nimenomaan niissä maissa, joissa koulutus on parempitasoista, ja tietoa enemmän saatavilla. En tiedä tulevatko uskonnot koskaan ihan kokonaan häviämään, mutta kyllä trendi on ihan selvä.

        "Miksi tiede ei voi sitä testata? Omalta kohdalta voin tehdä vertailuja vain kokemukseni kautta."

        Puhuin tosiaan juuri sinun elämäsi laadusta. Se selviäisi vain sillä, että itse kokeilisit eri uskontoja, eikä taida olla oikein mahdollista ruveta tuosta vaan uskomaan kokeeksi johonkin toiseen jumalaan, tai olemaan uskomatta mihinkään.. :) Ainakaan minä en osaisi alkaa edes kokeeksi tuosta vaan uskomaan...

        "Samoin elämää ylläpitäviä asioita on toivo ja usko siihen että jotain hyvää on olemassa. Kyllä näitäkin yritetään tutkitaan tieteellisesti. Tiede ei vain voi poistaa noita ilmiöitä ja kokemuksena ovat hyvin syvällisiä, johtuivat ne sitten mistä aivokemiasta tahansa."

        Miksi tiede niitä haluaisikaan poistaa? Tiede vain tutkii. Ihmismielen voima on suuri, ja sen toiminta on usein lähes käsittämätöntä. Tunteet kuuluvat tietoisuuden toimintaan.

        "Kun aikoinaan keskustelin ateistien kanssa mm. moraalista ja rakastamisesta, en oikein saanut kunnollisia vastauksia. Miksi meidän pitää yrittää tehdä hyvää, ellei meillä ole tiettyjä sääntöjä sekä uskoa siihen että noiden sääntöjen avulla voimme luoda parempi yhteiskunta."

        Erikoista. Itse olen kuullut parhaat vastaukset nimenomaan ateisteilta. Uskonnon vastaukset tuppaavat olemaan enemmän tyyppiä "koska Raamattu sanoo niin", eikä se ole mitään eettistä ajattelua - se on vain sokeaa auktoriteettiuskoa. Harmi kun en nyt ehdi alkaa etsimään linkkejä tai muuta, mutta on ihan selvä, että moraali ei perustu uskontoihin. Tästä on olemassa esim. tutkimuksia joissa todettiin että eri uskontoihin uskovat ja uskonnottomat pitivät lähes tarkalleen samoja asioita oikeina ja väärinä. Raamatun "tee muille kuten haluaisit itsellesi tehtävän" -tyyppiset ajatukset ovat yhteisiä varmaan kaikille moraalijärjestelmille, eivätkä suoraan sanoen vaadi hirveästi viisautta. Empatiakyky ja yhdessäelämisen perustaidot ovat ihmislajin luontaisia kykyjä, jotka perustuvat tuhansien tai jopa miljoonien vuosien evoluutioon laumaeläimenä. Nimenomaan ateistit painottavat yleensä tasa-arvoa ja suvaitsevaisuutta, ja toimivat mielestäni pyyteettömämmin esimerkiksi hyväntekeväisyyttä harrastaessaan. Uskonnottomat avustusjärjestötkään eivät ole liikkeellä taka-ajatuksenaan käännyttää pakanoita, vaan pelkästä auttamisen halusta. Ateisteja ei mikään näytä estävän haluamasta tehdä tästä maailmasta parempi paikka meille kaikille - ei pelkästään samanuskoisille ja seksuaalisesti samalla tavalla käyttäytyville. Eikä heiltä näytä puuttuvan uskoa siihen.

        Aiheesta on paljon kirjoituksia netissäkin, ja moraalista on kirjoitettu paljon kirjoja. Sieltä voi löytää tarkempia tietoja siitä, miksi ateisteillakin on moraali (vaikka jotkut uskovaiset eivät tunnu sitä millään ymmärtävän).

        "No, he tarjosivat tähän kyllin raskaita sanktioita jotta edes pelon avulla homma sujuu."

        Tarjosivatko ikuista kidutusta? :) Sori, vitsi vain. Kenenköhän kanssa olet keskustellut…?

        "Kukaan ei osaa eikä pysty ennustamaan, mihin suuntaan ja milloin kurssit laskevat ja nousevat. Pienikin huhu vaikka Amerikasta siihen suuntaan, että nyt menee huonosti ja aalto alkaa. Se onko se todenperäinen vai ei, sillä ei ole merkitystä. ihmiset käyttäytyvät siis kaikesta huolimatta yhtä epärationaalisesti kuin ennen tiedettäkin."

        Noh, onko reilua syyttää tiedettä keinottelijoiden puuhista? Ei talousjärjestelmää ole päässyt mikään tieteellinen diktatoorinen neuvottelukunta rakentamaan. Se on syntynyt ihan poliitikkojen ja liikemiesten kesken vapaasti satojen vuosien aikana. Toki tiede yrittää sen toimintaa ymmärtää, mutta markkinatalous on sen verran kaoottinen järjestelmä, kun toimijoina on muutama miljardi ihmistä, joilla kaikilla on oma vapaa tahto (ainakin suhteellisen) ja omat motiivit jotka eivät perustu rationaaliseen päätöksentekoon, että eihän sellaista ennustamaan pysty. Se nyt on vain ihan mahdotonta. Jos taloutta haluaa hallita, sitä pitäisi säädellä pienintä piirtoa myöten, eikä ihmisten tarpeita ja yhteiskunnan kehitystä ym. pysty ennustamaan niin, että se olisi mahdollista. Sehän nähtiin hyvin sosialistisissa viisivuotissuunnitelmissa…


      • Nasse.
        epikuros kirjoitti:

        sanaa - uus voitaisiin käyttää monissa yhteyksissäilman että se tuo mitään lisäarvoa entiselle sanalle - ateismi. Uusoikeisto, uusherännäisyys, uuslastaadiolaisuus ym. ym.
        Se on vähän samaa kun jokin esim. poliittinen ryhämä nostaa päätään niin oikopäätä nostetaan kissa pöydälle ja huudetaan "se on sitä populismia"

        Märittele tarkemmin miten ns. ateismi ja uusateismi eroavat toisistaan.

        alistamaa, manipuloimaan ja vääristelemään.


      • Nas.se kirjoitti:

        rehellinen että ymmärätisi satukirjassaan olevan 4 kpl luomiskertomuksia. Kaikki erilaisia ja keskenään ristiriitaisia.

        Miten hihhuli reagoi? Luo ihan oman luomiskertomuksensa ja jauhaa mitä jauhaa, palstalla.

        kaikkien uskovan samalla tavalla ja olevan kreationisteja? Eivät kaikki ateistit ole kommunisteja.

        Jos olet laatinut tiukat lokerot ihmisille ja sitten kiukuttelet siitä, etten minä mahdukaan noihin lokeroihisi, niin eikö ongelma ole sinun?


      • mummomuori kirjoitti:

        kaikkien uskovan samalla tavalla ja olevan kreationisteja? Eivät kaikki ateistit ole kommunisteja.

        Jos olet laatinut tiukat lokerot ihmisille ja sitten kiukuttelet siitä, etten minä mahdukaan noihin lokeroihisi, niin eikö ongelma ole sinun?

        toisaalta esität luomasi Jumalan olevan se ainoa oikea ja toisaalta vaaadit että muunlaisiakin Jumalia pitäisi digata.

        Eiköhän sulla ole vähän nyt pihat solmussa?

        Toki sikäli olet oikeassa että kaikki tunnetut Jumalat ovat ihmisten mielikuvituksen tuotetta ja siinä mielessä samanarvoisia.


      • Nas.se kirjoitti:

        toisaalta esität luomasi Jumalan olevan se ainoa oikea ja toisaalta vaaadit että muunlaisiakin Jumalia pitäisi digata.

        Eiköhän sulla ole vähän nyt pihat solmussa?

        Toki sikäli olet oikeassa että kaikki tunnetut Jumalat ovat ihmisten mielikuvituksen tuotetta ja siinä mielessä samanarvoisia.

        ei voisi ymmärtää? Onko siinäkin jotain väärää? : )


      • zlkdbc
        Joo, jos... kirjoitti:

        ...sattuu olemaan tyypillinen vanhoilliskristillinen ev.lutka!

        "...että Jumalasta voi saada konkreettisen kokemuksen."

        Voit myös saada ufoista todellisen tuntuisen kokemuksen, jos tiettyyn paikkaan aivoihisi johdetaan sähköä.

        Ufoista voi saada kokemuksen ilman, että aivoihin johdetaan sähköä. Toinen asia on sitten se, mitä nämä ns. ufot ovat.


      • zlkdbc
        Nasse. kirjoitti:

        ja kuten tiedämme, sen jumalat, taikatemput ja loitsut ovat plagiointia aiemmista mytologoista.

        Jumalaharhasi ovat harhoja, aivojen virhetoimintoja. Sinun konkreettinen jumalasi kuolee mukanasi.

        Toisekseen, jos isnä olet sitä mieltä että kokemukset Jumalsta todistavat sen olevan olemassa, joudut myöntämään että kilpailevien väkivaltauskontojen ja muiden uskontojen Jumalat ovat olemassa.

        Mikäli väität että oma mielihyväjumalasi on ainoa todellinen, syylliistyt (siinäkin) älylliseen epärehellisyyteen.

        Mikäli oma mielihyväkatsantokantasi maailmasta on ainoa todellinen, syyllistyt älylliseen epärehellisyyteen. Joudut myöntämään, että on ja ei ole jumalia.

        Raamattu selittää kaikkein parhaiten muiden uskontojen jumalat, jotka ovat pahuuden henkivaltoja, sikäli kun antavat kokemuksia kannattajilleen eli eivät ole kuolleita keksittyjä mielikuvituksen tuotteita.


      • Millä tutkit
        cei5 kirjoitti:

        Kun ihminen tai mikä tahansa eliö kuolee, niin se lakkaa olemasta. Sielua ei ole sen enempää sinulla kuin nasu-possullakaan.

        Pyhän hengen siittämistarinaa ei ole tutkittu tieteellisesti, koska se on vain tarina, taru, uskomus.

        Jumalan Hengen ja Hänen tekonsa ???


      • mummomuori kirjoitti:

        pitäisi niin tehdä? Johan mm. katollinen ja ortodoksinen kirkko toimii kuvaamallasi tavalla. Itse olen liittynyt kirkkoon juuri tuon moninaisuuden takia ja että kirkossa on tilaa erilaisuudelle. Onhan kirkon piirissä myös lestadiolaisia, körttejä jne. eli onko mielestäsi heidän vallattava koko kirkko?

        Sittenpä voi olla että moni lähtee etsimään itselleen sopivampaa paikkaa - ainakin minä teen niin.

        Kirkolla pitää olla vain yksi kanta, eikä liiioin kasvattaa erilaisia kantoja. Kirkko tulee säilyttää yhteisönä, eikä konservatiivisten ihmisten lahkona. Sinne suuntaanhan tässä ollaan jatkuvasti menossa, kun monet tapakristitytkin eroavat kirkosta, kun kirkko mielivaltaisesti esittää jokaisen kirkon työntekijän erilaiset kannat.
        Kirkon kanta pitää tulla kirkon päättäviltä elimiltä, jota tulee jokaisen kirkon työntekijän noudattaa tai muussa tapauksessa ko. henkilö saa lähteä ja hakea itsensä jonnekin prostestanttikirkon papiksi, jossa konservatiivisuus on jokapäiväistä.

        Ei täällä voi silloin olla "kirkko kuulolla" (tällöin kaikilla kirkon työntekijöillä tulisi olla samanlainen kanta asioihin), eikä niin että jokaisella kirkon työntekijällä on vuorotellen erilainen kanta asioihin?

        Siis täällä kysytään yleensä kirkon kantaa asiaan, eikä yksittäisen kirkon työntekijän kantaa asioihin eli toisin sanoen kirkolla ei ole mitään kantaa asioihin.


      • uskonnollinen
        kansalainen.. kirjoitti:

        neandertalinihmisetkin ennen Adamia? Kuuluivatko hekin rakennustyömaavaiheseen?
        Jos eivät kuuluneet, niin miksi heitä ei enää ole?

        Eihän Neandertalilaiset olleet nykyihmisiä lainkaan. Se vähän minkä evoluutioteoriasta tiedän pitää sisällään sen että nuo kaverit hävisivät jostain tuntemattomasta syystä maapallolta ja tilalle tuli nykyihminen.


      • uskonnollinen kirjoitti:

        Eihän Neandertalilaiset olleet nykyihmisiä lainkaan. Se vähän minkä evoluutioteoriasta tiedän pitää sisällään sen että nuo kaverit hävisivät jostain tuntemattomasta syystä maapallolta ja tilalle tuli nykyihminen.

        Kyllä meissä vielä neandertalilaista vertakin virtaa...
        http://fi.wikipedia.org/wiki/Neandertalinihminen#Kiistely.C3.A4_neandertalilaisten_ja_nykyihmisen_geenivirrasta

        Neandertalin ihmiset katosivat samalla tavalla kuin moni muukin kansa. He sulautuivat.


    • luterilainen.

      "- Miksi ihmisellä on häntäluu? Eikö se ole todiste evoluutiosta?
      http://fi.wikipedia.org/wiki/Häntäluu"

      Miksi evoluutio toi ihmiselle häntäluun ja miten se todistaa siitä?

      • Et tainnut lukea wikipediasta. Ajatellaan, että kun laji kehittyy niin silloin jokin osa surkastuu ja jokin kehittyy entisestään. Meidän kantaisällämme on ollut siis häntä.

        Sinustako häntäluu pitää perslihaksia paikoillaan ja toimii iskunvaimentajana. En tiedä vielä niin surkeata iskunvaimentajaa kuin kuin häntäluu, mikäli sen ois jokin ihmiselle mukamas luonut.

        Miten voi olla mahdollista, että simpanssilla ja ihmisellä on yli 95% samankaltainen geeniperimä, mikäli se ei olisi todiste evoluutiosta?


      • Quicunque
        ateisti1982 kirjoitti:

        Et tainnut lukea wikipediasta. Ajatellaan, että kun laji kehittyy niin silloin jokin osa surkastuu ja jokin kehittyy entisestään. Meidän kantaisällämme on ollut siis häntä.

        Sinustako häntäluu pitää perslihaksia paikoillaan ja toimii iskunvaimentajana. En tiedä vielä niin surkeata iskunvaimentajaa kuin kuin häntäluu, mikäli sen ois jokin ihmiselle mukamas luonut.

        Miten voi olla mahdollista, että simpanssilla ja ihmisellä on yli 95% samankaltainen geeniperimä, mikäli se ei olisi todiste evoluutiosta?

        Siunausta sinulle ateisti1982. Olet oikean asian äärellä jatkuvan kyselemisesi kanssa. Toivottavasti löydät totuuden, niin sinun ei tarvitse olla koko ajan vihainen. Sanon tämän ystävänä.

        Olen koulutukseltani kemian DI ja filosofian opettaja ja hyvin paljon käsitellyt tieteen ja uskon välistä suhdetta. En ole koskaan kokenut ristiriitaa omissa opinnoissani tai opetuksessani tieteen ja uskon välillä. Sen sijaan, kun mennään tarpeeksi syvälle molemmissa, niin huomataan monia yhtäläisyyksiä. Molekyylitasolla asiat tapahtuvat teoriassa sattumalta, mutta se johtuu vain siitä, ettemme osaa kutsua sitä muuksi kuin sattumaksi. Emme pysty määrittelemään molekyylien liikkeitä, vaikka ne tapahtuvat silmiemme edessä. Kuinka pystyisimme siis määrittelemään maailmankaikkeuden liikkeitä aikojen alussa?

        Energian määrä maailmankaikkeudessa on vakio. Energiaa ei voi syntyä tyhjästä, se vain muuttaa muotoaan. Ei ole siis mahdollista, että maailmankaikkeus olisi voinut syntyä tyhjästä. Jotain on siis täytynyt olla aina ja tulee aina olemaan. On turha kuvitella, että olisimme niin älykkäitä, että pystyisimme koskaan ratkaisemaan näitä arvoituksia. Asian edessä on vain nöyrryttävä ja tunnustettava oma vajavaisuutensa.

        Ja mitä evoluutioon tulee, niin on aivan samantekevää kuinka kehitys tapahtuu. Me emme ole edes niin älykkäitä, että osaisimme selittää puuttuvien lenkkien puutteen, saati molekyylien liikkeen. On siis turha puhua evoluutiosta tieteenä. Se perustuu uskomuksiin siinä kuin muutkin metafyysiset olettamukset. Tieteen yksi perusperiaatteista on mitattavuus. Se mitä ei voida mitata, ei ole siis tiedettä.

        Lopuksi Herramme Jeesuksen sana, joka on tarkoitettu juuri sinulle. Lue se ajatuksella. Se ei hyökkää sinua vastaan vaan haluaa sinulle hyvää. Älä sinäkään siis hyökkää sitä vastaan.

        "Tulkaa minun luokseni kaikki te työn ja kuormien uuvuttamat. Minä annan teille levon. Ottakaa minun ikeeni harteillenne ja katsokaa minua: minä olen sydämeltäni lempeä ja nöyrä. Näin teidän sielunne löytää levon. Minun ikeeni on hyvä kantaa ja minun kuormani on kevyt." Matt. 11: 28-30


        Ystävällisin terveisin, Mikko Rantalahti (mikko.rantalahti[at]gmail.com)


      • E=mc2
        Quicunque kirjoitti:

        Siunausta sinulle ateisti1982. Olet oikean asian äärellä jatkuvan kyselemisesi kanssa. Toivottavasti löydät totuuden, niin sinun ei tarvitse olla koko ajan vihainen. Sanon tämän ystävänä.

        Olen koulutukseltani kemian DI ja filosofian opettaja ja hyvin paljon käsitellyt tieteen ja uskon välistä suhdetta. En ole koskaan kokenut ristiriitaa omissa opinnoissani tai opetuksessani tieteen ja uskon välillä. Sen sijaan, kun mennään tarpeeksi syvälle molemmissa, niin huomataan monia yhtäläisyyksiä. Molekyylitasolla asiat tapahtuvat teoriassa sattumalta, mutta se johtuu vain siitä, ettemme osaa kutsua sitä muuksi kuin sattumaksi. Emme pysty määrittelemään molekyylien liikkeitä, vaikka ne tapahtuvat silmiemme edessä. Kuinka pystyisimme siis määrittelemään maailmankaikkeuden liikkeitä aikojen alussa?

        Energian määrä maailmankaikkeudessa on vakio. Energiaa ei voi syntyä tyhjästä, se vain muuttaa muotoaan. Ei ole siis mahdollista, että maailmankaikkeus olisi voinut syntyä tyhjästä. Jotain on siis täytynyt olla aina ja tulee aina olemaan. On turha kuvitella, että olisimme niin älykkäitä, että pystyisimme koskaan ratkaisemaan näitä arvoituksia. Asian edessä on vain nöyrryttävä ja tunnustettava oma vajavaisuutensa.

        Ja mitä evoluutioon tulee, niin on aivan samantekevää kuinka kehitys tapahtuu. Me emme ole edes niin älykkäitä, että osaisimme selittää puuttuvien lenkkien puutteen, saati molekyylien liikkeen. On siis turha puhua evoluutiosta tieteenä. Se perustuu uskomuksiin siinä kuin muutkin metafyysiset olettamukset. Tieteen yksi perusperiaatteista on mitattavuus. Se mitä ei voida mitata, ei ole siis tiedettä.

        Lopuksi Herramme Jeesuksen sana, joka on tarkoitettu juuri sinulle. Lue se ajatuksella. Se ei hyökkää sinua vastaan vaan haluaa sinulle hyvää. Älä sinäkään siis hyökkää sitä vastaan.

        "Tulkaa minun luokseni kaikki te työn ja kuormien uuvuttamat. Minä annan teille levon. Ottakaa minun ikeeni harteillenne ja katsokaa minua: minä olen sydämeltäni lempeä ja nöyrä. Näin teidän sielunne löytää levon. Minun ikeeni on hyvä kantaa ja minun kuormani on kevyt." Matt. 11: 28-30


        Ystävällisin terveisin, Mikko Rantalahti (mikko.rantalahti[at]gmail.com)

        "Energian määrä maailmankaikkeudessa on vakio. Energiaa ei voi syntyä tyhjästä, se vain muuttaa muotoaan. Ei ole siis mahdollista, että maailmankaikkeus olisi voinut syntyä tyhjästä."

        Ei siis ole mahdollista, että Jeesus olisi poistunut maailmankaikkeudesta. Hän on jossain tuolla avaruudessa, luultavasti kuollut uudelleen heti noustuaan pois maan ilmakehästä.


      • Quicunque kirjoitti:

        Siunausta sinulle ateisti1982. Olet oikean asian äärellä jatkuvan kyselemisesi kanssa. Toivottavasti löydät totuuden, niin sinun ei tarvitse olla koko ajan vihainen. Sanon tämän ystävänä.

        Olen koulutukseltani kemian DI ja filosofian opettaja ja hyvin paljon käsitellyt tieteen ja uskon välistä suhdetta. En ole koskaan kokenut ristiriitaa omissa opinnoissani tai opetuksessani tieteen ja uskon välillä. Sen sijaan, kun mennään tarpeeksi syvälle molemmissa, niin huomataan monia yhtäläisyyksiä. Molekyylitasolla asiat tapahtuvat teoriassa sattumalta, mutta se johtuu vain siitä, ettemme osaa kutsua sitä muuksi kuin sattumaksi. Emme pysty määrittelemään molekyylien liikkeitä, vaikka ne tapahtuvat silmiemme edessä. Kuinka pystyisimme siis määrittelemään maailmankaikkeuden liikkeitä aikojen alussa?

        Energian määrä maailmankaikkeudessa on vakio. Energiaa ei voi syntyä tyhjästä, se vain muuttaa muotoaan. Ei ole siis mahdollista, että maailmankaikkeus olisi voinut syntyä tyhjästä. Jotain on siis täytynyt olla aina ja tulee aina olemaan. On turha kuvitella, että olisimme niin älykkäitä, että pystyisimme koskaan ratkaisemaan näitä arvoituksia. Asian edessä on vain nöyrryttävä ja tunnustettava oma vajavaisuutensa.

        Ja mitä evoluutioon tulee, niin on aivan samantekevää kuinka kehitys tapahtuu. Me emme ole edes niin älykkäitä, että osaisimme selittää puuttuvien lenkkien puutteen, saati molekyylien liikkeen. On siis turha puhua evoluutiosta tieteenä. Se perustuu uskomuksiin siinä kuin muutkin metafyysiset olettamukset. Tieteen yksi perusperiaatteista on mitattavuus. Se mitä ei voida mitata, ei ole siis tiedettä.

        Lopuksi Herramme Jeesuksen sana, joka on tarkoitettu juuri sinulle. Lue se ajatuksella. Se ei hyökkää sinua vastaan vaan haluaa sinulle hyvää. Älä sinäkään siis hyökkää sitä vastaan.

        "Tulkaa minun luokseni kaikki te työn ja kuormien uuvuttamat. Minä annan teille levon. Ottakaa minun ikeeni harteillenne ja katsokaa minua: minä olen sydämeltäni lempeä ja nöyrä. Näin teidän sielunne löytää levon. Minun ikeeni on hyvä kantaa ja minun kuormani on kevyt." Matt. 11: 28-30


        Ystävällisin terveisin, Mikko Rantalahti (mikko.rantalahti[at]gmail.com)

        Lue Hawkingia, saattaisit oppia jotakin:

        However, neither observations nor theory indicates this to
        have been the case. The first law allows energy to convert from
        one type to another as long as the total for a closed system
        remains fixed. Remarkably, the total energy of the universe
        appears to be zero. As famed cosmologist Stephen Hawking said
        in his 1988 best seller, A Brief History of Time, "In the case of a
        universe that is approximately uniform in space, one can show
        that the negative gravitational energy exactly cancels the positive
        energy represented by the matter. So the total energy of the universe
        is zero.4 Specifically, within small measurement errors, the
        mean energy density of the universe is exactly what it should be
        for a universe that appeared from an initial state of zero energy,
        within a small quantum uncertainty.5
        A close balance between positive and negative energy is predicted
        by the modern extension of the big bang theory called the
        inflationary big bang, according to which the universe underwent a
        period of rapid, exponential inflation during a tiny fraction of its
        first second.6 The inflationary theory has recently undergone a
        number of stringent observational tests that would have been sufficient
        to prove it false. So far, it has successfully passed all these tests.
        In short, the existence of matter and energy in the universe did
        not require the violation of energy conservation at the assumed
        creation. In fact, the data strongly support the hypothesis that no
        such miracle occurred. If we regard such a miracle as predicted by
        the creator hypothesis, then that prediction is not confirmed.
        (Victor Stenger: God - The failed hypothesis)

        Ja evoluutio on tunnettu *fakta*. Sitä taas puolestaan selittää evoluutioteoria.


      • Quicunque kirjoitti:

        Siunausta sinulle ateisti1982. Olet oikean asian äärellä jatkuvan kyselemisesi kanssa. Toivottavasti löydät totuuden, niin sinun ei tarvitse olla koko ajan vihainen. Sanon tämän ystävänä.

        Olen koulutukseltani kemian DI ja filosofian opettaja ja hyvin paljon käsitellyt tieteen ja uskon välistä suhdetta. En ole koskaan kokenut ristiriitaa omissa opinnoissani tai opetuksessani tieteen ja uskon välillä. Sen sijaan, kun mennään tarpeeksi syvälle molemmissa, niin huomataan monia yhtäläisyyksiä. Molekyylitasolla asiat tapahtuvat teoriassa sattumalta, mutta se johtuu vain siitä, ettemme osaa kutsua sitä muuksi kuin sattumaksi. Emme pysty määrittelemään molekyylien liikkeitä, vaikka ne tapahtuvat silmiemme edessä. Kuinka pystyisimme siis määrittelemään maailmankaikkeuden liikkeitä aikojen alussa?

        Energian määrä maailmankaikkeudessa on vakio. Energiaa ei voi syntyä tyhjästä, se vain muuttaa muotoaan. Ei ole siis mahdollista, että maailmankaikkeus olisi voinut syntyä tyhjästä. Jotain on siis täytynyt olla aina ja tulee aina olemaan. On turha kuvitella, että olisimme niin älykkäitä, että pystyisimme koskaan ratkaisemaan näitä arvoituksia. Asian edessä on vain nöyrryttävä ja tunnustettava oma vajavaisuutensa.

        Ja mitä evoluutioon tulee, niin on aivan samantekevää kuinka kehitys tapahtuu. Me emme ole edes niin älykkäitä, että osaisimme selittää puuttuvien lenkkien puutteen, saati molekyylien liikkeen. On siis turha puhua evoluutiosta tieteenä. Se perustuu uskomuksiin siinä kuin muutkin metafyysiset olettamukset. Tieteen yksi perusperiaatteista on mitattavuus. Se mitä ei voida mitata, ei ole siis tiedettä.

        Lopuksi Herramme Jeesuksen sana, joka on tarkoitettu juuri sinulle. Lue se ajatuksella. Se ei hyökkää sinua vastaan vaan haluaa sinulle hyvää. Älä sinäkään siis hyökkää sitä vastaan.

        "Tulkaa minun luokseni kaikki te työn ja kuormien uuvuttamat. Minä annan teille levon. Ottakaa minun ikeeni harteillenne ja katsokaa minua: minä olen sydämeltäni lempeä ja nöyrä. Näin teidän sielunne löytää levon. Minun ikeeni on hyvä kantaa ja minun kuormani on kevyt." Matt. 11: 28-30


        Ystävällisin terveisin, Mikko Rantalahti (mikko.rantalahti[at]gmail.com)

        - "Molekyylitasolla asiat tapahtuvat teoriassa sattumalta, mutta se johtuu vain siitä, ettemme osaa kutsua sitä muuksi kuin sattumaksi. Emme pysty määrittelemään molekyylien liikkeitä, vaikka ne tapahtuvat silmiemme edessä."

        Molekyylitasolla? Tuota en tiennytkään. Onko sinulla jotain esimerkkejä teoriassa sattumalta tapahtuvista molekyylitason tapahtumista?

        - "Energian määrä maailmankaikkeudessa on vakio."

        Nollakin on vakio.

        - "Me emme ole edes niin älykkäitä, että osaisimme selittää puuttuvien lenkkien puutteen..."

        Mitäs lenkkejä sinulta vielä puuttuu?

        - "On siis turha puhua evoluutiosta tieteenä"

        Niin on. Evoluutio ei ole tiede.

        - "Se perustuu uskomuksiin siinä kuin muutkin metafyysiset olettamukset."

        On sinun onnesi että te protestantit ette joudu helvettiin valehtelemisesta.

        - "Se mitä ei voida mitata, ei ole siis tiedettä."

        Miten tämä liittyy evoluutioon?


      • Quicunque
        Schlechterwisser kirjoitti:

        - "Molekyylitasolla asiat tapahtuvat teoriassa sattumalta, mutta se johtuu vain siitä, ettemme osaa kutsua sitä muuksi kuin sattumaksi. Emme pysty määrittelemään molekyylien liikkeitä, vaikka ne tapahtuvat silmiemme edessä."

        Molekyylitasolla? Tuota en tiennytkään. Onko sinulla jotain esimerkkejä teoriassa sattumalta tapahtuvista molekyylitason tapahtumista?

        - "Energian määrä maailmankaikkeudessa on vakio."

        Nollakin on vakio.

        - "Me emme ole edes niin älykkäitä, että osaisimme selittää puuttuvien lenkkien puutteen..."

        Mitäs lenkkejä sinulta vielä puuttuu?

        - "On siis turha puhua evoluutiosta tieteenä"

        Niin on. Evoluutio ei ole tiede.

        - "Se perustuu uskomuksiin siinä kuin muutkin metafyysiset olettamukset."

        On sinun onnesi että te protestantit ette joudu helvettiin valehtelemisesta.

        - "Se mitä ei voida mitata, ei ole siis tiedettä."

        Miten tämä liittyy evoluutioon?

        Hawkingista: Teksti jota halusit minun tutkivan sisältää käsitteet negatiivisesta ja positiivisesta aineesta, jotka kumoavat toisensa. Siinä puhutaan siis matemaattisesti positiivisesta ja negatiivisesta. Aine ja vasta-aine kumoavat toisensa ja muodostuu nolla. Kuitenkin kyseisessä teoriassa on se ongelma, että se ylipäänsä sisältää termit aine ja vasta-aine. Jotain siis on, vaikka ne muodostaisivatkin yhteensä nollan (mittavirheitä lukuunottamatta, kuten tekstistäkin ilmeni). Jos jotain siis on (aine ja vasta-aine), ja vaikka ne muodostaisivatkin yhteensä matemaattisesti nollan, niin siitä ei voida vetää johtopäätöstä, että jotain olisi syntynyt tyhjästä, koska aine ja vasta-aine ovat olleet olemassa. Tyhjä on eri asia kuin nolla. Hawkingin mukaan nollasta voi syntyä jotain(, mikä on ainakin meikäläisen hankala käsittää, paitsi jos puhutaan matemaattisesta nollasta, jossa vastavoimat kumoavat toisensa), mutta tyhjästä ei edelleenkään voi syntyä mitään. (Kosmologia on kyllä mielenkiintoista ja siinä ollaan oikean asian äärellä. Siinä on vaan ylipäänsä se ongelma, että se on jatkuvaa teorioiden päivitystä ja käsitteiden muokkaamista, kun huomataan, että taas on menty vikaan. Mielestäni on kuitenkin hienoa, että jotkut osaavat tutkia asioita kuudensissa tai jopa seitsemänsissä ja suuremmissa ulottuvuuksissa ja vetää siitä johtopäätöksiä vallitsevaan maailmaan. Itse en osaisi. Tosin en tiedä kuinka relevanttia tietoa eri ulottuvuuksia tutkimalla voidaan saada näihin kolmeen vallitsevaan ulottuvuuteen, joissa elämme, mutta ehkäpä he (tutkijat) tietävät.) :)

        "Molekyylitasolla? Tuota en tiennytkään. Onko sinulla jotain esimerkkejä teoriassa sattumalta tapahtuvista molekyylitason tapahtumista?"

        Heisenbergin epävarmuusperiaatteen mukaisesti emme voi tietää tarkasti kappaleiden sijaintia emmekä liikettä. Tästä Schrödinger kehitteli mallin joka englanniksi kulkee nimellä "The particle in a Box". Sen mukaan (rajusti yksinkertaistaen) rajaamme alueen, jonka sisällä reaktiot tapahtuvat ja laskemme todennäköisyyksiä reaktioiden tapahtumille. Jotkut reaktiot tapahtuvat todennäköisemmin kuin toiset, mutta reaktioita voi olla silti useita jo pelkästään kahden molekyylin välillä. Reaktiot siis tapahtuvat sattumalta, emmekä voi tarkasti tietää mitä reaktiota kulloinkin tapahtuu vai tapahtuuko useita.


      • Quicunque
        Quicunque kirjoitti:

        Hawkingista: Teksti jota halusit minun tutkivan sisältää käsitteet negatiivisesta ja positiivisesta aineesta, jotka kumoavat toisensa. Siinä puhutaan siis matemaattisesti positiivisesta ja negatiivisesta. Aine ja vasta-aine kumoavat toisensa ja muodostuu nolla. Kuitenkin kyseisessä teoriassa on se ongelma, että se ylipäänsä sisältää termit aine ja vasta-aine. Jotain siis on, vaikka ne muodostaisivatkin yhteensä nollan (mittavirheitä lukuunottamatta, kuten tekstistäkin ilmeni). Jos jotain siis on (aine ja vasta-aine), ja vaikka ne muodostaisivatkin yhteensä matemaattisesti nollan, niin siitä ei voida vetää johtopäätöstä, että jotain olisi syntynyt tyhjästä, koska aine ja vasta-aine ovat olleet olemassa. Tyhjä on eri asia kuin nolla. Hawkingin mukaan nollasta voi syntyä jotain(, mikä on ainakin meikäläisen hankala käsittää, paitsi jos puhutaan matemaattisesta nollasta, jossa vastavoimat kumoavat toisensa), mutta tyhjästä ei edelleenkään voi syntyä mitään. (Kosmologia on kyllä mielenkiintoista ja siinä ollaan oikean asian äärellä. Siinä on vaan ylipäänsä se ongelma, että se on jatkuvaa teorioiden päivitystä ja käsitteiden muokkaamista, kun huomataan, että taas on menty vikaan. Mielestäni on kuitenkin hienoa, että jotkut osaavat tutkia asioita kuudensissa tai jopa seitsemänsissä ja suuremmissa ulottuvuuksissa ja vetää siitä johtopäätöksiä vallitsevaan maailmaan. Itse en osaisi. Tosin en tiedä kuinka relevanttia tietoa eri ulottuvuuksia tutkimalla voidaan saada näihin kolmeen vallitsevaan ulottuvuuteen, joissa elämme, mutta ehkäpä he (tutkijat) tietävät.) :)

        "Molekyylitasolla? Tuota en tiennytkään. Onko sinulla jotain esimerkkejä teoriassa sattumalta tapahtuvista molekyylitason tapahtumista?"

        Heisenbergin epävarmuusperiaatteen mukaisesti emme voi tietää tarkasti kappaleiden sijaintia emmekä liikettä. Tästä Schrödinger kehitteli mallin joka englanniksi kulkee nimellä "The particle in a Box". Sen mukaan (rajusti yksinkertaistaen) rajaamme alueen, jonka sisällä reaktiot tapahtuvat ja laskemme todennäköisyyksiä reaktioiden tapahtumille. Jotkut reaktiot tapahtuvat todennäköisemmin kuin toiset, mutta reaktioita voi olla silti useita jo pelkästään kahden molekyylin välillä. Reaktiot siis tapahtuvat sattumalta, emmekä voi tarkasti tietää mitä reaktiota kulloinkin tapahtuu vai tapahtuuko useita.

        "Nollakin on vakio"

        Nolla ei voi olla vakio, kun puhutaan energian tai minkä tahansa asian olevasta määrästä. Energiaa joko on, tai sitä ei ole. Jos sitä ei olisi, niin silloin sen määrä olisi nolla. Kuitenkin voimme osoittaa jo vaikkapa käyttämällä tätä tietokonetta, että energiaa on, joten sen määrä ei voi olla nolla. Mikäli siis energiaa ei olisi ja sen määrä olisi nolla, ei olisi mitään muutakaan.

        Evoluutiosta halusin tuoda esille sen, että sitä ei voida mitata. Se ei ole siis tiedettä, kuten itsekin mainitsit. Jos joku ei ole tiedettä, se on jotain muuta, mikä ei perustu tarkkoihin mittauksiin ja todisteisiin. Se on siis metafysiikkaa, eli fysiikan taakse sijoittuvaa "tietoa", josta ei voida saada lopullista varmuutta. On eri asia havainnoida mennyttä kuin mitata tapahtuvaa. Sama ongelma on esim. filosofialla ja historiatieteillä. Olemme näillä alueilla loppujen lopuksi pakottautuneita nojaamaan teorioihin, joita ei voida mitata. Se tekee siis kaikesta tästä metafysiikkaa ja siihen sisältyy myös evoluutio, halusit tai et. Tämä on siis jyrkän tieteellinen näkökulma asiaan. On toki olemassa erilaisia näkökulmia asiaan liittyen, kuten esimerkiksi mannermaiset filosofit esittivät, mutta se ei silti poista mittaamisen ongelmaa. Keskustelu jatkuu tämän asian osalta koko ajan, eikä vieläkään ole voitu esittää mitään varmaa tieteenfilosofisiin ongelmiin. Se tekee tästä myös toisaalta mielenkiintoista ja paneutumisen arvoista. :)

        Ja mitä tulee valehtelu-kysymykseen, niin "Jumala on rakastanut maailmaa niin paljon, että antoi ainoan poikansa, ettei yksikään, joka häneen uskoo joutuisi kadotukseen, vaan saisi iankaikkisen elämän." (Joh: 3:16)

        Mitä tämä siis tarkoittaa? Raamattu ilmoittaa Jumalan olevan Pyhä. Jumala ei siis siedä minkäänlaista syntiä. Ristinkuolemallaan Jeesus kuitenkin sovitti kaikkien ihmisten synnit. Näin ollen meille annetaan mahdollisuus päästä Jumalan luo turvautumalla Jeesuksen sovitustyöhön. Ei siis perustuen siihen mitä me teemme, tai emme tee, vaan siihen mitä Jeesus teki ristillä meidän puolestamme. Ihminen pelastuu yksin uskosta, yksin armosta, yksin Jeesuksen ristin tähden. Siihen ei vaikuta pätkän vertaa se pidämmekö evoluutioteoriaa oikeana vai vääränä. Kyllä se meille sitten aikanaan selviää. :)

        Eikö olekin hienoa, että puhun jonkun mielestä aikamoisella kontrastilla tiedettä ja uskoa. Itse ainakin hämmästyn jatkuvasti. Voisin jopa pitää itseäni melkoisena sekopäänä, jos en tietäisi mistä puhun. Kuitenkaan en näe näillä asioilla ristiriitaa, koska minulla ei ole tarvetta olla maailman synnyn suhteen oikeassa. Se on tapahtunut niin kuin on tapahtunut (minun mielestäni luomistyön kautta ja jonkun toisen mielestä jotenkin muuten) eikä se muuta suhdettani Jumalaan millään lailla. :)

        Kaikkea hyvää, Mikko Rantalahti (mikko.rantalahti[at]gmail.com)


      • Quicunque kirjoitti:

        Hawkingista: Teksti jota halusit minun tutkivan sisältää käsitteet negatiivisesta ja positiivisesta aineesta, jotka kumoavat toisensa. Siinä puhutaan siis matemaattisesti positiivisesta ja negatiivisesta. Aine ja vasta-aine kumoavat toisensa ja muodostuu nolla. Kuitenkin kyseisessä teoriassa on se ongelma, että se ylipäänsä sisältää termit aine ja vasta-aine. Jotain siis on, vaikka ne muodostaisivatkin yhteensä nollan (mittavirheitä lukuunottamatta, kuten tekstistäkin ilmeni). Jos jotain siis on (aine ja vasta-aine), ja vaikka ne muodostaisivatkin yhteensä matemaattisesti nollan, niin siitä ei voida vetää johtopäätöstä, että jotain olisi syntynyt tyhjästä, koska aine ja vasta-aine ovat olleet olemassa. Tyhjä on eri asia kuin nolla. Hawkingin mukaan nollasta voi syntyä jotain(, mikä on ainakin meikäläisen hankala käsittää, paitsi jos puhutaan matemaattisesta nollasta, jossa vastavoimat kumoavat toisensa), mutta tyhjästä ei edelleenkään voi syntyä mitään. (Kosmologia on kyllä mielenkiintoista ja siinä ollaan oikean asian äärellä. Siinä on vaan ylipäänsä se ongelma, että se on jatkuvaa teorioiden päivitystä ja käsitteiden muokkaamista, kun huomataan, että taas on menty vikaan. Mielestäni on kuitenkin hienoa, että jotkut osaavat tutkia asioita kuudensissa tai jopa seitsemänsissä ja suuremmissa ulottuvuuksissa ja vetää siitä johtopäätöksiä vallitsevaan maailmaan. Itse en osaisi. Tosin en tiedä kuinka relevanttia tietoa eri ulottuvuuksia tutkimalla voidaan saada näihin kolmeen vallitsevaan ulottuvuuteen, joissa elämme, mutta ehkäpä he (tutkijat) tietävät.) :)

        "Molekyylitasolla? Tuota en tiennytkään. Onko sinulla jotain esimerkkejä teoriassa sattumalta tapahtuvista molekyylitason tapahtumista?"

        Heisenbergin epävarmuusperiaatteen mukaisesti emme voi tietää tarkasti kappaleiden sijaintia emmekä liikettä. Tästä Schrödinger kehitteli mallin joka englanniksi kulkee nimellä "The particle in a Box". Sen mukaan (rajusti yksinkertaistaen) rajaamme alueen, jonka sisällä reaktiot tapahtuvat ja laskemme todennäköisyyksiä reaktioiden tapahtumille. Jotkut reaktiot tapahtuvat todennäköisemmin kuin toiset, mutta reaktioita voi olla silti useita jo pelkästään kahden molekyylin välillä. Reaktiot siis tapahtuvat sattumalta, emmekä voi tarkasti tietää mitä reaktiota kulloinkin tapahtuu vai tapahtuuko useita.

        Tavallinen uskomus edelleenkin lienee, että "tyhjässä ei ole mitään". Vanha väite, jonka mukaan "tyhjästä ei voi nyhjäistä mitään" on todellakin vanhentunut, sillä fysiikka on saanut selville paljon tietoa tyhjiöstä (vakuumi), jonka ymmärtäminen on voimakkaasti kehittynyt 1900-luvulla. Olemattomuutta ei ole, vaan tyhjiössäkin pulputtaa hiukkasien kattila. Kvanttiteorioista lähtien on ollut tiedossa, että kvanttifysikaalinen tyhjiö ei ole tyhjä, vaan siinä on automaattisesti tyhjiöenergiaa - se suorastaan kuplii hiukkasia. Kyseessä ovat todelliset ja havaitut hiukkaset eikä mikään teoreettinen spekulaatio: elektroni-positroni -pari voi tulla esiin tyhjästä ja elää Heisenbergin epätarkkuusperiaatteen mukaisesti kunnes se taas häviää kvanttimekaanisten todennäköisyyksien syövereihin. Kvanttifysikaalinen tyhjiön hiukkaskuplinta näkyy atomien energiatasoissa jälkinä, jotka ovat täysin havaittavia. Ainoa ongelma oli laskea tyhjiöenergian määrä, ei suinkaan sen olemassaolon sinänsä osoittaminen. Lisäksi pimeän energian määrä tilavuusyksikköä kohden ei muutu, ja tällä tavalla kosmologinen vakio on määritelty peräti.

        Epätarkkuusperiaatteen mukaan materiaa voi tulla tyhjästä jos "energian lainaus" maksetaan kuitenkin takaisin epämääräisyysajan puitteissa. Koko universumi on siis voinut syntyä tyhjästä, massaenergian kuplasta, jonka painovoimakentällä on ollut yhtä suuri määrä vastakkaista energiaa. Siten universumin kokonaisenergian määrä on nolla ja inflaatio on laajentanut sen pienen kuplan nykyään näkyväksi maailmankaikkeudeksi. Tämä teoria on täysin linjassa fysiikan kanssa.

        Jo 1980-luvulla kosmologiassa päästiin selvyyteen kosmisen energian lähteestä, josta aine muodostui (energiasta muodostuu ainetta jos energia on tarpeeksi keskittynyttä): maailmankaikkeuden kokonaisenergia on nolla eli kyseessä on "tyhjästä tyhjää-luonteinen tapahtuma" (Davies). Maailmankaikkeuden sisältämän aineen määrä on 10^50 tonnia vaikka universumin kokonaisenergian määrä on nolla, sillä sen gravitaatiokentällä on negatiivinen energia. Kosmologiassa pystyttiin selvittämään, kuinka positiivinen energia muuttui aineeksi ja yhtä suuri määrä negatiivista energiaa meni gravitaatiokenttään, joten kaikki kosminen aine syntyi ilmaiseksi. Tämän löydön jälkeen oli uskottava oletus, että maailmankaikkeus syntyi tyhjästä avaruudesta ja aine syntyi melko nopeasti sen jälkeen luonnollisten fysikaalisten prosessien avulla. Siten leikkaantui pois tarve oletukselle ajan alussa tapahtuneesta yliluonnollisesta aineen syöttämisestä (Paul Davies, Viides ihme s.50-51).

        Kvanttifyysikko Pascual Jordan laski, että jos mitä tahansa ainetta olevan kappaleen kaikki massa on keskittynyt yhteen pisteeseen, on sen vetovoimakentän negatiivisen energian oltava -mc2, joka mitätöi kappaleen - kuten tähden - positiivisen massaenergian. Minkä tahansa massaisen kappaleen gravitaatioenergia on negatiivinen ja mitä tiiviimpi kappale on, sitä negatiivisempi sen gravitaatioenergia on. Pascual Jordanin ajatuksiin liittyen fyysikko Georgy Gamow (k. 1968) esitti, että tähti saattoi syntyä tyhjästä (Gamow kertoi Einsteinin pysähtyneen paikoilleen keskellä katua kuultuaan tämän).

        Tässä ei ole kyse vain analogiasta tai ihmisen tavasta mitata energiaa. Kyseessä on merkittävä tosiasia maailmankaikkeuden toiminnasta: vetovoimakentillä on negatiivinen energia, joka kumoaa yhteen pisteeseen tiivistyneenä täsmällisesti ko. aineen massaenergian. Ja juuri näistä Pascualin ja Gamowin esittämistä ajatuksista muodostui kulmakivi teorialle, jonka mukaan koko maailmankaikkeus on voinut syntyä tyhjästä, siis massaenergian kuplasta, jonka painovoimakentällä on ollut yhtä suuri määrä vastakkaista energiaa. Täten energian kokonaismäärä maailmankaikkeudessa on nolla ja inflaatio on se prosessi, joka kasvatti kuplasta havaitsemamme maailmankaikkeuden (kvanttifysiikan mukaan tyhjässä voi aivan helposti olla jotakin, vaikka tavalliselle arkiajattelulle asia tuntuu oudolta).


      • Myöskään...
        Quicunque kirjoitti:

        "Nollakin on vakio"

        Nolla ei voi olla vakio, kun puhutaan energian tai minkä tahansa asian olevasta määrästä. Energiaa joko on, tai sitä ei ole. Jos sitä ei olisi, niin silloin sen määrä olisi nolla. Kuitenkin voimme osoittaa jo vaikkapa käyttämällä tätä tietokonetta, että energiaa on, joten sen määrä ei voi olla nolla. Mikäli siis energiaa ei olisi ja sen määrä olisi nolla, ei olisi mitään muutakaan.

        Evoluutiosta halusin tuoda esille sen, että sitä ei voida mitata. Se ei ole siis tiedettä, kuten itsekin mainitsit. Jos joku ei ole tiedettä, se on jotain muuta, mikä ei perustu tarkkoihin mittauksiin ja todisteisiin. Se on siis metafysiikkaa, eli fysiikan taakse sijoittuvaa "tietoa", josta ei voida saada lopullista varmuutta. On eri asia havainnoida mennyttä kuin mitata tapahtuvaa. Sama ongelma on esim. filosofialla ja historiatieteillä. Olemme näillä alueilla loppujen lopuksi pakottautuneita nojaamaan teorioihin, joita ei voida mitata. Se tekee siis kaikesta tästä metafysiikkaa ja siihen sisältyy myös evoluutio, halusit tai et. Tämä on siis jyrkän tieteellinen näkökulma asiaan. On toki olemassa erilaisia näkökulmia asiaan liittyen, kuten esimerkiksi mannermaiset filosofit esittivät, mutta se ei silti poista mittaamisen ongelmaa. Keskustelu jatkuu tämän asian osalta koko ajan, eikä vieläkään ole voitu esittää mitään varmaa tieteenfilosofisiin ongelmiin. Se tekee tästä myös toisaalta mielenkiintoista ja paneutumisen arvoista. :)

        Ja mitä tulee valehtelu-kysymykseen, niin "Jumala on rakastanut maailmaa niin paljon, että antoi ainoan poikansa, ettei yksikään, joka häneen uskoo joutuisi kadotukseen, vaan saisi iankaikkisen elämän." (Joh: 3:16)

        Mitä tämä siis tarkoittaa? Raamattu ilmoittaa Jumalan olevan Pyhä. Jumala ei siis siedä minkäänlaista syntiä. Ristinkuolemallaan Jeesus kuitenkin sovitti kaikkien ihmisten synnit. Näin ollen meille annetaan mahdollisuus päästä Jumalan luo turvautumalla Jeesuksen sovitustyöhön. Ei siis perustuen siihen mitä me teemme, tai emme tee, vaan siihen mitä Jeesus teki ristillä meidän puolestamme. Ihminen pelastuu yksin uskosta, yksin armosta, yksin Jeesuksen ristin tähden. Siihen ei vaikuta pätkän vertaa se pidämmekö evoluutioteoriaa oikeana vai vääränä. Kyllä se meille sitten aikanaan selviää. :)

        Eikö olekin hienoa, että puhun jonkun mielestä aikamoisella kontrastilla tiedettä ja uskoa. Itse ainakin hämmästyn jatkuvasti. Voisin jopa pitää itseäni melkoisena sekopäänä, jos en tietäisi mistä puhun. Kuitenkaan en näe näillä asioilla ristiriitaa, koska minulla ei ole tarvetta olla maailman synnyn suhteen oikeassa. Se on tapahtunut niin kuin on tapahtunut (minun mielestäni luomistyön kautta ja jonkun toisen mielestä jotenkin muuten) eikä se muuta suhdettani Jumalaan millään lailla. :)

        Kaikkea hyvää, Mikko Rantalahti (mikko.rantalahti[at]gmail.com)

        teologia ei ole tiedettä.


      • Quicunque
        Aatami9 kirjoitti:

        Tavallinen uskomus edelleenkin lienee, että "tyhjässä ei ole mitään". Vanha väite, jonka mukaan "tyhjästä ei voi nyhjäistä mitään" on todellakin vanhentunut, sillä fysiikka on saanut selville paljon tietoa tyhjiöstä (vakuumi), jonka ymmärtäminen on voimakkaasti kehittynyt 1900-luvulla. Olemattomuutta ei ole, vaan tyhjiössäkin pulputtaa hiukkasien kattila. Kvanttiteorioista lähtien on ollut tiedossa, että kvanttifysikaalinen tyhjiö ei ole tyhjä, vaan siinä on automaattisesti tyhjiöenergiaa - se suorastaan kuplii hiukkasia. Kyseessä ovat todelliset ja havaitut hiukkaset eikä mikään teoreettinen spekulaatio: elektroni-positroni -pari voi tulla esiin tyhjästä ja elää Heisenbergin epätarkkuusperiaatteen mukaisesti kunnes se taas häviää kvanttimekaanisten todennäköisyyksien syövereihin. Kvanttifysikaalinen tyhjiön hiukkaskuplinta näkyy atomien energiatasoissa jälkinä, jotka ovat täysin havaittavia. Ainoa ongelma oli laskea tyhjiöenergian määrä, ei suinkaan sen olemassaolon sinänsä osoittaminen. Lisäksi pimeän energian määrä tilavuusyksikköä kohden ei muutu, ja tällä tavalla kosmologinen vakio on määritelty peräti.

        Epätarkkuusperiaatteen mukaan materiaa voi tulla tyhjästä jos "energian lainaus" maksetaan kuitenkin takaisin epämääräisyysajan puitteissa. Koko universumi on siis voinut syntyä tyhjästä, massaenergian kuplasta, jonka painovoimakentällä on ollut yhtä suuri määrä vastakkaista energiaa. Siten universumin kokonaisenergian määrä on nolla ja inflaatio on laajentanut sen pienen kuplan nykyään näkyväksi maailmankaikkeudeksi. Tämä teoria on täysin linjassa fysiikan kanssa.

        Jo 1980-luvulla kosmologiassa päästiin selvyyteen kosmisen energian lähteestä, josta aine muodostui (energiasta muodostuu ainetta jos energia on tarpeeksi keskittynyttä): maailmankaikkeuden kokonaisenergia on nolla eli kyseessä on "tyhjästä tyhjää-luonteinen tapahtuma" (Davies). Maailmankaikkeuden sisältämän aineen määrä on 10^50 tonnia vaikka universumin kokonaisenergian määrä on nolla, sillä sen gravitaatiokentällä on negatiivinen energia. Kosmologiassa pystyttiin selvittämään, kuinka positiivinen energia muuttui aineeksi ja yhtä suuri määrä negatiivista energiaa meni gravitaatiokenttään, joten kaikki kosminen aine syntyi ilmaiseksi. Tämän löydön jälkeen oli uskottava oletus, että maailmankaikkeus syntyi tyhjästä avaruudesta ja aine syntyi melko nopeasti sen jälkeen luonnollisten fysikaalisten prosessien avulla. Siten leikkaantui pois tarve oletukselle ajan alussa tapahtuneesta yliluonnollisesta aineen syöttämisestä (Paul Davies, Viides ihme s.50-51).

        Kvanttifyysikko Pascual Jordan laski, että jos mitä tahansa ainetta olevan kappaleen kaikki massa on keskittynyt yhteen pisteeseen, on sen vetovoimakentän negatiivisen energian oltava -mc2, joka mitätöi kappaleen - kuten tähden - positiivisen massaenergian. Minkä tahansa massaisen kappaleen gravitaatioenergia on negatiivinen ja mitä tiiviimpi kappale on, sitä negatiivisempi sen gravitaatioenergia on. Pascual Jordanin ajatuksiin liittyen fyysikko Georgy Gamow (k. 1968) esitti, että tähti saattoi syntyä tyhjästä (Gamow kertoi Einsteinin pysähtyneen paikoilleen keskellä katua kuultuaan tämän).

        Tässä ei ole kyse vain analogiasta tai ihmisen tavasta mitata energiaa. Kyseessä on merkittävä tosiasia maailmankaikkeuden toiminnasta: vetovoimakentillä on negatiivinen energia, joka kumoaa yhteen pisteeseen tiivistyneenä täsmällisesti ko. aineen massaenergian. Ja juuri näistä Pascualin ja Gamowin esittämistä ajatuksista muodostui kulmakivi teorialle, jonka mukaan koko maailmankaikkeus on voinut syntyä tyhjästä, siis massaenergian kuplasta, jonka painovoimakentällä on ollut yhtä suuri määrä vastakkaista energiaa. Täten energian kokonaismäärä maailmankaikkeudessa on nolla ja inflaatio on se prosessi, joka kasvatti kuplasta havaitsemamme maailmankaikkeuden (kvanttifysiikan mukaan tyhjässä voi aivan helposti olla jotakin, vaikka tavalliselle arkiajattelulle asia tuntuu oudolta).

        Lyhyt vastaus. Nyt ei ehdi muuta. :)

        Kiitos tekstistä. Aina oppii uutta.

        Jos siis on niin, kuin kerroit, että tyhjiö ei ole tyhjä, niin silloinhan siellä on jotain eikä se itse asiassa ole tyhjä, vaikka onkin! :) Siistiä... Tyhjiöenergian (jota siis ilmeisesti on tyhjössä) määrä on siis nolla? Tästähän puhuin jo aiemmin. Se on siis matemaattisesti nolla, mikä taas aiheuttaa sen, että jotain on (aine ja vasta-aine). Tästä taas tullaan siihen, että jos kerran tyhjiö ei ole tyhjä, niin jotain on.

        Yhtä kaikki, mielenkiintoista se silti on ja kiitoksia vastauksesta. Ei ihme, että Einsteinkin seisahtui. Tämä ei kuitenkaan poista mitään. Se tuo mielenkiintoisella tavalla lisää. Lopulta päädymme kuitenkin aina samaan kysymykseen, aikaan ennen tyhjiöenergian syntymistä. Tiedän, että aika on solutettu yhteen kaiken muun kanssa, mutta en nyt ehdi keksiä ajalle uutta suomenkielen termiä, vaan sille on nyt annettava kaksoismerkitys. :)

        Palataan paremmalla ajalla vielä asiaan...


      • Palataan mielummin..
        Quicunque kirjoitti:

        Lyhyt vastaus. Nyt ei ehdi muuta. :)

        Kiitos tekstistä. Aina oppii uutta.

        Jos siis on niin, kuin kerroit, että tyhjiö ei ole tyhjä, niin silloinhan siellä on jotain eikä se itse asiassa ole tyhjä, vaikka onkin! :) Siistiä... Tyhjiöenergian (jota siis ilmeisesti on tyhjössä) määrä on siis nolla? Tästähän puhuin jo aiemmin. Se on siis matemaattisesti nolla, mikä taas aiheuttaa sen, että jotain on (aine ja vasta-aine). Tästä taas tullaan siihen, että jos kerran tyhjiö ei ole tyhjä, niin jotain on.

        Yhtä kaikki, mielenkiintoista se silti on ja kiitoksia vastauksesta. Ei ihme, että Einsteinkin seisahtui. Tämä ei kuitenkaan poista mitään. Se tuo mielenkiintoisella tavalla lisää. Lopulta päädymme kuitenkin aina samaan kysymykseen, aikaan ennen tyhjiöenergian syntymistä. Tiedän, että aika on solutettu yhteen kaiken muun kanssa, mutta en nyt ehdi keksiä ajalle uutta suomenkielen termiä, vaan sille on nyt annettava kaksoismerkitys. :)

        Palataan paremmalla ajalla vielä asiaan...

        aikaan ennen jumalan syntymistä. Eli koska se jumala päätti synnyttää itsensä? Kuten huomaat, niin ei sieltä uskonnosta löydy yhtään parempaa vastausta... (jumala on aina ollut olemassa vs. tyhjiöenergia on aina ollut olemassa)


      • Quicunque kirjoitti:

        "Nollakin on vakio"

        Nolla ei voi olla vakio, kun puhutaan energian tai minkä tahansa asian olevasta määrästä. Energiaa joko on, tai sitä ei ole. Jos sitä ei olisi, niin silloin sen määrä olisi nolla. Kuitenkin voimme osoittaa jo vaikkapa käyttämällä tätä tietokonetta, että energiaa on, joten sen määrä ei voi olla nolla. Mikäli siis energiaa ei olisi ja sen määrä olisi nolla, ei olisi mitään muutakaan.

        Evoluutiosta halusin tuoda esille sen, että sitä ei voida mitata. Se ei ole siis tiedettä, kuten itsekin mainitsit. Jos joku ei ole tiedettä, se on jotain muuta, mikä ei perustu tarkkoihin mittauksiin ja todisteisiin. Se on siis metafysiikkaa, eli fysiikan taakse sijoittuvaa "tietoa", josta ei voida saada lopullista varmuutta. On eri asia havainnoida mennyttä kuin mitata tapahtuvaa. Sama ongelma on esim. filosofialla ja historiatieteillä. Olemme näillä alueilla loppujen lopuksi pakottautuneita nojaamaan teorioihin, joita ei voida mitata. Se tekee siis kaikesta tästä metafysiikkaa ja siihen sisältyy myös evoluutio, halusit tai et. Tämä on siis jyrkän tieteellinen näkökulma asiaan. On toki olemassa erilaisia näkökulmia asiaan liittyen, kuten esimerkiksi mannermaiset filosofit esittivät, mutta se ei silti poista mittaamisen ongelmaa. Keskustelu jatkuu tämän asian osalta koko ajan, eikä vieläkään ole voitu esittää mitään varmaa tieteenfilosofisiin ongelmiin. Se tekee tästä myös toisaalta mielenkiintoista ja paneutumisen arvoista. :)

        Ja mitä tulee valehtelu-kysymykseen, niin "Jumala on rakastanut maailmaa niin paljon, että antoi ainoan poikansa, ettei yksikään, joka häneen uskoo joutuisi kadotukseen, vaan saisi iankaikkisen elämän." (Joh: 3:16)

        Mitä tämä siis tarkoittaa? Raamattu ilmoittaa Jumalan olevan Pyhä. Jumala ei siis siedä minkäänlaista syntiä. Ristinkuolemallaan Jeesus kuitenkin sovitti kaikkien ihmisten synnit. Näin ollen meille annetaan mahdollisuus päästä Jumalan luo turvautumalla Jeesuksen sovitustyöhön. Ei siis perustuen siihen mitä me teemme, tai emme tee, vaan siihen mitä Jeesus teki ristillä meidän puolestamme. Ihminen pelastuu yksin uskosta, yksin armosta, yksin Jeesuksen ristin tähden. Siihen ei vaikuta pätkän vertaa se pidämmekö evoluutioteoriaa oikeana vai vääränä. Kyllä se meille sitten aikanaan selviää. :)

        Eikö olekin hienoa, että puhun jonkun mielestä aikamoisella kontrastilla tiedettä ja uskoa. Itse ainakin hämmästyn jatkuvasti. Voisin jopa pitää itseäni melkoisena sekopäänä, jos en tietäisi mistä puhun. Kuitenkaan en näe näillä asioilla ristiriitaa, koska minulla ei ole tarvetta olla maailman synnyn suhteen oikeassa. Se on tapahtunut niin kuin on tapahtunut (minun mielestäni luomistyön kautta ja jonkun toisen mielestä jotenkin muuten) eikä se muuta suhdettani Jumalaan millään lailla. :)

        Kaikkea hyvää, Mikko Rantalahti (mikko.rantalahti[at]gmail.com)

        "Onko sinulla jotain esimerkkejä teoriassa sattumalta tapahtuvista molekyylitason tapahtumista?"
        "Heisenbergin epävarmuusperiaatteen mukaisesti emme voi tietää tarkasti kappaleiden sijaintia emmekä liikettä…"

        Tiedän Heisenbergin epävarmuusperiaatteen kyllä, mutta olin luullut että se koskee atomitason tapahtumia.

        "Nolla ei voi olla vakio, kun puhutaan energian tai minkä tahansa asian olevasta määrästä."

        Tuossa toisessa viestissä jo tulikin esiin, että maailmankaikkeuden energian kokonaismäärä saattaa hyvinkin olla nolla.

        "Evoluutiosta halusin tuoda esille sen, että sitä ei voida mitata. Se ei ole siis tiedettä, kuten itsekin mainitsit."

        Sanoin, että evoluutio ei ole tiedettä, koska evoluutio on luonnossa esiintyvä ilmiö. Ilmiö ei ole tiedettä. Tätä kyseistä ilmiötä tutkii biologia, joka on yksi tieteenala.

        "Se tekee siis kaikesta tästä metafysiikkaa ja siihen sisältyy myös evoluutio, halusit tai et. Tämä on siis jyrkän tieteellinen näkökulma asiaan."

        Jos näkökulmasi tosiaan on tieteellinen, niinkuin sanoit, niin sinusta tulee kuuluisa. Silloinhan voit yhdellä pienellä tieteellisellä artikkelilla kumota kaiken tähän asti evoluutiosta tehdyn tutkimuksen. Suosittelen ehdottomasti lähettämään teoriasi vaikkapa Natureen. Samalla muutama muukin tieteentekijä voisi kanssasi arvioida onko näkökulmasi aivan varmasti "jyrkän tieteellinen". Minä en liene siihen riittävän oppinut, sillä minä en millään ymmärrä mikä tekee biologiasta metafysiikkaa.

        Millaisia todisteita evoluutiosta noin niinkuin täsmälleen ottaen vielä haluaisit?

        "Eikö olekin hienoa, että puhun jonkun mielestä aikamoisella kontrastilla tiedettä ja uskoa. Itse ainakin hämmästyn jatkuvasti. Voisin jopa pitää itseäni melkoisena sekopäänä, jos en tietäisi mistä puhun."

        Olen samaa mieltä, paitsi että minä EN tiedä mistä sinä puhut.

        "Kuitenkaan en näe näillä asioilla ristiriitaa, koska minulla ei ole tarvetta olla maailman synnyn suhteen oikeassa."

        Minä taas näen niiden välillä ristiriidan, koska minulla ei ole tarvetta pitää väkisin jotakin jumalolentoa kaiken alkusyynä.

        Taidan olla vähän yksinkertainen kun en ymmärrä yhtä asiaa: Jos emme koskaan voi saada selville millä tavoin maailma on syntynyt, niin miksi emme voi vain sanoa ettemme tiedä? Miksi pitäisi alkaa uskomaan ja väittämään ilman mitään perusteita, että maailman on luonut joku tonttu tai muu myyttinen olento?


      • Quicunque kirjoitti:

        Lyhyt vastaus. Nyt ei ehdi muuta. :)

        Kiitos tekstistä. Aina oppii uutta.

        Jos siis on niin, kuin kerroit, että tyhjiö ei ole tyhjä, niin silloinhan siellä on jotain eikä se itse asiassa ole tyhjä, vaikka onkin! :) Siistiä... Tyhjiöenergian (jota siis ilmeisesti on tyhjössä) määrä on siis nolla? Tästähän puhuin jo aiemmin. Se on siis matemaattisesti nolla, mikä taas aiheuttaa sen, että jotain on (aine ja vasta-aine). Tästä taas tullaan siihen, että jos kerran tyhjiö ei ole tyhjä, niin jotain on.

        Yhtä kaikki, mielenkiintoista se silti on ja kiitoksia vastauksesta. Ei ihme, että Einsteinkin seisahtui. Tämä ei kuitenkaan poista mitään. Se tuo mielenkiintoisella tavalla lisää. Lopulta päädymme kuitenkin aina samaan kysymykseen, aikaan ennen tyhjiöenergian syntymistä. Tiedän, että aika on solutettu yhteen kaiken muun kanssa, mutta en nyt ehdi keksiä ajalle uutta suomenkielen termiä, vaan sille on nyt annettava kaksoismerkitys. :)

        Palataan paremmalla ajalla vielä asiaan...

        Ootko sä yhtä tyhjän kanssa Kemian DI:nä ja filosofian opettajana kuin Päivi Räsänen on lääkärinä ja kansanedustajana?

        Häneltä kysyttiin "Homoillassa". Mitä tekisit, mikäli lapsesi ilmoittaisi jonakin päivänä olevansa homo? Hän alkaisi ilmeisesti "eheyttämään" lastansa heteroksi!?

        Uskovaisilla on tunnetila ja homoseksuaaleilla on tahtotila. Yleensä tahtotila on vaan voimakkaampi kuin tunnetila!

        Ihmisarvoa viedään romukoppaan eli tällöinhän kirkko rikkoo pääsanomaansa, joka on "lähimmäisen rakkaus" eli nyt sitten halutaan "eheyttää" lähimmäisiä heteroksi!

        On se vaan kumma kun konservatiiviset ihmiset haluavat työntää nenäänsä muiden ihmisten "tahtotilaan".

        Minusta homoilla tulee olla samat oikeudet mennä avioliittoon kuin heteroillakin, koska homoilla on mahdollisuus rekisteröityyn parisuhteeseen, joka taas rikkoo heidän ihmisarvoja esim. hakiessa työtä eli ei välttämättä tule aina valituksi työsuhteeseen kun ovat ruksittaneet tai maininneet "rekisteröidyn parisuhteen"!!

        Itse olen mies ja olen elänyt n. 10 vuotta avoliitossa naisen kanssa, mutta meitä ei VIHITÄ avioliittoon vaan minusta tulisi olla sille muita uusia vaihtoehtoja eli esim. HAKEMINEN avioliittoon. Ns. "Avioliitto hakemuksen" täyttää nykyisinkin joko pappi tai maistraatin toimitsija puolisoiden puolesta, miksei siis PUOLISOT!?


      • Trollinaattori
        Quicunque kirjoitti:

        "Nollakin on vakio"

        Nolla ei voi olla vakio, kun puhutaan energian tai minkä tahansa asian olevasta määrästä. Energiaa joko on, tai sitä ei ole. Jos sitä ei olisi, niin silloin sen määrä olisi nolla. Kuitenkin voimme osoittaa jo vaikkapa käyttämällä tätä tietokonetta, että energiaa on, joten sen määrä ei voi olla nolla. Mikäli siis energiaa ei olisi ja sen määrä olisi nolla, ei olisi mitään muutakaan.

        Evoluutiosta halusin tuoda esille sen, että sitä ei voida mitata. Se ei ole siis tiedettä, kuten itsekin mainitsit. Jos joku ei ole tiedettä, se on jotain muuta, mikä ei perustu tarkkoihin mittauksiin ja todisteisiin. Se on siis metafysiikkaa, eli fysiikan taakse sijoittuvaa "tietoa", josta ei voida saada lopullista varmuutta. On eri asia havainnoida mennyttä kuin mitata tapahtuvaa. Sama ongelma on esim. filosofialla ja historiatieteillä. Olemme näillä alueilla loppujen lopuksi pakottautuneita nojaamaan teorioihin, joita ei voida mitata. Se tekee siis kaikesta tästä metafysiikkaa ja siihen sisältyy myös evoluutio, halusit tai et. Tämä on siis jyrkän tieteellinen näkökulma asiaan. On toki olemassa erilaisia näkökulmia asiaan liittyen, kuten esimerkiksi mannermaiset filosofit esittivät, mutta se ei silti poista mittaamisen ongelmaa. Keskustelu jatkuu tämän asian osalta koko ajan, eikä vieläkään ole voitu esittää mitään varmaa tieteenfilosofisiin ongelmiin. Se tekee tästä myös toisaalta mielenkiintoista ja paneutumisen arvoista. :)

        Ja mitä tulee valehtelu-kysymykseen, niin "Jumala on rakastanut maailmaa niin paljon, että antoi ainoan poikansa, ettei yksikään, joka häneen uskoo joutuisi kadotukseen, vaan saisi iankaikkisen elämän." (Joh: 3:16)

        Mitä tämä siis tarkoittaa? Raamattu ilmoittaa Jumalan olevan Pyhä. Jumala ei siis siedä minkäänlaista syntiä. Ristinkuolemallaan Jeesus kuitenkin sovitti kaikkien ihmisten synnit. Näin ollen meille annetaan mahdollisuus päästä Jumalan luo turvautumalla Jeesuksen sovitustyöhön. Ei siis perustuen siihen mitä me teemme, tai emme tee, vaan siihen mitä Jeesus teki ristillä meidän puolestamme. Ihminen pelastuu yksin uskosta, yksin armosta, yksin Jeesuksen ristin tähden. Siihen ei vaikuta pätkän vertaa se pidämmekö evoluutioteoriaa oikeana vai vääränä. Kyllä se meille sitten aikanaan selviää. :)

        Eikö olekin hienoa, että puhun jonkun mielestä aikamoisella kontrastilla tiedettä ja uskoa. Itse ainakin hämmästyn jatkuvasti. Voisin jopa pitää itseäni melkoisena sekopäänä, jos en tietäisi mistä puhun. Kuitenkaan en näe näillä asioilla ristiriitaa, koska minulla ei ole tarvetta olla maailman synnyn suhteen oikeassa. Se on tapahtunut niin kuin on tapahtunut (minun mielestäni luomistyön kautta ja jonkun toisen mielestä jotenkin muuten) eikä se muuta suhdettani Jumalaan millään lailla. :)

        Kaikkea hyvää, Mikko Rantalahti (mikko.rantalahti[at]gmail.com)

        "Kuitenkaan en näe näillä asioilla ristiriitaa, koska minulla ei ole tarvetta olla maailman synnyn suhteen oikeassa. "

        ...mutta kuitenkin sinulla tuntuu olevan tarve olla oikeassa siinä, että maailma on syntynyt. Vaikka kovasti vakuutat menneen tutkimisen olevan metafysiikkaa.

        "Voisin jopa pitää itseäni melkoisena sekopäänä, jos en tietäisi mistä puhun. "

        " Energiaa joko on, tai sitä ei ole. Jos sitä ei olisi, niin silloin sen määrä olisi nolla. "

        Pidä itseäsi melkoisena sekopäänä. Ehtolausekkeesi ei tosin kuulunut että:voisin pitää itseäni sekopäänä, jos tietäisin mistä puhun.

        Mistäköhän energiasta sinä puhut? Energia muuttaa muotoaan. Kun jossain on "negatiivinen" on toisaalla "positiivinen". Yhdessä nämä muodostavat nollan. Et siis tiedä mistä puhut. Voithan toki hakkailla olkiukkoasi yhä uudelelleen ja uudelleen, kanssauskovaisten lyöden tahtiä innoissansa karvaisilla käpälillään.


      • Rompix
        Quicunque kirjoitti:

        Siunausta sinulle ateisti1982. Olet oikean asian äärellä jatkuvan kyselemisesi kanssa. Toivottavasti löydät totuuden, niin sinun ei tarvitse olla koko ajan vihainen. Sanon tämän ystävänä.

        Olen koulutukseltani kemian DI ja filosofian opettaja ja hyvin paljon käsitellyt tieteen ja uskon välistä suhdetta. En ole koskaan kokenut ristiriitaa omissa opinnoissani tai opetuksessani tieteen ja uskon välillä. Sen sijaan, kun mennään tarpeeksi syvälle molemmissa, niin huomataan monia yhtäläisyyksiä. Molekyylitasolla asiat tapahtuvat teoriassa sattumalta, mutta se johtuu vain siitä, ettemme osaa kutsua sitä muuksi kuin sattumaksi. Emme pysty määrittelemään molekyylien liikkeitä, vaikka ne tapahtuvat silmiemme edessä. Kuinka pystyisimme siis määrittelemään maailmankaikkeuden liikkeitä aikojen alussa?

        Energian määrä maailmankaikkeudessa on vakio. Energiaa ei voi syntyä tyhjästä, se vain muuttaa muotoaan. Ei ole siis mahdollista, että maailmankaikkeus olisi voinut syntyä tyhjästä. Jotain on siis täytynyt olla aina ja tulee aina olemaan. On turha kuvitella, että olisimme niin älykkäitä, että pystyisimme koskaan ratkaisemaan näitä arvoituksia. Asian edessä on vain nöyrryttävä ja tunnustettava oma vajavaisuutensa.

        Ja mitä evoluutioon tulee, niin on aivan samantekevää kuinka kehitys tapahtuu. Me emme ole edes niin älykkäitä, että osaisimme selittää puuttuvien lenkkien puutteen, saati molekyylien liikkeen. On siis turha puhua evoluutiosta tieteenä. Se perustuu uskomuksiin siinä kuin muutkin metafyysiset olettamukset. Tieteen yksi perusperiaatteista on mitattavuus. Se mitä ei voida mitata, ei ole siis tiedettä.

        Lopuksi Herramme Jeesuksen sana, joka on tarkoitettu juuri sinulle. Lue se ajatuksella. Se ei hyökkää sinua vastaan vaan haluaa sinulle hyvää. Älä sinäkään siis hyökkää sitä vastaan.

        "Tulkaa minun luokseni kaikki te työn ja kuormien uuvuttamat. Minä annan teille levon. Ottakaa minun ikeeni harteillenne ja katsokaa minua: minä olen sydämeltäni lempeä ja nöyrä. Näin teidän sielunne löytää levon. Minun ikeeni on hyvä kantaa ja minun kuormani on kevyt." Matt. 11: 28-30


        Ystävällisin terveisin, Mikko Rantalahti (mikko.rantalahti[at]gmail.com)

        Evoluutiota voidaan mitatat vertaamalla eri sukupolvien geenejä ja niissä tapahtuneita muutoksia.


      • annuliinunen
        ateisti1982 kirjoitti:

        Ootko sä yhtä tyhjän kanssa Kemian DI:nä ja filosofian opettajana kuin Päivi Räsänen on lääkärinä ja kansanedustajana?

        Häneltä kysyttiin "Homoillassa". Mitä tekisit, mikäli lapsesi ilmoittaisi jonakin päivänä olevansa homo? Hän alkaisi ilmeisesti "eheyttämään" lastansa heteroksi!?

        Uskovaisilla on tunnetila ja homoseksuaaleilla on tahtotila. Yleensä tahtotila on vaan voimakkaampi kuin tunnetila!

        Ihmisarvoa viedään romukoppaan eli tällöinhän kirkko rikkoo pääsanomaansa, joka on "lähimmäisen rakkaus" eli nyt sitten halutaan "eheyttää" lähimmäisiä heteroksi!

        On se vaan kumma kun konservatiiviset ihmiset haluavat työntää nenäänsä muiden ihmisten "tahtotilaan".

        Minusta homoilla tulee olla samat oikeudet mennä avioliittoon kuin heteroillakin, koska homoilla on mahdollisuus rekisteröityyn parisuhteeseen, joka taas rikkoo heidän ihmisarvoja esim. hakiessa työtä eli ei välttämättä tule aina valituksi työsuhteeseen kun ovat ruksittaneet tai maininneet "rekisteröidyn parisuhteen"!!

        Itse olen mies ja olen elänyt n. 10 vuotta avoliitossa naisen kanssa, mutta meitä ei VIHITÄ avioliittoon vaan minusta tulisi olla sille muita uusia vaihtoehtoja eli esim. HAKEMINEN avioliittoon. Ns. "Avioliitto hakemuksen" täyttää nykyisinkin joko pappi tai maistraatin toimitsija puolisoiden puolesta, miksei siis PUOLISOT!?

        Olen itse uskovainen ollut jo lapsesta . No vastaan sinulle . Päivi ei ole yhtä tyhjän kanssa kansanedustajana, päinvastoin erittäin älykäs ja menestyjätyyppi, eräässäkin visailuohjelmassa pärjäsi niin ettei ikinä voisi moista uskoa . Äänestin itse häntä, oli sillloin muuten harvinaisuus vielä nainen keskisuuren puolueen johtajana. Tuosta sitten että mitä jos lapsi olisi niin tytön osalta
        meillä annetaan tällaiset kovat arvot esimerkin voimalla en ole itse mikään plösö ja tytärkin saa sellaiset elämänarvot että pitää kunnostaan huolta ja syö terveellisesti niin ei ole vartalo täydellisen pilalla kuten valitettavasti näin tv:n pride-kulkueessa että naisilla oli vartalot aivan piloilla, ei heitä taida kukaan mies haluta katsella tai kajota. Sitten on toki eri juttu jos on kauhean miesmäinen tai valtavan rumat kasvot sille ei sitten niin voi mitään vaikuttaa. Tai jos hormonitasapainossa sitten on jotain vikaa. Mutta aina sanotaan että nämä rakastavat toisiian mutta keitä sitten vihaavat niin ei tätä esim Päiviä ollenkaan hänhän on nainen ja pärjännyt, jos jotain vihaavat niin miehiä ja miesten ylivaltaa yhteiskunnan tietyillä osa-alueilla. Eli kyseessä on intensiivinen miesviha ehkä jopa kosto miehille. Totta on että työmarkkinoilla
        se ett' on perhetausta on meriitti. Se on varmasti kurja juttu että jos työnantaja tietää että hakija on homo niin ei pääse esim haastatteluun ja ei ole helppo vaihtaa työpaikkaa kun ei ole naimisissa ja perhettä
        Ja kun ei ole elämänsisältönä edes lapsia niin aina pahempaa kun ei pääse eteenpäin uralla eikä johtajaksi
        y.m. toki varmaan syrjitään ja rajusti en sitä epäile .

        Näillä naisilla voi olla esim helvetillinen aivan kauhean tasoinen ylipaino-ongelma , jota haukkumallasi päivillä esim ei ole. Tai osalla näytti olleen kehitysvammaisuuden piirteitä tuossa pridekulkueessa.
        Eräs nainen joskus sanoi että vika oli siinä että lapsena ei tuettu naiseutta tai tyttöyttä kun oli vammainen yksilö niin tuli sitten tällainen seksuaalivähemmistöläinen

        Se että jokin virkailija täyttää hakemuksen ja ettei voi tehdä esim netissä tai pois paikan päältä niin
        en tiedä mutta olla että virkailija katsoo esim että molemmat on vapaaehtoiset,tahtoo ja toinen ei esim huumattu tai muuta vastaavaa eron tullessa puolittuu omaisuus kuten eräs lähisukulainen hyvin oman kokemuksen kautta muutaman vuoden liiton jälkeen tiesi


      • Rompix kirjoitti:

        Evoluutiota voidaan mitatat vertaamalla eri sukupolvien geenejä ja niissä tapahtuneita muutoksia.

        Siis nyt en ymmärrä mitä tarkoitat ja ajat takaa?

        Eihän evoluutio tarkoita sitä että samaa lajia olleita tutkitaan eli ihmisiä ja ihmisten jälkeläisiä vaan sitä, miten esimerkiksi ihminen on kehittynyt apinasta ihmiseksi eli meillä on ollut sama kantaisä ja sitä tässä tutkitaan, miten neljällä jalalla liikkuvista apinaeläimistä on kehittynyt vähitellen kahdella jalalla liikkuva ihminen.

        Samaa lajia tutkitaan vain isyystesteissä tai halutaan saada tietää DNA? Mielestäni isyystestit tulisi toteuttaa myös silloin kun lapsen isä ja äiti elävät avioliitossa. Ihmettelen kyllä paljon sitä että avioliitossa syntyneiden lasten isyys tunnustetaan automaattisesti avioliiton merkityksen kautta. Tässä ei ole kysymys luottamuksen puutteesta vaan oikeudesta. Monikohan avioliitossa elänyt isä on lopulta lapsensa TODELLINEN isä!?

        Eräänlaista evoluutiota sekin on että meillä on erilaisia kasveja esim. kehäkukkaa ja tulppaaneja?


      • annuliinunen kirjoitti:

        Olen itse uskovainen ollut jo lapsesta . No vastaan sinulle . Päivi ei ole yhtä tyhjän kanssa kansanedustajana, päinvastoin erittäin älykäs ja menestyjätyyppi, eräässäkin visailuohjelmassa pärjäsi niin ettei ikinä voisi moista uskoa . Äänestin itse häntä, oli sillloin muuten harvinaisuus vielä nainen keskisuuren puolueen johtajana. Tuosta sitten että mitä jos lapsi olisi niin tytön osalta
        meillä annetaan tällaiset kovat arvot esimerkin voimalla en ole itse mikään plösö ja tytärkin saa sellaiset elämänarvot että pitää kunnostaan huolta ja syö terveellisesti niin ei ole vartalo täydellisen pilalla kuten valitettavasti näin tv:n pride-kulkueessa että naisilla oli vartalot aivan piloilla, ei heitä taida kukaan mies haluta katsella tai kajota. Sitten on toki eri juttu jos on kauhean miesmäinen tai valtavan rumat kasvot sille ei sitten niin voi mitään vaikuttaa. Tai jos hormonitasapainossa sitten on jotain vikaa. Mutta aina sanotaan että nämä rakastavat toisiian mutta keitä sitten vihaavat niin ei tätä esim Päiviä ollenkaan hänhän on nainen ja pärjännyt, jos jotain vihaavat niin miehiä ja miesten ylivaltaa yhteiskunnan tietyillä osa-alueilla. Eli kyseessä on intensiivinen miesviha ehkä jopa kosto miehille. Totta on että työmarkkinoilla
        se ett' on perhetausta on meriitti. Se on varmasti kurja juttu että jos työnantaja tietää että hakija on homo niin ei pääse esim haastatteluun ja ei ole helppo vaihtaa työpaikkaa kun ei ole naimisissa ja perhettä
        Ja kun ei ole elämänsisältönä edes lapsia niin aina pahempaa kun ei pääse eteenpäin uralla eikä johtajaksi
        y.m. toki varmaan syrjitään ja rajusti en sitä epäile .

        Näillä naisilla voi olla esim helvetillinen aivan kauhean tasoinen ylipaino-ongelma , jota haukkumallasi päivillä esim ei ole. Tai osalla näytti olleen kehitysvammaisuuden piirteitä tuossa pridekulkueessa.
        Eräs nainen joskus sanoi että vika oli siinä että lapsena ei tuettu naiseutta tai tyttöyttä kun oli vammainen yksilö niin tuli sitten tällainen seksuaalivähemmistöläinen

        Se että jokin virkailija täyttää hakemuksen ja ettei voi tehdä esim netissä tai pois paikan päältä niin
        en tiedä mutta olla että virkailija katsoo esim että molemmat on vapaaehtoiset,tahtoo ja toinen ei esim huumattu tai muuta vastaavaa eron tullessa puolittuu omaisuus kuten eräs lähisukulainen hyvin oman kokemuksen kautta muutaman vuoden liiton jälkeen tiesi

        Päivi on äärimmäisen rasittava ja ahdasmielinen ihminen, ettei toista voi tässä maassa enää olla toista.

        "Tuosta sitten että mitä jos lapsi olisi niin tytön osalta
        meillä annetaan tällaiset kovat arvot esimerkin voimalla en ole itse mikään plösö ja tytärkin saa sellaiset elämänarvot että pitää kunnostaan huolta ja syö terveellisesti niin ei ole vartalo täydellisen pilalla kuten valitettavasti näin tv:n pride-kulkueessa että naisilla oli vartalot aivan piloilla, ei heitä taida kukaan mies haluta katsella tai kajota"

        Itse en pahemmin katsellut Pride-kulkueessa olleiden naisten perään, mutta paheksun vaan sitä miten heitä kohdeltiin ko. kulkueessa. Oliko se sinusta oikeutettua? Minusta ei!

        Loppu sinulta on julmaa ja turhaa rasistista tekstiä ja sen tuomaa vaahtoomista, jota varmasti Raamattu hyväksyy ja eräät miespuoliset kirkosta irtisanotut pastorit, jotka eivät voi työskennellä naisten kanssa!!

        Kukin saa olla niin ruma kuin itse tykkää tai näyttää kauniilta?! Ei sen pitäisi olla sinulta tai minulta mitenkään pois?


      • annuliinunen kirjoitti:

        Olen itse uskovainen ollut jo lapsesta . No vastaan sinulle . Päivi ei ole yhtä tyhjän kanssa kansanedustajana, päinvastoin erittäin älykäs ja menestyjätyyppi, eräässäkin visailuohjelmassa pärjäsi niin ettei ikinä voisi moista uskoa . Äänestin itse häntä, oli sillloin muuten harvinaisuus vielä nainen keskisuuren puolueen johtajana. Tuosta sitten että mitä jos lapsi olisi niin tytön osalta
        meillä annetaan tällaiset kovat arvot esimerkin voimalla en ole itse mikään plösö ja tytärkin saa sellaiset elämänarvot että pitää kunnostaan huolta ja syö terveellisesti niin ei ole vartalo täydellisen pilalla kuten valitettavasti näin tv:n pride-kulkueessa että naisilla oli vartalot aivan piloilla, ei heitä taida kukaan mies haluta katsella tai kajota. Sitten on toki eri juttu jos on kauhean miesmäinen tai valtavan rumat kasvot sille ei sitten niin voi mitään vaikuttaa. Tai jos hormonitasapainossa sitten on jotain vikaa. Mutta aina sanotaan että nämä rakastavat toisiian mutta keitä sitten vihaavat niin ei tätä esim Päiviä ollenkaan hänhän on nainen ja pärjännyt, jos jotain vihaavat niin miehiä ja miesten ylivaltaa yhteiskunnan tietyillä osa-alueilla. Eli kyseessä on intensiivinen miesviha ehkä jopa kosto miehille. Totta on että työmarkkinoilla
        se ett' on perhetausta on meriitti. Se on varmasti kurja juttu että jos työnantaja tietää että hakija on homo niin ei pääse esim haastatteluun ja ei ole helppo vaihtaa työpaikkaa kun ei ole naimisissa ja perhettä
        Ja kun ei ole elämänsisältönä edes lapsia niin aina pahempaa kun ei pääse eteenpäin uralla eikä johtajaksi
        y.m. toki varmaan syrjitään ja rajusti en sitä epäile .

        Näillä naisilla voi olla esim helvetillinen aivan kauhean tasoinen ylipaino-ongelma , jota haukkumallasi päivillä esim ei ole. Tai osalla näytti olleen kehitysvammaisuuden piirteitä tuossa pridekulkueessa.
        Eräs nainen joskus sanoi että vika oli siinä että lapsena ei tuettu naiseutta tai tyttöyttä kun oli vammainen yksilö niin tuli sitten tällainen seksuaalivähemmistöläinen

        Se että jokin virkailija täyttää hakemuksen ja ettei voi tehdä esim netissä tai pois paikan päältä niin
        en tiedä mutta olla että virkailija katsoo esim että molemmat on vapaaehtoiset,tahtoo ja toinen ei esim huumattu tai muuta vastaavaa eron tullessa puolittuu omaisuus kuten eräs lähisukulainen hyvin oman kokemuksen kautta muutaman vuoden liiton jälkeen tiesi

        Hyvin ulkoisesti säilynyt nainen ei välttämättä tarkoita, etteikö olisi päässä jotakin vikaa? Eli sisäinen kauneus puuttuu.

        Esimerkkini:
        Ulkoa vastaan kävelee minua hyvin kaunis nainen, jolla on tupakka sormien välissä. Minun mielenkiintoni lopahti juuri siihen paikkaan. Nainen on pilannut minun silmissäni jyrkästi sanottuna ns. "sisäisen kauneutensa", polttamalla tupakkaa!


      • .........
        ateisti1982 kirjoitti:

        Siis nyt en ymmärrä mitä tarkoitat ja ajat takaa?

        Eihän evoluutio tarkoita sitä että samaa lajia olleita tutkitaan eli ihmisiä ja ihmisten jälkeläisiä vaan sitä, miten esimerkiksi ihminen on kehittynyt apinasta ihmiseksi eli meillä on ollut sama kantaisä ja sitä tässä tutkitaan, miten neljällä jalalla liikkuvista apinaeläimistä on kehittynyt vähitellen kahdella jalalla liikkuva ihminen.

        Samaa lajia tutkitaan vain isyystesteissä tai halutaan saada tietää DNA? Mielestäni isyystestit tulisi toteuttaa myös silloin kun lapsen isä ja äiti elävät avioliitossa. Ihmettelen kyllä paljon sitä että avioliitossa syntyneiden lasten isyys tunnustetaan automaattisesti avioliiton merkityksen kautta. Tässä ei ole kysymys luottamuksen puutteesta vaan oikeudesta. Monikohan avioliitossa elänyt isä on lopulta lapsensa TODELLINEN isä!?

        Eräänlaista evoluutiota sekin on että meillä on erilaisia kasveja esim. kehäkukkaa ja tulppaaneja?

        Wikipedia sen kertoo;
        "Evoluutio tarkoittaa sukupolvien myötä tapahtuvia muutoksia eliöiden muodostaman populaation geenivarastossa. "

        Evoluutio ei ola välttämättä sitä että eliöt lajiutuvat eri lajeiksi, vaan myös niitä pienenpieniä muutoksia populaatiossa, joita ei silmin edes havaitse.

        "miten esimerkiksi ihminen on kehittynyt apinasta ihmiseksi"

        Ihminen ei ole vain kehittynyt apinasta, vaan ihminen on apina, samoin kuin ihminen on kädellinen, nisäkäs, selkärankainen, selkäjänteinen ja aitotumallinen.


      • Quicunque
        Schlechterwisser kirjoitti:

        "Onko sinulla jotain esimerkkejä teoriassa sattumalta tapahtuvista molekyylitason tapahtumista?"
        "Heisenbergin epävarmuusperiaatteen mukaisesti emme voi tietää tarkasti kappaleiden sijaintia emmekä liikettä…"

        Tiedän Heisenbergin epävarmuusperiaatteen kyllä, mutta olin luullut että se koskee atomitason tapahtumia.

        "Nolla ei voi olla vakio, kun puhutaan energian tai minkä tahansa asian olevasta määrästä."

        Tuossa toisessa viestissä jo tulikin esiin, että maailmankaikkeuden energian kokonaismäärä saattaa hyvinkin olla nolla.

        "Evoluutiosta halusin tuoda esille sen, että sitä ei voida mitata. Se ei ole siis tiedettä, kuten itsekin mainitsit."

        Sanoin, että evoluutio ei ole tiedettä, koska evoluutio on luonnossa esiintyvä ilmiö. Ilmiö ei ole tiedettä. Tätä kyseistä ilmiötä tutkii biologia, joka on yksi tieteenala.

        "Se tekee siis kaikesta tästä metafysiikkaa ja siihen sisältyy myös evoluutio, halusit tai et. Tämä on siis jyrkän tieteellinen näkökulma asiaan."

        Jos näkökulmasi tosiaan on tieteellinen, niinkuin sanoit, niin sinusta tulee kuuluisa. Silloinhan voit yhdellä pienellä tieteellisellä artikkelilla kumota kaiken tähän asti evoluutiosta tehdyn tutkimuksen. Suosittelen ehdottomasti lähettämään teoriasi vaikkapa Natureen. Samalla muutama muukin tieteentekijä voisi kanssasi arvioida onko näkökulmasi aivan varmasti "jyrkän tieteellinen". Minä en liene siihen riittävän oppinut, sillä minä en millään ymmärrä mikä tekee biologiasta metafysiikkaa.

        Millaisia todisteita evoluutiosta noin niinkuin täsmälleen ottaen vielä haluaisit?

        "Eikö olekin hienoa, että puhun jonkun mielestä aikamoisella kontrastilla tiedettä ja uskoa. Itse ainakin hämmästyn jatkuvasti. Voisin jopa pitää itseäni melkoisena sekopäänä, jos en tietäisi mistä puhun."

        Olen samaa mieltä, paitsi että minä EN tiedä mistä sinä puhut.

        "Kuitenkaan en näe näillä asioilla ristiriitaa, koska minulla ei ole tarvetta olla maailman synnyn suhteen oikeassa."

        Minä taas näen niiden välillä ristiriidan, koska minulla ei ole tarvetta pitää väkisin jotakin jumalolentoa kaiken alkusyynä.

        Taidan olla vähän yksinkertainen kun en ymmärrä yhtä asiaa: Jos emme koskaan voi saada selville millä tavoin maailma on syntynyt, niin miksi emme voi vain sanoa ettemme tiedä? Miksi pitäisi alkaa uskomaan ja väittämään ilman mitään perusteita, että maailman on luonut joku tonttu tai muu myyttinen olento?

        Huomaa Schlechterwisser, että toin esille myös muita näkökulmia kuin tieteellisen. Ajattelin vain puhua sillä kielellä, kun täällä oli monia, jotka tuntuvat olevan siitä hyvin perillä ja ymmärtävät tieteen kieltä parhaiten. (Kiitos muuten tarkennuksesta biologian suhteen. Sitä pidän hyvinkin tärkeänä tieteen alana. :))

        Emme voikaan olla varmoja siitä mikä on totta, koska emme voi saada siitä kuin henkilökohtaisen kokemuksen omista lähtokohdistamme. Tämän vuoksi meidän on pakko tehdä päätelmät omista lähtökohdistamme ja oman kokemuksemme tai pohdintamme kautta. Voimme toki nojautua auktoriteetteihin tai erilaisiin teorioihin, mutta lopulta joudumme kuitenkin päättämään mitä haluamme tehdä elämällämme ja kuinka siitä ajattelemme ja meillä on siihen täysi oikeus. Tämä elämä on meitä itseämme varten.

        Itse tulin uskoon n. 14 vuotta sitten epäilysten ja hyvin samantyyppisten pohdintojen kautta kuin täällä nyt käydään. En suostunut ottamaan mitään annettuna vaan etenin hyvin johdonmukaisesti epäillen. (En nyt sentään Descartesin tapaan lähtenyt epäilemään ulkomaailman olemassaoloa, mutta epäilin kuitenkin monia asioita nimenomaan Jumalassa, jota muuten Descartes ei koskaan tehnyt.) Lopulta minulle ei jäänyt muuta vaihtoehtoa kuin rukous, koska olin hyvin ahdistunut kaiken suhteen. Epäillen ryhdyin rukoilemaan ja rukoilin jotenkin näin (en muista tarkkoja sanoja, koska siitä on jo sen verran aikaa): "Jeesus, jos sinä kerran olet olemassa, niin paljastu minulle. Jos olet olemassa, niin sitten minäkin haluan tulla uskoon." Rukoilin melko pitkään ja paljon muitakin asioita keskellä yötä ja aikaa kului ehkä noin tunnin verran. Tämän jälkeen valo ei laskeutunut ylleni tai mitään muuta yhtä merkittävää, mutta ahdistus, joka minulla oli ollut, oli poissa. Sitten uskon asiat vähän niin kuin unohtuivat, kun opiskelut alkoivat ja menin mukaan opiskelijaelämään ja elin Jumalan tahtoa vastaan. Koko ajan minusta kuitenkin pidettiin kiinni, vaikka tein paljon asioita, joista en ole erityisen ylpeä. Eheytyminen uskoni suhteen ja elämäni suhteen muutenkin tapahtui lopulta, kun tapasin nykyisen vaimoni, joka toi uskonasiat uudelleen pinnalle. Tajusin, että olin elänyt väärin sekä Jumalaa, että itseäni vastaan, puhumattakaan muista ihmisistä. Silloin Jumala lopulta paljastui minulle lopullisesti ja sitä tilaa tai tunnetta en osaa selittää kuin sellaiselle, joka sen on kokenut. (Verbaaliset taitoni eivät yksinkertaisesti riitä.) Olin joutunut uudestaan tilanteeseen, jossa minulla ei ollut muuta vaihtoehtoa ulos kuin vapautus synnistä ja se ei voinut tapahtua muuten kuin Jeesuksen avulla, koska itse en osannut enkä voinut itseäni auttaa. Siitä alkoi eheytyminen ihmisenä ja prosessi, joka jatkuu vielä nykyäänkin useiden vuosien jälkeen ja tuskinpa tässä valmiiksi tullaankaan. Pelastumisestani olen kuitenkin varma jo tässä maanpäällisessä elämässäni ja se on minulle tarpeeksi tapahtuipa mitä tahansa.

        Tämän olen kirjoittanut omista kokemuksistani, omista lähtökohdistani ja omalla nimelläni. Sinun lähtökohtasi on varmasti erilainen, mutta samaa siinä on mielestäni ainakin se, että kyse on sinun omasta elämästäsi ja mitä haluat sen kanssa tehdä. Epäilys ja totuuden etsiminen on hyvä lähtökohta. :)

        Kaikkea hyvää, Mikko Rantalahti (mikko.rantalahti[at]gmail.com)


      • Quicunque
        Trollinaattori kirjoitti:

        "Kuitenkaan en näe näillä asioilla ristiriitaa, koska minulla ei ole tarvetta olla maailman synnyn suhteen oikeassa. "

        ...mutta kuitenkin sinulla tuntuu olevan tarve olla oikeassa siinä, että maailma on syntynyt. Vaikka kovasti vakuutat menneen tutkimisen olevan metafysiikkaa.

        "Voisin jopa pitää itseäni melkoisena sekopäänä, jos en tietäisi mistä puhun. "

        " Energiaa joko on, tai sitä ei ole. Jos sitä ei olisi, niin silloin sen määrä olisi nolla. "

        Pidä itseäsi melkoisena sekopäänä. Ehtolausekkeesi ei tosin kuulunut että:voisin pitää itseäni sekopäänä, jos tietäisin mistä puhun.

        Mistäköhän energiasta sinä puhut? Energia muuttaa muotoaan. Kun jossain on "negatiivinen" on toisaalla "positiivinen". Yhdessä nämä muodostavat nollan. Et siis tiedä mistä puhut. Voithan toki hakkailla olkiukkoasi yhä uudelelleen ja uudelleen, kanssauskovaisten lyöden tahtiä innoissansa karvaisilla käpälillään.

        Trollinaattori: Lue alta.


      • annuliinunen
        ateisti1982 kirjoitti:

        Hyvin ulkoisesti säilynyt nainen ei välttämättä tarkoita, etteikö olisi päässä jotakin vikaa? Eli sisäinen kauneus puuttuu.

        Esimerkkini:
        Ulkoa vastaan kävelee minua hyvin kaunis nainen, jolla on tupakka sormien välissä. Minun mielenkiintoni lopahti juuri siihen paikkaan. Nainen on pilannut minun silmissäni jyrkästi sanottuna ns. "sisäisen kauneutensa", polttamalla tupakkaa!

        Kysymys on myös paljolti siitä miten voi saada lapsen, lesboparille se ei ilman ulkoista apua onnistu siis keskenään ei onnistu ja homomiehille se ei onnistu lainkaan. Todennäkiöisesti isäkandidaatin puhuminen lesboparin avuksi on vaikeaa. Jonkin sopivan miehen taivuttelussa on varmaan kova vaiva ja vaatii kykyä mieskontaktiin. Kaikki klinikat ei hoida ja hoito ei aina onnistu ei kellekään ja tulee kalliiksi. Mutta nämä on sitten hintaa siitä jutusta. Luonto karsii nämä tyypit ja darwinin laki jyllää. Päivillä jos häntä nyt esimerkkinä vielä käyttää on viisi lasta.


      • .........
        annuliinunen kirjoitti:

        Kysymys on myös paljolti siitä miten voi saada lapsen, lesboparille se ei ilman ulkoista apua onnistu siis keskenään ei onnistu ja homomiehille se ei onnistu lainkaan. Todennäkiöisesti isäkandidaatin puhuminen lesboparin avuksi on vaikeaa. Jonkin sopivan miehen taivuttelussa on varmaan kova vaiva ja vaatii kykyä mieskontaktiin. Kaikki klinikat ei hoida ja hoito ei aina onnistu ei kellekään ja tulee kalliiksi. Mutta nämä on sitten hintaa siitä jutusta. Luonto karsii nämä tyypit ja darwinin laki jyllää. Päivillä jos häntä nyt esimerkkinä vielä käyttää on viisi lasta.

        Eli jos lapsensaanti on edellytyksenä parisuhteelle, hedelmättömät pariskunnat eivät saa olla naimisissa ja hedelmällisen iän ylittäneiden pariskuntien pitää erota?


      • Rompix
        ateisti1982 kirjoitti:

        Siis nyt en ymmärrä mitä tarkoitat ja ajat takaa?

        Eihän evoluutio tarkoita sitä että samaa lajia olleita tutkitaan eli ihmisiä ja ihmisten jälkeläisiä vaan sitä, miten esimerkiksi ihminen on kehittynyt apinasta ihmiseksi eli meillä on ollut sama kantaisä ja sitä tässä tutkitaan, miten neljällä jalalla liikkuvista apinaeläimistä on kehittynyt vähitellen kahdella jalalla liikkuva ihminen.

        Samaa lajia tutkitaan vain isyystesteissä tai halutaan saada tietää DNA? Mielestäni isyystestit tulisi toteuttaa myös silloin kun lapsen isä ja äiti elävät avioliitossa. Ihmettelen kyllä paljon sitä että avioliitossa syntyneiden lasten isyys tunnustetaan automaattisesti avioliiton merkityksen kautta. Tässä ei ole kysymys luottamuksen puutteesta vaan oikeudesta. Monikohan avioliitossa elänyt isä on lopulta lapsensa TODELLINEN isä!?

        Eräänlaista evoluutiota sekin on että meillä on erilaisia kasveja esim. kehäkukkaa ja tulppaaneja?

        "Siis nyt en ymmärrä mitä tarkoitat ja ajat takaa? "

        Vastasin sille kemian DI:lle, kun hän väitti ettei evoluutiota voi mitata ja se ei siksi olisi tiedettä.

        En minäkään oikein tajunnut mitä sinä ajoit vastauksellasi takaa :)


      • Quicunque
        ateisti1982 kirjoitti:

        Ootko sä yhtä tyhjän kanssa Kemian DI:nä ja filosofian opettajana kuin Päivi Räsänen on lääkärinä ja kansanedustajana?

        Häneltä kysyttiin "Homoillassa". Mitä tekisit, mikäli lapsesi ilmoittaisi jonakin päivänä olevansa homo? Hän alkaisi ilmeisesti "eheyttämään" lastansa heteroksi!?

        Uskovaisilla on tunnetila ja homoseksuaaleilla on tahtotila. Yleensä tahtotila on vaan voimakkaampi kuin tunnetila!

        Ihmisarvoa viedään romukoppaan eli tällöinhän kirkko rikkoo pääsanomaansa, joka on "lähimmäisen rakkaus" eli nyt sitten halutaan "eheyttää" lähimmäisiä heteroksi!

        On se vaan kumma kun konservatiiviset ihmiset haluavat työntää nenäänsä muiden ihmisten "tahtotilaan".

        Minusta homoilla tulee olla samat oikeudet mennä avioliittoon kuin heteroillakin, koska homoilla on mahdollisuus rekisteröityyn parisuhteeseen, joka taas rikkoo heidän ihmisarvoja esim. hakiessa työtä eli ei välttämättä tule aina valituksi työsuhteeseen kun ovat ruksittaneet tai maininneet "rekisteröidyn parisuhteen"!!

        Itse olen mies ja olen elänyt n. 10 vuotta avoliitossa naisen kanssa, mutta meitä ei VIHITÄ avioliittoon vaan minusta tulisi olla sille muita uusia vaihtoehtoja eli esim. HAKEMINEN avioliittoon. Ns. "Avioliitto hakemuksen" täyttää nykyisinkin joko pappi tai maistraatin toimitsija puolisoiden puolesta, miksei siis PUOLISOT!?

        En nyt ymmärrä tahtotilaa ja tunnetilaa tässä asiayhteydessä. Voisitko tarkentaa mitä niillä tarkoitat?

        Avioliitosta: Minulla on useita ystäviä, jotka ovat eläneet pitkään avoliitossa ja menneet naimisiin esim. yhteisen lapsen synnyttyä. En osaa vastata heidän puolestaan, mutta tuskinpa sanamuoto esim. maistraatissa suoritetun vihkimisen suhteen on siinä vaiheessa kovin tärkeää. Eikö tärkeämpää ole yhdessäolo ja yhteinen perhe?


      • annuliinunen
        ......... kirjoitti:

        Eli jos lapsensaanti on edellytyksenä parisuhteelle, hedelmättömät pariskunnat eivät saa olla naimisissa ja hedelmällisen iän ylittäneiden pariskuntien pitää erota?

        Hedelämtön pari voi adoptoida tai heille voi kuitenkin siunaantua lapsi tai voivat toimia sijaisvanhempina.
        Tai voivat hakea elämään sisältöä muuta kautta.
        Mummon ja vaarin ei toki tarvii erota niin kuin ei meidän yhteiskunnassa yleensä tapahdukaan .
        Avioliittoon voi mies ja nainen mennä toki myäs korkealla iällä esim seuran ja turvan avun tai muun syyn vuoksi.Avioliitto on miehen ja naisen välinen liitto ei esim naisen ja kissan liitto tai kahden naisen liitto. Laajemmin maailmalla on miesten moniavioisuus ja joissain kulttuureissa naisten moniavioisuus ilmeiseti jossain määrin tapana ja vanhaan aikaan Raamatussa esim Aabrahamilla oli neljä vaimoa jota varmaan muslimit imitoivat . Tässä mielessä mikäli kulttuuriin kuuluu mielestäni luonnollisempaa olisi tämän tilanteen legitimointi esim siis muslimimiehen useamman vaimon mikäli tosiasiallinen tilanne tämä jo on kuin homoliitto. Eli siis mikäli on perhettä parin naisen kanssa niin kuin ilmeisesti afrikkalaista alkuperää olevilla joskus usassa esim on niin voisi sitten toinenkin perhe olla täydessä arvossa.


      • Rompix kirjoitti:

        "Siis nyt en ymmärrä mitä tarkoitat ja ajat takaa? "

        Vastasin sille kemian DI:lle, kun hän väitti ettei evoluutiota voi mitata ja se ei siksi olisi tiedettä.

        En minäkään oikein tajunnut mitä sinä ajoit vastauksellasi takaa :)

        Tämä keskustelupalsta on perseestä. Eihän täällä tiedä, ketä kirjoittaa kenellekin. Ois vaan mukavaa, jos jokainen ottaisi jonkinlaisen LAINAUKSEN edellisen kirjoittajan viestistä!?!?


      • .........
        annuliinunen kirjoitti:

        Hedelämtön pari voi adoptoida tai heille voi kuitenkin siunaantua lapsi tai voivat toimia sijaisvanhempina.
        Tai voivat hakea elämään sisältöä muuta kautta.
        Mummon ja vaarin ei toki tarvii erota niin kuin ei meidän yhteiskunnassa yleensä tapahdukaan .
        Avioliittoon voi mies ja nainen mennä toki myäs korkealla iällä esim seuran ja turvan avun tai muun syyn vuoksi.Avioliitto on miehen ja naisen välinen liitto ei esim naisen ja kissan liitto tai kahden naisen liitto. Laajemmin maailmalla on miesten moniavioisuus ja joissain kulttuureissa naisten moniavioisuus ilmeiseti jossain määrin tapana ja vanhaan aikaan Raamatussa esim Aabrahamilla oli neljä vaimoa jota varmaan muslimit imitoivat . Tässä mielessä mikäli kulttuuriin kuuluu mielestäni luonnollisempaa olisi tämän tilanteen legitimointi esim siis muslimimiehen useamman vaimon mikäli tosiasiallinen tilanne tämä jo on kuin homoliitto. Eli siis mikäli on perhettä parin naisen kanssa niin kuin ilmeisesti afrikkalaista alkuperää olevilla joskus usassa esim on niin voisi sitten toinenkin perhe olla täydessä arvossa.

        "Avioliitto on miehen ja naisen välinen liitto ei ... kahden naisen liitto"

        Viittaus kissan ja ihmisen väliseen liittoon oli niin täysin typerä että jätän kommentoimatta kummemmin. Osaatko kuitenkin perustella jotenkin, miksei avioliitto voisi olla kahden naisen välinen liitto? Esiin tuli, että edellisestä viestistäsi huolimatta, syynä ei ole lastenhankinta.


      • Quicunque kirjoitti:

        Huomaa Schlechterwisser, että toin esille myös muita näkökulmia kuin tieteellisen. Ajattelin vain puhua sillä kielellä, kun täällä oli monia, jotka tuntuvat olevan siitä hyvin perillä ja ymmärtävät tieteen kieltä parhaiten. (Kiitos muuten tarkennuksesta biologian suhteen. Sitä pidän hyvinkin tärkeänä tieteen alana. :))

        Emme voikaan olla varmoja siitä mikä on totta, koska emme voi saada siitä kuin henkilökohtaisen kokemuksen omista lähtokohdistamme. Tämän vuoksi meidän on pakko tehdä päätelmät omista lähtökohdistamme ja oman kokemuksemme tai pohdintamme kautta. Voimme toki nojautua auktoriteetteihin tai erilaisiin teorioihin, mutta lopulta joudumme kuitenkin päättämään mitä haluamme tehdä elämällämme ja kuinka siitä ajattelemme ja meillä on siihen täysi oikeus. Tämä elämä on meitä itseämme varten.

        Itse tulin uskoon n. 14 vuotta sitten epäilysten ja hyvin samantyyppisten pohdintojen kautta kuin täällä nyt käydään. En suostunut ottamaan mitään annettuna vaan etenin hyvin johdonmukaisesti epäillen. (En nyt sentään Descartesin tapaan lähtenyt epäilemään ulkomaailman olemassaoloa, mutta epäilin kuitenkin monia asioita nimenomaan Jumalassa, jota muuten Descartes ei koskaan tehnyt.) Lopulta minulle ei jäänyt muuta vaihtoehtoa kuin rukous, koska olin hyvin ahdistunut kaiken suhteen. Epäillen ryhdyin rukoilemaan ja rukoilin jotenkin näin (en muista tarkkoja sanoja, koska siitä on jo sen verran aikaa): "Jeesus, jos sinä kerran olet olemassa, niin paljastu minulle. Jos olet olemassa, niin sitten minäkin haluan tulla uskoon." Rukoilin melko pitkään ja paljon muitakin asioita keskellä yötä ja aikaa kului ehkä noin tunnin verran. Tämän jälkeen valo ei laskeutunut ylleni tai mitään muuta yhtä merkittävää, mutta ahdistus, joka minulla oli ollut, oli poissa. Sitten uskon asiat vähän niin kuin unohtuivat, kun opiskelut alkoivat ja menin mukaan opiskelijaelämään ja elin Jumalan tahtoa vastaan. Koko ajan minusta kuitenkin pidettiin kiinni, vaikka tein paljon asioita, joista en ole erityisen ylpeä. Eheytyminen uskoni suhteen ja elämäni suhteen muutenkin tapahtui lopulta, kun tapasin nykyisen vaimoni, joka toi uskonasiat uudelleen pinnalle. Tajusin, että olin elänyt väärin sekä Jumalaa, että itseäni vastaan, puhumattakaan muista ihmisistä. Silloin Jumala lopulta paljastui minulle lopullisesti ja sitä tilaa tai tunnetta en osaa selittää kuin sellaiselle, joka sen on kokenut. (Verbaaliset taitoni eivät yksinkertaisesti riitä.) Olin joutunut uudestaan tilanteeseen, jossa minulla ei ollut muuta vaihtoehtoa ulos kuin vapautus synnistä ja se ei voinut tapahtua muuten kuin Jeesuksen avulla, koska itse en osannut enkä voinut itseäni auttaa. Siitä alkoi eheytyminen ihmisenä ja prosessi, joka jatkuu vielä nykyäänkin useiden vuosien jälkeen ja tuskinpa tässä valmiiksi tullaankaan. Pelastumisestani olen kuitenkin varma jo tässä maanpäällisessä elämässäni ja se on minulle tarpeeksi tapahtuipa mitä tahansa.

        Tämän olen kirjoittanut omista kokemuksistani, omista lähtökohdistani ja omalla nimelläni. Sinun lähtökohtasi on varmasti erilainen, mutta samaa siinä on mielestäni ainakin se, että kyse on sinun omasta elämästäsi ja mitä haluat sen kanssa tehdä. Epäilys ja totuuden etsiminen on hyvä lähtökohta. :)

        Kaikkea hyvää, Mikko Rantalahti (mikko.rantalahti[at]gmail.com)

        "Huomaa Schlechterwisser, että toin esille myös muita näkökulmia kuin tieteellisen. Ajattelin vain puhua sillä kielellä, kun täällä oli monia, jotka tuntuvat olevan siitä hyvin perillä ja ymmärtävät tieteen kieltä parhaiten."

        Mihin asiaan vastasit tällä kommentilla? Esitin sinulle viestissäni monta kysymystä, mutta en oikein ymmärrä mihin niistä tämä vastaa.

        "Voimme toki nojautua auktoriteetteihin tai erilaisiin teorioihin…"

        Kuiten Raamattuun ja Jumalaan vai? Miksi niihin pitäisi nojautua? Minun mielestäni auktoriteetit saavat todistaa väitteensä ihan samalla tavalla kuin muutkin. Nojaudut viesteissäsi Raamattuun ja puhut Jeesuksesta ikään kuin ne olisivat totta, ja evoluutiosta ikään kuin se ei olisi varmaa. Minkä auktoriteetin perusteella olet päätynyt noihin mielipiteisiisi? Raamatun ja Puolimatkan?

        Kysyin jo äsken: Jos emme koskaan voi saada selville millä tavoin maailma on syntynyt, niin miksi emme voi vain sanoa ettemme tiedä? Miksi pitäisi alkaa uskomaan ja väittämään ilman mitään perusteita, että maailman on luonut joku tonttu tai muu myyttinen olento?

        "Sinun lähtökohtasi on varmasti erilainen, mutta samaa siinä on mielestäni ainakin se, että kyse on sinun omasta elämästäsi ja mitä haluat sen kanssa tehdä."

        Kas, siinähän onkin jo paljon samaa, kyse on molempien omasta elämästä. Minä olin nuorempana hyvin samanlainen kuin sinä nyt, ja saatoin innokkaasti uskoa asioihin joista ei ollut mitään todisteita. Luulin olevani parempi ja viisaampi kuin muut (vaikka muille vähättelinkin arvoani ja korostin nöyryyttäni), ja jaoin mielelläni neuvoja muille, ja kerroin omista kokemuksistani, koska luulin että ne olisivat muillekin tärkeitä ja merkityksellisiä.

        Nyt vanhempana olen ymmärtänyt elämästä ja todellisuudesta paljon enemmän, ja tajunnut millaisessa hybriksessä nuorempana elin. Nyt tiedän, että emme tiedä paljon mitään, ja voimme vain yrittää hankkia mahdollisimman paljon, mahdollisimman hyvää tietoa, jotta voimme järjestää asiat niin, että kaikilla meillä olisi parempi elää. Oloni on nyt paljon tasapainoisempi ja tyynempi, ja ahdistuksista ei ole tietoakaan. Toivot että sinäkin joskus vielä kehityt tähän tilaan.

        Ja sitten uudelleen näitä kysymyksiä joita unohdit kommentoida:

        Joko olet ymmärtänyt, että energian kokonaismäärä maailmankaikkeudessa voi ihan hyvin olla nolla?

        Mikä olikaan se "jyrkän tieteellinen" osuus väitteistäsi, joka tekee biologiasta metafysiikaa?

        Millaisia todisteita täsmälleen ottaen vielä haluaisit evoluutiosta?


      • annuliinunen
        ......... kirjoitti:

        "Avioliitto on miehen ja naisen välinen liitto ei ... kahden naisen liitto"

        Viittaus kissan ja ihmisen väliseen liittoon oli niin täysin typerä että jätän kommentoimatta kummemmin. Osaatko kuitenkin perustella jotenkin, miksei avioliitto voisi olla kahden naisen välinen liitto? Esiin tuli, että edellisestä viestistäsi huolimatta, syynä ei ole lastenhankinta.

        Meille tämä on yleensä itsestään selvää , parisuhde ja seksuaalinen parisuhde
        on miehen ja naisen välinen samaa sukupuolta olevien välillä on ystävyyttä toveruutta , kaveruutta työtoveruutta mitä sitten onkin mutta parinmuodostus
        perhe seksuaaliasiat kuuluu mielestämme eri sukupuolta oleville ja mikäli tätä asiaa odotat niin Raamatun mukaan on myös näin ja uskonnon perinteen mukaan .Mutta jos suomipoitsulla on jotain tähään vastaan sanomista niin suomipoitsu sitten korvataan Jumalaan
        uskovalla ja miehen ja naisen liittoa ainoana mahdollisuutena arvostavalla esim turkkilaismiehellä .Että emme tule vastaan panemaan mikäli suomipoitsu tuntee epävarmuutta tai ei ole varma sukupuolisesta identiteetistään .
        Itse asiassa kristityn ja uskovan kristityn kannalta harras muslimi on parempi
        kuin esim kirkosta eronnut ateisti On niinkuin enemmän yhteistä.
        Täytyy vaan sitten naisporukalla valitella että kun ei ole identiteetiltään selvää miesporukkaa vai mitä ?

        Että niin kuin joko tai .

        Itse uskon että naiset kammoaisivat jotenkin homosuhteissa ollutta tai niihin kovin taipuvaista miestä mutta mitään varmaa tietoa asiasta ei toki minulla ole.Tod näk tämän taipumuksen joutuu naisilta / tytöiltä salaamaan.
        Muodostuu muuten esteeksi .


      • Oletko...
        annuliinunen kirjoitti:

        Meille tämä on yleensä itsestään selvää , parisuhde ja seksuaalinen parisuhde
        on miehen ja naisen välinen samaa sukupuolta olevien välillä on ystävyyttä toveruutta , kaveruutta työtoveruutta mitä sitten onkin mutta parinmuodostus
        perhe seksuaaliasiat kuuluu mielestämme eri sukupuolta oleville ja mikäli tätä asiaa odotat niin Raamatun mukaan on myös näin ja uskonnon perinteen mukaan .Mutta jos suomipoitsulla on jotain tähään vastaan sanomista niin suomipoitsu sitten korvataan Jumalaan
        uskovalla ja miehen ja naisen liittoa ainoana mahdollisuutena arvostavalla esim turkkilaismiehellä .Että emme tule vastaan panemaan mikäli suomipoitsu tuntee epävarmuutta tai ei ole varma sukupuolisesta identiteetistään .
        Itse asiassa kristityn ja uskovan kristityn kannalta harras muslimi on parempi
        kuin esim kirkosta eronnut ateisti On niinkuin enemmän yhteistä.
        Täytyy vaan sitten naisporukalla valitella että kun ei ole identiteetiltään selvää miesporukkaa vai mitä ?

        Että niin kuin joko tai .

        Itse uskon että naiset kammoaisivat jotenkin homosuhteissa ollutta tai niihin kovin taipuvaista miestä mutta mitään varmaa tietoa asiasta ei toki minulla ole.Tod näk tämän taipumuksen joutuu naisilta / tytöiltä salaamaan.
        Muodostuu muuten esteeksi .

        Päivi Räsänen???


      • Quicunque kirjoitti:

        Siunausta sinulle ateisti1982. Olet oikean asian äärellä jatkuvan kyselemisesi kanssa. Toivottavasti löydät totuuden, niin sinun ei tarvitse olla koko ajan vihainen. Sanon tämän ystävänä.

        Olen koulutukseltani kemian DI ja filosofian opettaja ja hyvin paljon käsitellyt tieteen ja uskon välistä suhdetta. En ole koskaan kokenut ristiriitaa omissa opinnoissani tai opetuksessani tieteen ja uskon välillä. Sen sijaan, kun mennään tarpeeksi syvälle molemmissa, niin huomataan monia yhtäläisyyksiä. Molekyylitasolla asiat tapahtuvat teoriassa sattumalta, mutta se johtuu vain siitä, ettemme osaa kutsua sitä muuksi kuin sattumaksi. Emme pysty määrittelemään molekyylien liikkeitä, vaikka ne tapahtuvat silmiemme edessä. Kuinka pystyisimme siis määrittelemään maailmankaikkeuden liikkeitä aikojen alussa?

        Energian määrä maailmankaikkeudessa on vakio. Energiaa ei voi syntyä tyhjästä, se vain muuttaa muotoaan. Ei ole siis mahdollista, että maailmankaikkeus olisi voinut syntyä tyhjästä. Jotain on siis täytynyt olla aina ja tulee aina olemaan. On turha kuvitella, että olisimme niin älykkäitä, että pystyisimme koskaan ratkaisemaan näitä arvoituksia. Asian edessä on vain nöyrryttävä ja tunnustettava oma vajavaisuutensa.

        Ja mitä evoluutioon tulee, niin on aivan samantekevää kuinka kehitys tapahtuu. Me emme ole edes niin älykkäitä, että osaisimme selittää puuttuvien lenkkien puutteen, saati molekyylien liikkeen. On siis turha puhua evoluutiosta tieteenä. Se perustuu uskomuksiin siinä kuin muutkin metafyysiset olettamukset. Tieteen yksi perusperiaatteista on mitattavuus. Se mitä ei voida mitata, ei ole siis tiedettä.

        Lopuksi Herramme Jeesuksen sana, joka on tarkoitettu juuri sinulle. Lue se ajatuksella. Se ei hyökkää sinua vastaan vaan haluaa sinulle hyvää. Älä sinäkään siis hyökkää sitä vastaan.

        "Tulkaa minun luokseni kaikki te työn ja kuormien uuvuttamat. Minä annan teille levon. Ottakaa minun ikeeni harteillenne ja katsokaa minua: minä olen sydämeltäni lempeä ja nöyrä. Näin teidän sielunne löytää levon. Minun ikeeni on hyvä kantaa ja minun kuormani on kevyt." Matt. 11: 28-30


        Ystävällisin terveisin, Mikko Rantalahti (mikko.rantalahti[at]gmail.com)

        "Olen koulutukseltani kemian DI ja filosofian opettaja"

        Olen biokemian ammattilainen.

        "Sen sijaan, kun mennään tarpeeksi syvälle molemmissa, niin huomataan monia yhtäläisyyksiä."

        Onko sinulla tullut mieleen, että mahdollisesti yrität sovittaa ristiriitaiset asiat toisiinsa haluamillasi tulkinnoilla? Välillä tulkitset toiseen suuntaan ja välillä toiseen suuntaan.

        "Molekyylitasolla asiat tapahtuvat teoriassa sattumalta, mutta se johtuu vain siitä, ettemme osaa kutsua sitä muuksi kuin sattumaksi."

        Ne tapahtuvat tilastollisella tasolla. Myöhemmin mainitsemasi Heisenbergin epätarkkuusperiaate koskee kvanttitasoa ja molekyyleissä voimme siitä huolimatta tehdä avaruudellisen tilastollisen jakauman. Näiden pohjalta muodostuu molekyyliorbitaalit, jotka eivät ole puhtaasti satunnaisia vaan tilastollisia ja niiden perusteella voimme hyvin pätevästi ennustaa molekyylien käyttäytymistä. Siitä me emme voi olla täysin varmoja, onko aitoa sattumaa edes olemassa, mutta samalla tavalla me emme voi olla mistään fysikaalisen maailman ilmiöstä täysin varmoja, koska aina voidaan keksiä jokin todisteeton kokonaisuus ilmiön taustalle. Joudumme siis tyytymään siihen, miltä asiat meille näyttää absoluuttisen tiedon sijaan. Me kutsumme sitä sattumaksi, koska meille se näyttäytyy sattumana ja siihen me joudumme tyytymään tällä hetkellä.

        "Emme pysty määrittelemään molekyylien liikkeitä, vaikka ne tapahtuvat silmiemme edessä."

        Kyllä pystymme vaikka emme pystykään siihen absoluuttisen varmasti, kuten emme pysty määrittelemään yhtään mitään muutakaan paitsi abstraktien logiikoiden sisäisiä tapahtumia, jotka pohjautuvat meidän määrittelemiin lähtökohtiin (esim. matematiikassa).

        "Kuinka pystyisimme siis määrittelemään maailmankaikkeuden liikkeitä aikojen alussa? "

        Havainnoimalla kehitystä ja näkemällä siinä ne säännönmukaisuudet, joiden mukaan kehitys kulkee. Siten voimme mennä ajassa taakse päin mallissamme.

        "Energian määrä maailmankaikkeudessa on vakio."

        Sen on arvioitu olevan 0. Materia on positiivinen energia ja painovoima negatiivinen.

        "Energiaa ei voi syntyä tyhjästä, se vain muuttaa muotoaan. "

        Näyttäisi siltä, että pystyy, vaikkakin hetkellisesti (kvanttifluktuaatio).

        http://www.astrosociety.org/pubs/mercury/31_02/nothing.html

        "On turha kuvitella, että olisimme niin älykkäitä, että pystyisimme koskaan ratkaisemaan näitä arvoituksia. Asian edessä on vain nöyrryttävä ja tunnustettava oma vajavaisuutensa. "

        Samalla mentaliteetilla olisi historiassa jäänyt ratkaisematta monta tieteellistä ongelmaa. Varsinkin, kun se, mitä sanot olevan turha kuvitella on modernissa fysiikassa kehittynyt viimeisen sadan vuoden aikana aika paljon.

        "Ja mitä evoluutioon tulee, niin on aivan samantekevää kuinka kehitys tapahtuu."

        Ei todellakaan ole. Sillä on ihan käytännön merkitystä hyvin monilla osa-alueilla, lääketieteestä tietotekniikkaan. Evoluutiota ymmärtämällä on kehitetty paljon käytännön sovelluksia ja evoluutioteoria ja evoluution ymmärtäminen ovat ehkä merkittävimpiä teorioita biologian alalla.

        "Me emme ole edes niin älykkäitä, että osaisimme selittää puuttuvien lenkkien puutteen, saati molekyylien liikkeen."

        Kyllä me osaamme selittää puuttuvien linkkien puutteen. Puute johtuu siitä, että kaikista eliöistä ei säily fossiileja ja kaikkia fossiileja ei ole vielä löydetty. Silti näitä linkkejä löytyy jatkuvasti ja lisäksi vielä ennusteiden mukaan. Paraatiesimerkki siitä on Tiktaalik.

        "Se perustuu uskomuksiin siinä kuin muutkin metafyysiset olettamukset."

        Höpöhöpö. Se perustuu todisteisiin. On kummallista, miten sinä voit olla koulutuksestasi huolimatta noin pihalla koko asiasta.

        "Tieteen yksi perusperiaatteista on mitattavuus. Se mitä ei voida mitata, ei ole siis tiedettä. "

        Kyllä voidaan mitata ja mitataankin jatkuvasti. Tässä yksi hyvä esimerkki: http://en.wikipedia.org/wiki/E._coli_long-term_evolution_experiment


      • Quicunque
        ertert kirjoitti:

        "Olen koulutukseltani kemian DI ja filosofian opettaja"

        Olen biokemian ammattilainen.

        "Sen sijaan, kun mennään tarpeeksi syvälle molemmissa, niin huomataan monia yhtäläisyyksiä."

        Onko sinulla tullut mieleen, että mahdollisesti yrität sovittaa ristiriitaiset asiat toisiinsa haluamillasi tulkinnoilla? Välillä tulkitset toiseen suuntaan ja välillä toiseen suuntaan.

        "Molekyylitasolla asiat tapahtuvat teoriassa sattumalta, mutta se johtuu vain siitä, ettemme osaa kutsua sitä muuksi kuin sattumaksi."

        Ne tapahtuvat tilastollisella tasolla. Myöhemmin mainitsemasi Heisenbergin epätarkkuusperiaate koskee kvanttitasoa ja molekyyleissä voimme siitä huolimatta tehdä avaruudellisen tilastollisen jakauman. Näiden pohjalta muodostuu molekyyliorbitaalit, jotka eivät ole puhtaasti satunnaisia vaan tilastollisia ja niiden perusteella voimme hyvin pätevästi ennustaa molekyylien käyttäytymistä. Siitä me emme voi olla täysin varmoja, onko aitoa sattumaa edes olemassa, mutta samalla tavalla me emme voi olla mistään fysikaalisen maailman ilmiöstä täysin varmoja, koska aina voidaan keksiä jokin todisteeton kokonaisuus ilmiön taustalle. Joudumme siis tyytymään siihen, miltä asiat meille näyttää absoluuttisen tiedon sijaan. Me kutsumme sitä sattumaksi, koska meille se näyttäytyy sattumana ja siihen me joudumme tyytymään tällä hetkellä.

        "Emme pysty määrittelemään molekyylien liikkeitä, vaikka ne tapahtuvat silmiemme edessä."

        Kyllä pystymme vaikka emme pystykään siihen absoluuttisen varmasti, kuten emme pysty määrittelemään yhtään mitään muutakaan paitsi abstraktien logiikoiden sisäisiä tapahtumia, jotka pohjautuvat meidän määrittelemiin lähtökohtiin (esim. matematiikassa).

        "Kuinka pystyisimme siis määrittelemään maailmankaikkeuden liikkeitä aikojen alussa? "

        Havainnoimalla kehitystä ja näkemällä siinä ne säännönmukaisuudet, joiden mukaan kehitys kulkee. Siten voimme mennä ajassa taakse päin mallissamme.

        "Energian määrä maailmankaikkeudessa on vakio."

        Sen on arvioitu olevan 0. Materia on positiivinen energia ja painovoima negatiivinen.

        "Energiaa ei voi syntyä tyhjästä, se vain muuttaa muotoaan. "

        Näyttäisi siltä, että pystyy, vaikkakin hetkellisesti (kvanttifluktuaatio).

        http://www.astrosociety.org/pubs/mercury/31_02/nothing.html

        "On turha kuvitella, että olisimme niin älykkäitä, että pystyisimme koskaan ratkaisemaan näitä arvoituksia. Asian edessä on vain nöyrryttävä ja tunnustettava oma vajavaisuutensa. "

        Samalla mentaliteetilla olisi historiassa jäänyt ratkaisematta monta tieteellistä ongelmaa. Varsinkin, kun se, mitä sanot olevan turha kuvitella on modernissa fysiikassa kehittynyt viimeisen sadan vuoden aikana aika paljon.

        "Ja mitä evoluutioon tulee, niin on aivan samantekevää kuinka kehitys tapahtuu."

        Ei todellakaan ole. Sillä on ihan käytännön merkitystä hyvin monilla osa-alueilla, lääketieteestä tietotekniikkaan. Evoluutiota ymmärtämällä on kehitetty paljon käytännön sovelluksia ja evoluutioteoria ja evoluution ymmärtäminen ovat ehkä merkittävimpiä teorioita biologian alalla.

        "Me emme ole edes niin älykkäitä, että osaisimme selittää puuttuvien lenkkien puutteen, saati molekyylien liikkeen."

        Kyllä me osaamme selittää puuttuvien linkkien puutteen. Puute johtuu siitä, että kaikista eliöistä ei säily fossiileja ja kaikkia fossiileja ei ole vielä löydetty. Silti näitä linkkejä löytyy jatkuvasti ja lisäksi vielä ennusteiden mukaan. Paraatiesimerkki siitä on Tiktaalik.

        "Se perustuu uskomuksiin siinä kuin muutkin metafyysiset olettamukset."

        Höpöhöpö. Se perustuu todisteisiin. On kummallista, miten sinä voit olla koulutuksestasi huolimatta noin pihalla koko asiasta.

        "Tieteen yksi perusperiaatteista on mitattavuus. Se mitä ei voida mitata, ei ole siis tiedettä. "

        Kyllä voidaan mitata ja mitataankin jatkuvasti. Tässä yksi hyvä esimerkki: http://en.wikipedia.org/wiki/E._coli_long-term_evolution_experiment

        Kiitos vastauksistasi. Ne olivat asiallisia ja rakentavia ja opin niistä taatusti uutta. Lupaan tutustua tarkemmin antamiisi linkkeihin ja perusteluihin paremmalla ajalla, kunhan jaksan paremmin keskittyä. Tänään alkaa olla jo kello liian paljon. :)


      • Quicunque
        Schlechterwisser kirjoitti:

        "Huomaa Schlechterwisser, että toin esille myös muita näkökulmia kuin tieteellisen. Ajattelin vain puhua sillä kielellä, kun täällä oli monia, jotka tuntuvat olevan siitä hyvin perillä ja ymmärtävät tieteen kieltä parhaiten."

        Mihin asiaan vastasit tällä kommentilla? Esitin sinulle viestissäni monta kysymystä, mutta en oikein ymmärrä mihin niistä tämä vastaa.

        "Voimme toki nojautua auktoriteetteihin tai erilaisiin teorioihin…"

        Kuiten Raamattuun ja Jumalaan vai? Miksi niihin pitäisi nojautua? Minun mielestäni auktoriteetit saavat todistaa väitteensä ihan samalla tavalla kuin muutkin. Nojaudut viesteissäsi Raamattuun ja puhut Jeesuksesta ikään kuin ne olisivat totta, ja evoluutiosta ikään kuin se ei olisi varmaa. Minkä auktoriteetin perusteella olet päätynyt noihin mielipiteisiisi? Raamatun ja Puolimatkan?

        Kysyin jo äsken: Jos emme koskaan voi saada selville millä tavoin maailma on syntynyt, niin miksi emme voi vain sanoa ettemme tiedä? Miksi pitäisi alkaa uskomaan ja väittämään ilman mitään perusteita, että maailman on luonut joku tonttu tai muu myyttinen olento?

        "Sinun lähtökohtasi on varmasti erilainen, mutta samaa siinä on mielestäni ainakin se, että kyse on sinun omasta elämästäsi ja mitä haluat sen kanssa tehdä."

        Kas, siinähän onkin jo paljon samaa, kyse on molempien omasta elämästä. Minä olin nuorempana hyvin samanlainen kuin sinä nyt, ja saatoin innokkaasti uskoa asioihin joista ei ollut mitään todisteita. Luulin olevani parempi ja viisaampi kuin muut (vaikka muille vähättelinkin arvoani ja korostin nöyryyttäni), ja jaoin mielelläni neuvoja muille, ja kerroin omista kokemuksistani, koska luulin että ne olisivat muillekin tärkeitä ja merkityksellisiä.

        Nyt vanhempana olen ymmärtänyt elämästä ja todellisuudesta paljon enemmän, ja tajunnut millaisessa hybriksessä nuorempana elin. Nyt tiedän, että emme tiedä paljon mitään, ja voimme vain yrittää hankkia mahdollisimman paljon, mahdollisimman hyvää tietoa, jotta voimme järjestää asiat niin, että kaikilla meillä olisi parempi elää. Oloni on nyt paljon tasapainoisempi ja tyynempi, ja ahdistuksista ei ole tietoakaan. Toivot että sinäkin joskus vielä kehityt tähän tilaan.

        Ja sitten uudelleen näitä kysymyksiä joita unohdit kommentoida:

        Joko olet ymmärtänyt, että energian kokonaismäärä maailmankaikkeudessa voi ihan hyvin olla nolla?

        Mikä olikaan se "jyrkän tieteellinen" osuus väitteistäsi, joka tekee biologiasta metafysiikaa?

        Millaisia todisteita täsmälleen ottaen vielä haluaisit evoluutiosta?

        Luin Aatami9:n laittamat tiedot energian nollamäärästä ja opin paljon uutta. Pitää tutustua vielä tarkemmin aiheeseen muistakin lähteistä. Äärimmäisen mielenkiintoinen aihe se joka tapauksessa on. Siinä sekoittuvat filosofia ja luonnontieteet sopivan haastavalla tavalla ja mikäs sen mukavampaa. Laajennan mielelläni tietämystäni.

        Jyrkän tieteellinen osuus oli loogisen positivismin peruja mittauksien vaatimuksista ja havainnoinnista. Itse en ole sitä mieltä, vaan pidän tieteenä monia muitakin aloja kuin luonnontieteitä. Se ei kylläkään tainnut viesteistäni välittyä, kun yritin pitäytyä logiikan tyylissä. Viimeisin viestini oli sen sijaan täysin minulta itseltäni peräisin, kuten siinä mainitsinkin ja taisin muuten mainita siitäkin, että biologiaa pidän hyvinkin tärkeänä tieteen alana.

        Evoluutiosta voit kertoa niin paljon kuin haluat. Kuten jo aiemmin sanoin, laajennan mielelläni tietämystäni.

        Jatkan mielelläni keskustelua kanssasi, mutta mieluummin sähköpostin välityksellä, jos sinulle sopii. Mielestäni se toimii tällaisen keskustelun kanavana huomattavasti paremmin kuin julkinen keskustelupalsta. Mikäli et halua keskustella kanssani omalla nimelläsi, niin ehkä voisit luoda sähköpostiosoitteen keksityllä nimellä. Se sopii minulle vallan mainiosti.

        Siunattua yötä ja kaikkea hyvää, Mikko Rantalahti (mikko.rantalahti[at]gmail.com)


      • Quicunque
        Quicunque kirjoitti:

        Kiitos vastauksistasi. Ne olivat asiallisia ja rakentavia ja opin niistä taatusti uutta. Lupaan tutustua tarkemmin antamiisi linkkeihin ja perusteluihin paremmalla ajalla, kunhan jaksan paremmin keskittyä. Tänään alkaa olla jo kello liian paljon. :)

        Vedin aika paljon mutkia suoriksi puhuessani esim. Heisenbergin epävarmuusteoriasta. En kokenut aiheelliseksi tuoda koko teoriaa julki, koska siihenhän voi jokainen tutustua haluamastaan lähteestä. Olen kanssasi täysin samaa mieltä siitä, että voimme tilastollisesti ennustaa molekyylien käyttäytymistä, mutta emme varmuudella, kuten itsekin mainitsit. Todennäköisyyksiinhän laskelmamme perustuvatkin ja kun puhutaan todennäköisyyksistä, niin aina jää varaa epätodennäköisyyksille. Varman tiedon tulee olla aukotonta, jos siis puhutaan varmuudesta määritelmästä käsin.

        "Ei todellakaan ole. Sillä on ihan käytännön merkitystä hyvin monilla osa-alueilla, lääketieteestä tietotekniikkaan. Evoluutiota ymmärtämällä on kehitetty paljon käytännön sovelluksia ja evoluutioteoria ja evoluution ymmärtäminen ovat ehkä merkittävimpiä teorioita biologian alalla."

        Tarkoitin, että on ihan sama miten asiat maailmassa tapahtuvat. Sillähän ei ole merkitystä, koska ne tapahtuvat niin kuin tapahtuvat. Niin vain on ja näitä tuloksia voimme hyödyntää monilla eri aloilla, kuten toitkin esille. Kaikki kehittyy ja niin on hyvä. Minun ei kannata ryhtyä väittelemään evoluutiosta sen kummemmin, koska minusta se on luonnollista kehitystä siinä kuin vaikkapa lihaskunnon parantaminen. Minulle kyse onkin mahtavasta "designista" kaiken taustalla. :)

        "Havainnoimalla kehitystä ja näkemällä siinä ne säännönmukaisuudet, joiden mukaan kehitys kulkee. Siten voimme mennä ajassa taakse päin mallissamme"

        Puhuit myös matemaattisesta varmuudesta ja mallin sisällähän me pystymme tekemään kaikki tarvittavat oletukset ja todistukset virheettömästi. Ongelmaksi asia muodostuu siinä vaiheessa, kun meidän pitäisi tehdä tulkintoja mallin ulkopuolelta. Tällöin samat oletukset eivät enää päde vaan niitä on muokattava. On helpompi muokata teoriaa kuin todellisuutta. Tätä tarkoitan myös sillä, että meidän on tunnustettava vajavaisuutemme. Voimme lähestyä totuutta ja on hyvä, että siihen pyritään, mutta lähestyttäessä aukottomuutta joudumme jatkuvasti muokkaamaan lähtökohtiamme ja teoriaamme. Joudumme siis aina mallintamaan kaiken uudelleen ja uudelleen ja kohtaamme taas samat ongelmat.

        Energian on arvioitu olevan nolla ja se perustuu itse asiassa voiman ja vastavoiman lakiin, kuten antamastasi linkistäkin käy ilmi. Jos on olemassa voima, niin sillä on myös vastavoima. Tämä jo klassisesta mekaniikasta tuttu Newtonin laki on nyt otettu hyötykäyttöön. Tämä on kuitenkin harhaanjohtavaa siinä mielessä, että vaikka kokonaisenergia on nolla, niin on olemassa toisiaan kumoavia voimia. Voimat (tai energia) ovat siis joko negatiivisia tai positiivisia ja niitä on olemassa. Ne vain kumoavat toisensa, josta siis voimme muodostaa matemaattisesti yhteenlaskun tuloksena nollan.


      • Quicunque
        Quicunque kirjoitti:

        Vedin aika paljon mutkia suoriksi puhuessani esim. Heisenbergin epävarmuusteoriasta. En kokenut aiheelliseksi tuoda koko teoriaa julki, koska siihenhän voi jokainen tutustua haluamastaan lähteestä. Olen kanssasi täysin samaa mieltä siitä, että voimme tilastollisesti ennustaa molekyylien käyttäytymistä, mutta emme varmuudella, kuten itsekin mainitsit. Todennäköisyyksiinhän laskelmamme perustuvatkin ja kun puhutaan todennäköisyyksistä, niin aina jää varaa epätodennäköisyyksille. Varman tiedon tulee olla aukotonta, jos siis puhutaan varmuudesta määritelmästä käsin.

        "Ei todellakaan ole. Sillä on ihan käytännön merkitystä hyvin monilla osa-alueilla, lääketieteestä tietotekniikkaan. Evoluutiota ymmärtämällä on kehitetty paljon käytännön sovelluksia ja evoluutioteoria ja evoluution ymmärtäminen ovat ehkä merkittävimpiä teorioita biologian alalla."

        Tarkoitin, että on ihan sama miten asiat maailmassa tapahtuvat. Sillähän ei ole merkitystä, koska ne tapahtuvat niin kuin tapahtuvat. Niin vain on ja näitä tuloksia voimme hyödyntää monilla eri aloilla, kuten toitkin esille. Kaikki kehittyy ja niin on hyvä. Minun ei kannata ryhtyä väittelemään evoluutiosta sen kummemmin, koska minusta se on luonnollista kehitystä siinä kuin vaikkapa lihaskunnon parantaminen. Minulle kyse onkin mahtavasta "designista" kaiken taustalla. :)

        "Havainnoimalla kehitystä ja näkemällä siinä ne säännönmukaisuudet, joiden mukaan kehitys kulkee. Siten voimme mennä ajassa taakse päin mallissamme"

        Puhuit myös matemaattisesta varmuudesta ja mallin sisällähän me pystymme tekemään kaikki tarvittavat oletukset ja todistukset virheettömästi. Ongelmaksi asia muodostuu siinä vaiheessa, kun meidän pitäisi tehdä tulkintoja mallin ulkopuolelta. Tällöin samat oletukset eivät enää päde vaan niitä on muokattava. On helpompi muokata teoriaa kuin todellisuutta. Tätä tarkoitan myös sillä, että meidän on tunnustettava vajavaisuutemme. Voimme lähestyä totuutta ja on hyvä, että siihen pyritään, mutta lähestyttäessä aukottomuutta joudumme jatkuvasti muokkaamaan lähtökohtiamme ja teoriaamme. Joudumme siis aina mallintamaan kaiken uudelleen ja uudelleen ja kohtaamme taas samat ongelmat.

        Energian on arvioitu olevan nolla ja se perustuu itse asiassa voiman ja vastavoiman lakiin, kuten antamastasi linkistäkin käy ilmi. Jos on olemassa voima, niin sillä on myös vastavoima. Tämä jo klassisesta mekaniikasta tuttu Newtonin laki on nyt otettu hyötykäyttöön. Tämä on kuitenkin harhaanjohtavaa siinä mielessä, että vaikka kokonaisenergia on nolla, niin on olemassa toisiaan kumoavia voimia. Voimat (tai energia) ovat siis joko negatiivisia tai positiivisia ja niitä on olemassa. Ne vain kumoavat toisensa, josta siis voimme muodostaa matemaattisesti yhteenlaskun tuloksena nollan.

        Antamastasi linkistä kvanttifluktuaation suhteen kiinnitin huomiota siihen, missä puhuttiin tyhjyydestä syntyvästä energiasta. Syntyy siis materiaa ja antimateriaa kvanttifluktuaation seurauksena ja otollisissa olosuhteissa nämä ovat jääneet henkiin ja muodostaneet valtavan energiapiikin, josta maailmankaikkeus on saanut alkunsa. Mielenkiintoista teoriassa oli varsinkin se, jossa ehdotettiin, että maailmankaikkeuksia saattaisi olla useita. Mielenkiintoista siitä tekee myös se, että jos ajatellaan tyhjyyttä, niin se on ääretöntä. Tyhjää on siis äärettömän paljon. Tämä taas aiheuttaa sen, että myös kvanttifluktuaatioita täytyy tapahtua ääretön määrä jatkuvasti. Äärettömyyteen taas sisältyy sellainen ominaisuus, että siinä on kaikki aika ja kaikki tapahtumat. Tästä johtuen universumeja tulisi siis syntyä koko ajan, mikäli tuntemamme universumi olisi syntynyt teoriassa esitetyllä tavalla. Vaikka niitä syntyisi vain yksi äärettömän kaukana tyhjyydessä, niin sen voima vaikuttaisi myös meidän universumiimme ja ne ryhtyisivät vetämäään toisiaan puoleensa. Tyhjässä voima vaikuttaa koko ajan samaan suuntaan, jos ei mikään muu voima ryhdy siihen vaikuttamaan. Meidän tulisi siis lähestyä koko ajan muita universumeja. Myös, jos ajatellaan universumimme reuna-alueita (siis aluetta, joka on juuri ja juuri universumin ulkopuolella, eli siis tyhjää universumin vieressä), niin myös niillä tapahtuisi jatkuvia kvanttifluktuaatioita lähes ääretön määrä. Tämä taas aiheuttaisi uusien universumien syntymisen aivan oman universumimme reuna-alueilla tai jopa oman universumimme sisällä, koska siis kvanttifluktuaatioita tapahtuu koko ajan joka paikassa.

        Mielestäni on kyseenalaista vetää liian suuria johtopäätöksiä liian pienistä teorioista, kuten itsekin edellisessä kappaleessa esim. tein. Halusin vain osoittaa sen, että voimme yrittää lokeroida ja järjestää kaiken mahdollisimman hyvin ja pyrkiä kontrolloimaan kaikkea. Liialliset johtopäätökset tieteellisistä teorioista ovat toisaalta myös hyvä lähtökohta pohtimiselle, mutta useimmiten teorioissa puhutaan aina epävarmoin sanankääntein, kuten vaikkapa antamassasi linkistä (http://www.astrosociety.org/pubs/mercury/31_02/nothing.html) käy ilmi. Tämän vuoksi niistä ei mielestäni pitäisi vetää liiallisia johtopäätöksiä varman tiedon suhteen.


      • Quicunque kirjoitti:

        Luin Aatami9:n laittamat tiedot energian nollamäärästä ja opin paljon uutta. Pitää tutustua vielä tarkemmin aiheeseen muistakin lähteistä. Äärimmäisen mielenkiintoinen aihe se joka tapauksessa on. Siinä sekoittuvat filosofia ja luonnontieteet sopivan haastavalla tavalla ja mikäs sen mukavampaa. Laajennan mielelläni tietämystäni.

        Jyrkän tieteellinen osuus oli loogisen positivismin peruja mittauksien vaatimuksista ja havainnoinnista. Itse en ole sitä mieltä, vaan pidän tieteenä monia muitakin aloja kuin luonnontieteitä. Se ei kylläkään tainnut viesteistäni välittyä, kun yritin pitäytyä logiikan tyylissä. Viimeisin viestini oli sen sijaan täysin minulta itseltäni peräisin, kuten siinä mainitsinkin ja taisin muuten mainita siitäkin, että biologiaa pidän hyvinkin tärkeänä tieteen alana.

        Evoluutiosta voit kertoa niin paljon kuin haluat. Kuten jo aiemmin sanoin, laajennan mielelläni tietämystäni.

        Jatkan mielelläni keskustelua kanssasi, mutta mieluummin sähköpostin välityksellä, jos sinulle sopii. Mielestäni se toimii tällaisen keskustelun kanavana huomattavasti paremmin kuin julkinen keskustelupalsta. Mikäli et halua keskustella kanssani omalla nimelläsi, niin ehkä voisit luoda sähköpostiosoitteen keksityllä nimellä. Se sopii minulle vallan mainiosti.

        Siunattua yötä ja kaikkea hyvää, Mikko Rantalahti (mikko.rantalahti[at]gmail.com)

        "Jyrkän tieteellinen osuus oli loogisen positivismin peruja mittauksien vaatimuksista ja havainnoinnista."

        Aivan, mutta minulle jäi epäselväksi se, mikä osa tekstistäsi perustui jyrkkään tieteellisyyteen. Eli mikä osa tekstistäsi oli se tieteellinen argumentti evoluutioteoriaa vastaan?

        "Jatkan mielelläni keskustelua kanssasi, mutta mieluummin sähköpostin välityksellä, jos sinulle sopii. Mielestäni se toimii tällaisen keskustelun kanavana huomattavasti paremmin kuin julkinen keskustelupalsta."

        Tätäkin keskustelupalstaa lukee moni ihminen, joka on kiinnostunut näistä asioista, hakee tukea ajatuksille ja ehkä perusteita sille, miten maailmankuvaa pitäisi rakentaa. Mielestäni teemme samalla vaivalla paljon hyvää työtä antaessamme muidenkin lukea argumenttejamme, ja halutessaan osallistua keskusteluun. Se on mielestäni paljon hedelmällisempää kuin kahden ihmisen yksityinen sähköpostittelu. Ja koska aikani on rajallista, olen pahoillani, mutta aion jatkossakin rajata kirjoitteluni vain tälle julkiselle foorumille.


      • Quicunque
        Schlechterwisser kirjoitti:

        "Jyrkän tieteellinen osuus oli loogisen positivismin peruja mittauksien vaatimuksista ja havainnoinnista."

        Aivan, mutta minulle jäi epäselväksi se, mikä osa tekstistäsi perustui jyrkkään tieteellisyyteen. Eli mikä osa tekstistäsi oli se tieteellinen argumentti evoluutioteoriaa vastaan?

        "Jatkan mielelläni keskustelua kanssasi, mutta mieluummin sähköpostin välityksellä, jos sinulle sopii. Mielestäni se toimii tällaisen keskustelun kanavana huomattavasti paremmin kuin julkinen keskustelupalsta."

        Tätäkin keskustelupalstaa lukee moni ihminen, joka on kiinnostunut näistä asioista, hakee tukea ajatuksille ja ehkä perusteita sille, miten maailmankuvaa pitäisi rakentaa. Mielestäni teemme samalla vaivalla paljon hyvää työtä antaessamme muidenkin lukea argumenttejamme, ja halutessaan osallistua keskusteluun. Se on mielestäni paljon hedelmällisempää kuin kahden ihmisen yksityinen sähköpostittelu. Ja koska aikani on rajallista, olen pahoillani, mutta aion jatkossakin rajata kirjoitteluni vain tälle julkiselle foorumille.

        Itseltäni ei löydy riittävästi arvovaltaa tieteellisen kritiikin esittämiseen, mutta tässä olisi luento professori John Waltonilta aiheeseen liittyen:

        http://edinburghcreationgroup.org/originoflife.xml

        Luento kestää lähes tunnin, joten kannattaa varata popcornia ja limpparia. :)


      • Quicunque kirjoitti:

        Itseltäni ei löydy riittävästi arvovaltaa tieteellisen kritiikin esittämiseen, mutta tässä olisi luento professori John Waltonilta aiheeseen liittyen:

        http://edinburghcreationgroup.org/originoflife.xml

        Luento kestää lähes tunnin, joten kannattaa varata popcornia ja limpparia. :)

        Ei kun kysyin, että mikä oli se "jyrkän tieteellinen" näkökulma, jonka SINÄ sanoit antaneesi. Minua ei välttämättä kiinnosta kuunnella tuntia samoja vanhoja kreationistisia argumentteja, jota on tullut luettua jo jonkin verran tässä vuosien varrella. Koska kyse oli tieteellisistä argumenteista, niin palataan asiaan vaikka sitten, kun sinulla on linkittää meille edes yhteen kreationistien tekemään tieteelliset kriteerit täyttävään tutkimukseen asiasta.

        Linkittämäsi "tieteellistä arvovaltaa omaava" henkilö on sitä paitsi teologi, ja eikös se niin ollut, että kun teologiaa ei voi mitata, niin se ei voi olla tiedettä?


      • Quicunque
        Schlechterwisser kirjoitti:

        Ei kun kysyin, että mikä oli se "jyrkän tieteellinen" näkökulma, jonka SINÄ sanoit antaneesi. Minua ei välttämättä kiinnosta kuunnella tuntia samoja vanhoja kreationistisia argumentteja, jota on tullut luettua jo jonkin verran tässä vuosien varrella. Koska kyse oli tieteellisistä argumenteista, niin palataan asiaan vaikka sitten, kun sinulla on linkittää meille edes yhteen kreationistien tekemään tieteelliset kriteerit täyttävään tutkimukseen asiasta.

        Linkittämäsi "tieteellistä arvovaltaa omaava" henkilö on sitä paitsi teologi, ja eikös se niin ollut, että kun teologiaa ei voi mitata, niin se ei voi olla tiedettä?

        Rakas ystävä. Kuten jo edellä mainitsin. Minulla ei ole kompetenssia antaa tieteellistä evoluutiokritiikkiä muuten kuin nojautumalla auktoriteetteihin.

        Tässä faktoja luennoitsijasta. Mielestäni ne osoittavat hänen pätevyytensä tieteen parissa. En itse asiassa edes tiennyt hänen olevan teologi. Kiitos, että kerroit.

        John C. Walton

        B.Sc., Ph.D., D.Sc.
        Chartered Chemist
        Fellow of the Royal Society of Chemistry
        Fellow of the Royal Society of Edinburgh
        Research Professor of Chemistry University of St. Andrews

        Jos et jaksa katsoa koko luentoa, niin ehdottaisin, että katsot alusta 5 min ja lopusta 5 min (ja keskeltä 50 min :D). Alussa hän esittelee aiheen ja kertoo perustelut luennon pitämiselle. Lopussa ovat johtopäätökset.


      • Quicunque kirjoitti:

        Rakas ystävä. Kuten jo edellä mainitsin. Minulla ei ole kompetenssia antaa tieteellistä evoluutiokritiikkiä muuten kuin nojautumalla auktoriteetteihin.

        Tässä faktoja luennoitsijasta. Mielestäni ne osoittavat hänen pätevyytensä tieteen parissa. En itse asiassa edes tiennyt hänen olevan teologi. Kiitos, että kerroit.

        John C. Walton

        B.Sc., Ph.D., D.Sc.
        Chartered Chemist
        Fellow of the Royal Society of Chemistry
        Fellow of the Royal Society of Edinburgh
        Research Professor of Chemistry University of St. Andrews

        Jos et jaksa katsoa koko luentoa, niin ehdottaisin, että katsot alusta 5 min ja lopusta 5 min (ja keskeltä 50 min :D). Alussa hän esittelee aiheen ja kertoo perustelut luennon pitämiselle. Lopussa ovat johtopäätökset.

        Katsoin jonkin verran alusta. Ainakaan se parikymmenminuuttinen ei käsitellyt millään tavalla evoluutiota, joka on tieteellinen fakta eikä evoluutioteoriaakaan. Se käsitteli elämän syntyä, joka oli myös luennon nimi.


      • Quicunque
        Aatami9 kirjoitti:

        Katsoin jonkin verran alusta. Ainakaan se parikymmenminuuttinen ei käsitellyt millään tavalla evoluutiota, joka on tieteellinen fakta eikä evoluutioteoriaakaan. Se käsitteli elämän syntyä, joka oli myös luennon nimi.

        Kiitokset Aatami9 huomautuksesta. En muistanutkaan, että minun piti antaa kritiikkiä juuri evoluutioteoriaa vastaan. Luento kylläkin sivuaa aihetta, vaikkei sitä kohtaan suoraan asetukaan. Mutta eikös kaikki liity kaikkeen. :)

        Luento on mielenkiintoinen ja siihen kannattaa sinunkin tutustua ihan loppuun asti, jos olet jaksanut jo noin pitkälle. Tsemppiä! :)¨

        Samoilta sivuilta (http://edinburghcreationgroup.org/) muuten löytyy monenlaista materiaalia luennoista artikkeleihin. Silmäilin nopeasti muidenkin luennoitsijoiden ansioluetteloita ja ihan päteviltä tyypeiltä he vaikuttavat noin niin kuin tieteenkin näkökulmasta, kun sitä kerran täällä aina vaaditaan.


      • Quicunque kirjoitti:

        Rakas ystävä. Kuten jo edellä mainitsin. Minulla ei ole kompetenssia antaa tieteellistä evoluutiokritiikkiä muuten kuin nojautumalla auktoriteetteihin.

        Tässä faktoja luennoitsijasta. Mielestäni ne osoittavat hänen pätevyytensä tieteen parissa. En itse asiassa edes tiennyt hänen olevan teologi. Kiitos, että kerroit.

        John C. Walton

        B.Sc., Ph.D., D.Sc.
        Chartered Chemist
        Fellow of the Royal Society of Chemistry
        Fellow of the Royal Society of Edinburgh
        Research Professor of Chemistry University of St. Andrews

        Jos et jaksa katsoa koko luentoa, niin ehdottaisin, että katsot alusta 5 min ja lopusta 5 min (ja keskeltä 50 min :D). Alussa hän esittelee aiheen ja kertoo perustelut luennon pitämiselle. Lopussa ovat johtopäätökset.

        "En itse asiassa edes tiennyt hänen olevan teologi."

        Hups, anteeksi. Luulin että kyseessä oli John H. Walton, joka on kreationismista kirjoitellut teologi. Tämä oli paha huolimattomuusvirhe, olen pahoillani!


      • Quicunque
        Schlechterwisser kirjoitti:

        "En itse asiassa edes tiennyt hänen olevan teologi."

        Hups, anteeksi. Luulin että kyseessä oli John H. Walton, joka on kreationismista kirjoitellut teologi. Tämä oli paha huolimattomuusvirhe, olen pahoillani!

        Kein problem Schlechterwisser. :)


    • D.Arvi N.

      ...ja Jumala.
      No kaikilla on tietysti tärkeysjärjestyksensä.

    Ketjusta on poistettu 0 sääntöjenvastaista viestiä.

    Luetuimmat keskustelut

    1. Nykynuoret puhuu nolosti englantia suomen sekaan, hävetkää!

      Kamalan kuuloista touhua. Oltiin ravintolassa ja viereen tuli 4 semmosta 20-25v lasta. Kaikki puhui samaan tyyliin. Nolo
      Maailman menoa
      193
      4384
    2. 57-vuotiads muka liian vanha töihin?

      On tämä sairas maailma. Mihin yli 55-vuotiaat sitten muka enää kelpaavat? Hidasta itsemurhaa tekemään, kun eläkkeelle ei
      Maailman menoa
      240
      2375
    3. 133
      1657
    4. Luovutetaanko nainen?

      En taida olla sinulle edes hyvän päivän tuttu. Nauratkin pilkallisesti jo selän takana.
      Ikävä
      67
      1351
    5. Haluatteko miellyttää kumppaninne silmää?

      Entä muita aisteja? Mitä olette valmiita tekemään sen eteen että kumppani näkisi teissä kunnioitettavan yksilön? Olette
      Sinkut
      181
      1127
    6. J-miehelle toivon

      Hyvää yötä. Voisiko nykyistä tilannetta uhmaten vielä pienintäkään toivetta olla, päästä kainaloosi joskus lepääämään.
      Ikävä
      80
      1066
    7. Miten olette lähestyneet kiinnostuksen kohdettanne?

      Keskusteluita seuranneena tilanne tuntuu usein olevan sellainen, että palstan anonyymit kaipaajat eivät ole koskaan suor
      Ikävä
      64
      1064
    8. Haluatko S

      vielä yrittää?
      Ikävä
      57
      1000
    9. Onko kaivattunne suosittu?

      Onko teillä paljon kilpailijoita? Mies valitettavasti näyttää olevan paljonkin naisten suosiossa :(
      Ikävä
      76
      1000
    10. By the way, olet

      mielessäni. Olet minulle tärkeä, niin suunnattoman tärkeä. En kestäisi sitä jos sinulle tapahtuisi jotain. Surullani ei
      Ikävä
      71
      985
    Aihe